Sunteți pe pagina 1din 221

CLINICAL & TEACHING CASES

OBSTETRICS & GYNECOLOGY

www.apgo.org

APGO COPYRIGHT
The APGO Medical Student Educational Objectives, 8th edition, may be utilized as follows:
1. Faculty/residents/students /institutions may purchase hard copies for personal and general student
use. The book may be photocopied for teaching purposes, but may not be retyped, altered or
otherwise manipulated in any way. APGO must be acknowledged in all photocopied material.
2. The online Objectives may be printed out, copied and utilized as is, but may not be retyped, altered
or otherwise manipulated in any way. APGO must be acknowledged in all photocopied material.
3. The PDF outlines may be printed out and used as is, but may not be retyped, altered or otherwise
manipulated in any way. APGO must be acknowledged in all photocopied material
4. The online cases in Word may be manipulated to suit faculty purposes. APGO need not be
acknowledged.

TABLE OF CONTENTS / LINKS TO INDIVIDUAL CASES


1.
2.
3.
4.
5.
6.
7.
8.
9.
10.
11.
12.
13.
14.
15.
16.
17.
18.
19.
20.
21.
22.
23.
24.
25.
26.
27.
28.
29.
30.
31.
32.
33.
34.

History
Examination
Pap Smear and Cultures
Personal Interaction / Communication Skills
Legal Issues / Ethics in Obstetrics & Gynecology
Preventive Care and Health Management
Maternal-Fetal Physiology
Preconception Care
Antepartum Care
Intrapartum Care
Immediate Care of the Newborn
Postpartum Care
Lactation
Ectopic Pregnancy
Spontaneous Abortion
Medical & Surgical Conditions in pregnancy
Preeclampsia-Eclampsia Syndrome
Isoimmunization
Multifetal Gestation
Fetal Death
Abnormal Labor
Third-Trimester Bleeding
Preterm Labor
Premature Rupture of Membranes
Intrapartum Fetal Surveillance
Postpartum hemorrhage
Postpartum Infection
Anxiety and Depression
Postterm Pregnancy
Fetal Growth Abnormalities
Obstetric Procedures
Contraception and Sterilization
Abortion
Vulvar and Vaginal Disease

35.
36.
37.
38.
39.
40.
41.
42.
43.
44.
45.
46.
47.
48.
49.
50.
51.
52.
53.
54.
55.
56.
57.

Sexually Transmitted Infections & Urinary Tract Infections


Pelvic Relaxation & Urinary Incontinence
Endometriosis
Chronic Pelvic Pain
Disorders of the Breasts
Gynecological Procedures
Puberty
Amenorrhea
Hirsutism and Virilization
Normal and Abnormal Uterine Bleeding
Dysmenorrhea
Climacteric / Menopause
Infertility
Premenstrual Syndrome & Premenstrual Dysphoric Disorder
Gestational Trophoblastic Neoplasia
Vulvar Neoplasms
Cervical Disease and Neoplasia
Uterine Leiomyomas
Endometrial Carcinoma
Ovarian Neoplasms
Sexuality and Modes of Sexual Expression
Sexual Assault
Domestic Violence

UNIT ONE:

APPROACH TO THE PATIENT

1. History
2. Examination
3. Pap Smear and Cultures
4. Personal Interaction / Communication Skills
5. Legal Issues / Ethics in Obstetrics & Gynecology
6. Preventive Care and Health Management

TEACHING TOOL 1

History
This tool is designed to help the teacher lead an interactive session during which the
students collectively develop a history taking format they can practice and internalize
throughout the rotation. It is geared to taking a complete gynecologic history.
Approximate duration: 60 minutes
Resources needed: Large blackboard or erasable board vs. 2 to 3 flip charts
Suggested curricular timing: First day or week of the rotation
Rationale for inclusion in the curriculum:
1. History taking gets little formal curricular attention
2. It is a skill students must have
3. It is a skill that students can actually do, giving them a legitimate role on the
health care team and building self-esteem
4. Opportunities to record things in charts is often limited
5. Many institutions have adopted pre-printed history check-lists into their
formal medical records, encouraging students to become passive box checkers,
rather than active information gatherers. In attempts to be concise, these forms
often are poorly organized and leave little room for the students to record the
patients actual story.
QUESTION 1:
What is the first thing that gets recorded in a patients history?
ANSWER:
The chief complaint.
QUESTION 2:
Outside of prenatal care, why do patients come to see an obstetrician-gynecologist?
ANSWER:

Going around the room, the students create a list of around 30-40 reasons girls and
women seek gynecologic care. It helps set the stage for what they will be expected to
learn the rest of the rotation. Their list typically includes:
Annual exams
Abnormal Pap smears
Missed periods
Heavy periods
Painful periods
Bleeding between periods
Irregular periods
Needs contraception
Contraceptive complaints
Pelvic Pain
Dysmenorrhea
Dyspareunia
Vaginal discharge
Vaginal itching
Vaginal odor
Ovarian cysts

Menopause
Infertility
Pre-conception counseling
STD checks
Vulvar pain
Vulvar lesions
Breast pain
Breast masses
Postmenopausal bleeding
Leiomyomata
Bartholins duct cysts
Pelvic prolapse
Urinary incontinence
Post-operative care
Wound infections
Sexual assault

QUESTION 3: What comes next in the history?


ANSWER: The HPI.
Indicate that each item they have listed has its own unique HPI.
Describe what you want to hear at the beginning of every HPI, i.e.
The patient is a 26 year old gravida X para Y with a last menstrual period on
DATE here today for . . .
Introduce helpful mnemonics, i.e. An Alphabet of Pain for the HPI
P = Pain
Q = Quality
R = Region and radiation
S = Severity
T = Timing
U = Undiagnosed?
V = Vomiting and other associated symptoms
W = What makes it better /worse

OR, the ten Bs of the post-partum visit

Bladder
Bowel
Bottom (episiotomy/lacerations)
Bleeding (lochia /periods)
Breasts
Blues
Birth control
Boinking (OK-not technically acceptable, but it starts with a B)
Baby (doing well?)
Beaten/battered
QUESTION 4: What is the next component of the history during this rotation?
Answer:

The past ob-gyn history

Then present this to the students in the format and order you want them to collect and
recorded during the rotation.
QUESTION 5: What are the next components of the history that you will need to collect?
Answer:

Record these on the board in the order you want them collected and
presented during the rotation:
Past medical history
Past surgical history
Past psychiatric history
Medications
Medication allergies
Family history
Social history

QUESTION 6: Small group activity. Break the students into the following four groups:
1.
2.
3.
4.

Past medical history


Past surgical history
Past psychiatric history
Family history

Each group generates a list of ten common diseases or conditions that they can
specifically ask their patients about in their assigned area. When all of the
groups are done (5-10 minutes), they report these back to the group as a whole.
Teacher records lists on the board.

Past Medical History - Lists typically include:


Asthma
Allergies
Seizure disorders
UTIs /pyelo
Hypertension
Diabetes

Headaches
SLE
URIs
Thyroid diseases
DVTs/ PE
Anemia

IBD
GERD
Hepatitis
CVA
Coagulopathies
Hypercholesterolemia

From this list (limit to 10), brainstorm why each disease matters to a practicing
ob-gyn. Issues typically include associated menstrual abnormalities, symptoms
mimicking gynecologic conditions, potentially teratogenic therapies, high-risk
pregnancy status, operative risks and impact on birth control decisions.

Past Surgical History (non-gynecologic) - Lists typically include:


Appendectomy
Tonsillectomy
Pressure equalization tubes
ACL repair
Breast biopsy
Cholecystectomy

Tooth extractions
Thyroidectomy
Hernia repair
Correction of congenital anomalies
Carpal tunnel release
Plastic surgery

From this list (limit to 10), brainstorm why each disease matters to a practicing
ob-gyn. Issues typically include anesthetic complications, bleeding
abnormalities, blood transfusions and post-operative intra-abdominal
adhesions.

Past Psychiatric History - Lists typically include:


Depression
Anorexia
Bulimia
OC disorder
Psychosis

Substance abuse
Schizophrenia
Anxiety disorders
Suicidality
Post-traumatic stress disorders

From this list (limit to 10), brainstorm why each disease matters to a practicing
ob-gyn. Issues typically include associated menstrual abnormalities, symptoms
mimicking gynecologic conditions, therapy during pregnancy and lactation, birth
control options and associations with prior abuse histories.

Family History - Lists typically include:

Thyroid diseases
Hypertension
Diabetes
Cancers
Birth defects
Coagulopathies

CVA
DVTs /PE
Inherited anemias
Chromosomal abnormalities
CAD
Hypercholesterolemia

From this list (limit to 10), brainstorm why each disease matters to a practicing
ob-gyn. Issues typically include inheritance patterns and impacts on personal
health care decision-making.

QUESTION 7: What do you routinely include in the social history?


ANSWER:

Alcohol use
Illicit drug use
Tobacco use
Occupation
Family and living situation
Exercise
Domestic violence

If time permits, review the myriad impacts of smoking on reproductive health, i.e.
Menstrual irregularity
Premature menopause
Infertility
Miscarriage
IUGR
PROM
PTL/PTD
Cervical dysplasia/carcinoma

Conclusion:
Show the students the template they have collectively developed. Encourage them to
practice using it over and over and over again so that it is completely internalized by
the end of the rotation.

CLINICAL CASE 2

Examination
Pelvic exam/Annual exam
At the end of this exercise the student will be able to:
o Discuss the indications for pelvic exam
o Understand the important components of the pelvic exam
o Know the indications for additional exam of the anterior and posterior
vaginal wall
o Describe the importance of the rectal/vaginal exam.
52-year-old female comes in for annual gynecologic exam. She stopped menstruating
two years ago and has experienced hot flashes and some insomnia. She, however, feels
she is doing well and is not interested in HRT. Menarche was at age 13, menses were
regular until last year prior to cessation of menses, but then she stopped and has had no
more bleeding since that time. She had two pregnancies and two vaginal births; her
children are 25 and 28.
Her ROS is negative except for occasional loss of a small amount of urine in her aerobics
class; if she is careful to empty her bladder prior to class, this isnt a problem. She is
sexually active and needs to use a lubricant; however, sex is not painful.
PMH is negative except for an appendectomy and breast biopsy, which was negative.
She is married in a good relationship with no history of abuse and she exercises
3x/week in aerobics class or walks. She drinks a glass of wine with dinner and has
never smoked or used illicit drugs. She drinks 3 glasses of milk/day. Family history is
significant for mother with osteoporosis, sister with hypothyroidism and father with
elevated cholesterol.
Physical exam
Normal appearing female 120/70, weight 130, height 55
Thyroid-WNL; Lungs- Clear; COR- RRR no murmur or gallops; Breast exam breasts are
symmetrical with fibrocystic changes in the upper out section of each breast; no distinct
masses are noted with evidence of small scar on R breast; Abdomen soft non-tender;
Ext genitalia normal with evidence of midline scar consistent with vagina no lesions,
slightly atrophic; Cervix appears to be WNL and Pap smear obtained. The speculum is
taken apart and the lower blade of the speculum is used by placing in the vagina
pressing downward to evaluate the anterior wall of the vagina. There is evidence of
small cystocele and urethrocele, which descends slightly with Valsalva, but not to the
introitus. There is no loss of urine during this maneuver. Bimanual reveals an

anteverted uterus with no adnexal masses palpate. In fact, the adnexa are not well
appreciated, recto-vaginal exam is negative, confirms the vaginal exam and the stool is
heme negative.
You assure the patient that her exam is WNL. You begin counseling her re: lifestyle
issues, encourage her to continue exercising regularly and perhaps add some weights to
further decrease risk of osteoporosis. You counsel her regarding her needs for 1200 mg
of calcium/day. Additionally, she should consider either a sigmoidoscopy or
colonoscopy for colorectal screening. A mammogram is ordered, as well as screening
cholesterol and thyroid screen. She is counseled regarding her Kegel exercises and need
to keep bladder empty during exercise and to do Kegels to strengthen pelvic floor. She
is sent out with a slip for cholesterol screening, thyroid screening, urinalysis and a set of
three fecal occult blood cards.
Discussion/teaching points
The annual exam is more than just a breast exam and pelvic exam. It is the opportunity
to assess a patients overall health and health care needs. The same exam/laboratory
tests are not done on each patient. One needs to consider age and risk factors when
deciding what to do. This patient is 52-years-old; therefore, it is important to include
colorectal screening and counseling in the exam. She should also have annual
mammography, counsel regarding calcium intake and consider thyroid screening.
Although SBE has not been shown to decrease morbidity or mortality from breast
cancer, one should inquire if the patient does SBE and offer to teach her if she doesnt
know how to do this.
PE findings of urethrocele and mild cystocele are very common in older women who
have had children. This is best detected on physical exam using a Sims speculum and
having the patient Valsalva. This allows one to examine the movement and degree of
prolapse of the anterior and posterior vaginal wall, as well as the cervix. A ureterocele,
cystocele and rectocele do not necessarily require surgical repair. Mild symptoms of
stress urinary incontinence, such as this patient reports, need to be addressed and
usually respond well to behavioral measures, such as keeping the bladder empty during
exercise and minimizing caffeinated beverages and to pelvic strengthening exercises or
Kegels.
On pelvic exam, the adnexa were not well appreciated, which is normal in
postmenopausal women. Often the adnexa are not felt and, if they are palpated, might
raise concern and necessitate further evaluation if thought to be enlarged. The
rectal/vaginal is done to confirm the findings of the vaginal exam and is important in
this age patient to evaluate for rectal cancers. Approximately 10% of rectal cancers can
be detected with rectal exam. However, it is important to add fecal occult blood testing,
which should not only be done at the time of the exam, but the patient also needs to do
this at home on three consecutive bowel movements. This will increase detection of all
colorectal cancers.

TEACHING CASE 3
Pap Smear and Cultures
Student Handout
Clinical Case:
JA is a 22-year-old G0 female who presents for an annual exam. She had an abnormal
Pap smear 4 years ago followed by a colposcopy but no biopsies were performed. Her
follow-up Pap smears were normal and the last one was 2 years ago. She is currently
sexually active with a new partner for the last 2 months, and has had four lifetime
partners. She does not report any history of sexually transmitted disease. She is
currently using birth control pills for contraception. She is just finishing her menstrual
cycle and is spotting; her periods are regular without problems. Her review of systems
is negative. She smokes one pack of cigarettes per week; she does not drink and states
that she has occasionally used marijuana. She is a freshman in college and is majoring in
business. Her vitals and physical exam are normal.
Discussion Questions:
1. What are the different techniques available to obtain a pap smear?
2. In addition to performing a Pap smear, what cultures might you recommend for
this patient, and how do you proceed in obtaining them?
3. How often does this patient need to undergo a Pap smear?
4. What risk factors does this patient have for cervical dysplasia and cancer?
5. If while performing the speculum exam, you notice a gross lesion on the cervix,
what would your next step in management be?
6. What other counseling or advice you need to discuss with this patient?

References:
Obstetrics and Gynecology by Beckmann 5th Edition, 2006; Chapter 1 Health Care for Women. Pages 1624.
Essentials of Obstetrics and Gynecology by Hacker and Moore 4th Edition, 2004; Chapter 2 Clinical
Approach to the Patient. Pages 14-18.

Pap Smear and Cultures


Preceptor Handout

The Pap smear is one of the most effective screening tests used in medicine today. Proper
technique in performing the Pap smear and obtaining specimens for microbiologic culture
will improve accuracy.
The APGO Educational Objectives related to this topic are the following:
A. Perform an adequate Pap smear*
B. Obtain specimens to detect sexually transmitted diseases*
C. Handle specimens properly to improve diagnostic accuracy*
D. Provide an explanation to the patient regarding the purpose of these tests*

*Designated as Priority One in the APGO Medical Student Educational Objectives, 8 th


Edition

Pap Smear and Cultures


Preceptor Handout

Clinical Case:
JA is a 22-year-old G0 female who presents for an annual exam. She had an abnormal
Pap smear 4 years ago followed by a colposcopy but no biopsies were performed. Her
follow-up Pap smears were normal and the last one was 2 years ago. She is currently
sexually active with a new partner for the last 2 months, and has had four lifetime
partners. She does not report any history of sexually transmitted disease. She is
currently using birth control pills for contraception. She is just finishing her menstrual
cycle and is spotting; her periods are regular without problems. Her review of systems
is negative. She smokes one pack of cigarettes per week; she does not drink and states
that she has occasionally used marijuana. She is a freshman in college and is majoring in
business. Her vitals and physical exam are normal.

Discussion Questions:
1. What are the different techniques available to obtain a pap smear?
a. Discuss conventional Pap smear and Thin Prep: Both collected from the
endocervical canal and transformation zone with spatula and cytobrush.
Do not use any gel prior to performing the Pap smear.
b. Thin prep has a higher sensitivity rate
c. Discuss the importance of properly labeling specimens and sending off to
promptly to the lab.
2. In addition to performing a Pap smear, what cultures might you recommend for
this patient, and how do you proceed in obtaining them?
a. Since the patient is sexually active with a new partner, you should offer
her screening for sexually transmitted diseases including performing
endocervical cultures or DNA probe for gonorrhea and Chlamydia.
b. After performing the Pap smear, you wipe any excess cervical mucous
and you insert the cotton swab to collect the cultures or the DNA probe in
the endocervical canal.

3. How often does this patient need to undergo a Pap smear?


a. Patients who have had an abnormal Pap smear previously, will need
more frequent Pap smears based on the actual abnormality. For this
patient, since it appears that she had normal follow-up, she needs to
continue to have annual Pap smears assuming this one is normal.
b. For monogamous patients who have had 3 normal Pap smears in a row,
may space out the Pap smears to once every 2-3 years.
4. What risk factors does this patient have for cervical dysplasia and cancer?
a. Multiple partners, sexual activity at an early age, history of abnormal Pap
smear, smoking.
5. If while performing the speculum exam, you notice a gross lesion on the cervix,
what would your next step in management be?
a. Any gross lesion on the cervix will need to be directly biopsied as Pap
smears have a false negative rate and cervical cancer might be missed.
6. What other counseling or advice you need to discuss with this patient?
a. Discuss the importance of cervical cancer screening guidelines
b. Discuss the importance of safe sex practices

References:
Obstetrics and Gynecology by Beckmann 5th Edition, 2006; Chapter 1 Health Care for Women. Pages 1624.
Essentials of Obstetrics and Gynecology by Hacker and Moore 4th Edition, 2004; Chapter 2 Clinical
Approach to the Patient. Pages 14-18.

CLINICAL CASE 4

Personal Interaction and Communication Skills


A 30-year-old female returns to the busy hospital clinic to get the results of her Pap
smear. She has had a Pap smear every other year since age 20 all with normal results.
Reviewing her chart, you see that she has been married for three years and has no
children. She smokes two packs of cigarettes a week and is currently on oral
contraceptives. She has no history of sexually transmitted diseases and appears to have
a very conservative sexual history.
Your supervising resident has been rushing from room to room and is now heading
your way. Upon questioning, you explain to the resident that you understand the
diagnosis and the next course of action. You enter the room and the patient is sitting in
the chair. With both of you standing in front of the patient, the conversation between
the resident and the patient is as follows:
Resident: Hi, Im Dr. Jones and this is _______, one of our medical students.
Patient: Hello
Resident: (Full eye contact with the patient) As you know, we always have patients return to
get the results of their Pap smear when things dont turn out as well as we would like.
Patient: (Complete silence; looks concerned)
Resident: (Speaking rather rapidly, steps back slightly, crosses arms, maintains eye contact) In
your case, the results show HGSIL that means a high-grade squamous intraepithelial lesion.
Patient: (Remains silent; squirms a little in her chair; now looks frightened)
Resident: While I cant say exactly what caused this result, I can tell you that several factors
can play role, including smoking, oral contraceptive use, experience with multiple sexual
partners and, of course, having a male partner with multiple consorts.
Patient: (Mutters while staring at the floor) I didnt know that.
Resident: (Maintains eye contact, except for glancing at you a couple of times) Our standard
protocol is to have a colposcopy with endocervical curettage and directed biopsies as indicated.
There really isnt anything to worry about. On your way out, please stop at the front desk to
arrange your appointment. Of course, if you have any questions after you get home, please feel
free to call the clinic. (Looking at the clock on the wall), Im sorry I have to run, but we have a
very full clinic this morning.
Patient: (Looking both upset and puzzled) OK, doctor.
The resident glances at you and leaves the room. You quickly assess what you need to do next.

Diagnosis/Management
The table below is an evaluation of how well the resident interacted with this patient.
Look closely at this evaluation and determine whether you agree with it. If not, how
would you evaluate the interaction? In other words, how would you diagnosis the
interaction and what would be your management plan for such an interaction?
Skill
Establish rapport
Informed patient of the
clinical diagnosis
Gave reasons as to
possible etiology for
clinical outcome
(educated patient).
Open door for further
questions
Informed the patient of
management steps
Worked with other
members of health team

Evidence in Exchange
Introduced self and
medical student
Told her of HGSIL

Performed Skill Y/N


Yes

Told patient several


etiologies that are
epidemiologically related
to HGSIL
Explained she could call
the clinic anytime
Asked her to schedule an
appointment after telling
her what needs to be done
Implied clinic personnel
could handle her
questions

Yes
Yes

Yes
Yes
Yes

To help you with your decision-making, consider the follow items:


1.
2.
3.
4.
5.
6.
7.

How do you think the resident would answer you if you asked how the
visit with this patient went? In other words, what do you think would be
the residents perception of this conversation?
What do you believe would be the patients perception of this
conversation?
Given the setting and atmosphere, would you approach the patient
differently?
What was done to establish rapport with the patient? Could more have
been done?
What can you glean from the patients body language and from what she
said or, perhaps, didnt say?
Did the resident treat the whole patient?
While what the resident said may be accurate, what impact might it
have on the patient and why? (Suggest you take it comment by comment).

If you had to evaluate this information exchange, what would be your criteria?
Construct a new table with your criteria and apply it to this case scenario.
Now that you have thought all of this through, what would you do if you were the
medical student who has been left alone with the patient when the resident exited?

Teaching points:
1.

This quotable quote from Quotes in Medicine on American College of


Physicians-American Society of Internal Medicine Web site
(www.acponline.org) says much about the importance of communication.

The most important difference between a good and indifferent clinician lies in the amount
of attention paid to the story of a patient. Farquhar Buzzard
2.

Effective communication between physician and patient is similar to


communication between good friends or partners. This statement can be
a discussion catalyst among students.

Some of the Dos for effective communication in both cases are:

Use I statements to express yourself (rather than saying we or


you)

Be specific not only with medical information but with feeling


type comments

Show respect, warmth and caring

Be genuine

Be an active listener

Maintain eye contact

Give feedback this includes repeating or paraphrasing what the


patient (or friend/partner) has said so both of you know what was
really said
Some of the Donts for effective communication in both cases are:

Dont assume you know what your patient (friend/partner) is


thinking

Dont use yes/no questions unless they are necessary

Dont use labels (especially cultural stereotypes, assumptions)

Dont act disinterested or be impatient

Dont overload the patient (friend/partner)


Students can generate these and additional points and apply them to the case scenario.

TEACHING CASE 5
Legal and Ethical Issues in Obstetrics and Gynecology
Student Handout
Clinical Case:
A 33 year-old G2P1 Caucasian patient at 33 weeks gestation presents to your office for
her scheduled prenatal visit. She reports no problems. Her psychosocial history is
unremarkable. Her prior pregnancy resulted in a cesarean delivery with a transverse
incision. There were no complications associated with the delivery. Her physical
examination reveals normal blood pressure and weight. Fundal height, fetal position,
and heart rate are unremarkable. Her diagnostic test results show Hgb: 12.4 g/dL; WBC:
11,000; Urinalysis: negative for bacteria and leucocytes; and urine drug screen:
negative.
Discussion Questions:
1. What would you tell this patient regarding her options for delivery?
2. What are the underlying ethical principles in informed consent?
3. What is the role of the physician in informed consent?
4. What is the role of the patient?
5. What topics might be included in an informed consent at this time?
6. What should be done if the patient declines a trial of labor after a history of lowtransverse c-section?

References:
Obstetrics and Gynecology by Beckmann 5th Edition, 2006; Chapter 2 Ethics in Obstetrics and
Gynecology. Pages 25-29.
Essentials of Obstetrics and Gynecology by Hacker and Moore 4th Edition, 2004;
Chapter 1 Practice Management and Ethics in Obstetrics and Gynecology. Pages 3-8.
Chapter 5 Clinical Performance Improvement: Assessing the Quality and Safety of Womens Health
Care. Pages 46-54.
Gabbe SG, Niebyl JR, Simpson JL. Obstetrics: Normal and Problem Pregnancies (4th ed.). Philadelphia:
Churchill-Livingstone, 2002. pp. 146-150, 585-587.

Legal and Ethical Issues in Obstetrics and Gynecology


Preceptor Handout

Rationale: Legal obligations to protect patients interests are effective only if understood
and applied. Recognizing and understanding the basis of ethical conflicts in obstetrics and
gynecology will allow better patient care and prevent critical errors in treatment
planning.
The APGO Educational Objectives related to this topic are the following:
A. Explain the issues involved in informed consent
B. Demonstrate the role of confidentiality in clinical activities
C. List the local laws requiring the reporting of suspected child abuse and
domestic violence
D. Discuss the legal and ethical issues in the care of minors
E. Describe issues of justice relating to access to obstetric- gynecologic care
F. Explain the basis of ethical conflict in maternal-fetal medicine
G. Discuss ethical issues raised by induced abortion, contraception and
reproductive technology

*Designated as Priority One in the APGO Medical Student Educational Objectives, 8 th


Edition

Legal and Ethical Issues in Obstetrics and Gynecology


Preceptor Handout
Clinical Case:
A 33 year-old G2P1 Caucasian patient at 33 weeks gestation presents to your office for
her scheduled prenatal visit. She reports no problems. Her psychosocial history is
unremarkable. Her prior pregnancy resulted in a cesarean delivery with a transverse
incision. There were no complications associated with the delivery. Her physical
examination reveals normal blood pressure and weight. Fundal height, fetal position,
and heart rate are unremarkable. Her diagnostic test results show Hgb: 12.4 g/dL; WBC:
11,000; Urinalysis: negative for bacteria and leucocytes; and urine drug screen:
negative.

Discussion Questions:
1. What would you tell this patient regarding her options for delivery?

Trial of labor for a vaginal birth after a c-section (VBAC)


Cesarean delivery

2. What are the underlying ethical principles in informed consent?

Autonomy or self-determination the patient has the ultimate say


Rational decision-making requires disclosure of information by both the
physician and the patient

3. What is the role of the physician in informed consent?

Establish a relationship conducive to the sharing of information and trust


Be certain the patient (and when appropriate her partner) is well-informed
Ensure the patient has, under ordinary circumstances, reasonable time to think
about the information provided by the physician
Be open to questions from the patient and the patients partner
Document the process as well as the decision

4. What is the role of the patient?

To participate in the decision-making process


To provide accurate and complete information
To ask questions

5. What topics might be included in an informed consent at this time?

Management of labor and associated risks


Possible interventions
Risks and benefits that can be reviewed in detail
Specifics include:
o VBAC or Cesarean section
o Benefits and risks of a trial of labor
o IV fluids
o Fetal monitoring
o Labor augmentation
o Use of forceps and vacuum delivery

6. What should be done if the patient declines a trial of labor after a history of lowtransverse c-section (VBAC)?

A woman should not be coerced to experience a VBAC


Generally acknowledged that trial of labor for a VBAC is safe with a high success
rate
Almost one-half of eligible patients request an elective repeat procedure
o To avoid labor-related pain
o Convenience of scheduling
o Geographic, social factors may play a role

References:
Obstetrics and Gynecology by Beckmann 5th Edition, 2006; Chapter 2 Ethics in Obstetrics and
Gynecology. Pages 25-29.
Essentials of Obstetrics and Gynecology by Hacker and Moore 4th Edition, 2004;
Chapter 1 Practice Management and Ethics in Obstetrics and Gynecology. Pages 3-8.
Chapter 5 Clinical Performance Improvement: Assessing the Quality and Safety of Womens Health
Care. Pages 46-54.
Gabbe SG, Niebyl JR, Simpson JL. Obstetrics: Normal and Problem Pregnancies (4th ed.). Philadelphia:
Churchill-Livingstone, 2002. pp. 146-150, 585-587.

TEACHING CASE 6
Preventive Care and Health Maintenance
Student Handout

Clinical Case:
A 51-year old G3P3 woman comes to the office for a health maintenance exam. She has
no concerns. She is in good health. She had three normal vaginal deliveries. She is
sexually active with her husband and has been using condoms for contraception. She
has no history of abnormal Pap smears or sexually transmitted diseases. Her last Pap
smear was one year ago.Her cycles are irregular as she only had 4 menstrual periods
last year. Her last menstrual period was 2 months ago. She is not taking any
medications. Her family history is significant for a maternal aunt who was diagnosed
with ovarian cancer at age 60. On examination, she has normal vital signs. Her heart,
lungs and abdominal exams are normal. On pelvic examination, she has normal
external genitalia, normal vagina and cervix. On bimanual exam, she has a slightly
enlarged uterus and no palpable adnexal masses. Rectovaginal exam confirms.

Discussion Questions:
1. What is the next step in the management of this patient?
2. Does this patient need to continue using contraception?
3. Does this patient need a bone density scan?
4. What are your recommendations regarding the frequency of Pap Smears for this
patient?
5. How would the recommendations for the Pap Smear be different if this patient
was 32 years of age?

References:
Obstetrics and Gynecology by Beckmann 5th Edition, 2006; Chapter 1 Health Care for Women. Pages 311.
Essentials of Obstetrics and Gynecology by Hacker and Moore 4th Edition,2004; Chapter 2 Clinical
Approach to the Patient. Pages 9-10.

Preventive Care and Health Maintenance


Preceptor Handout

The student will recognize the value of routine health surveillance as a part of health
promotion and disease prevention.

The APGO Educational Objectives related to this topic are the following:
A. List age-appropriate screening procedures and recommended time intervals for
mammogram, bone density, Pap tests, STI evaluation and other screening tests*
B. Counsel patient regarding contraception, domestic abuse/ violence and
prevention of sexually transmitted infections*

*Designated as Priority One in the APGO Medical Student Educational Objectives, 8 th


Edition

Preventive Care and Health Maintenance


Preceptor Handout
Clinical Case:
A 51-year old G3P3 woman comes to the office for a health maintenance exam. She has
no concerns. She is in good health. She had three normal vaginal deliveries. She is
sexually active with her husband and has been using condoms for contraception. She
has no history of abnormal Pap smears or sexually transmitted diseases. Her last Pap
smear was one year ago. Her cycles are irregular as she only had 4 menstrual periods
last year. Her last menstrual period was 2 months ago. She is not taking any
medications. Her family history is significant for a maternal aunt who was diagnosed
with ovarian cancer at age 60. On examination, she has normal vital signs. Her heart,
lungs and abdominal exams are normal. On pelvic examination, she has normal
external genitalia, normal vagina and cervix. On bimanual exam, she has a slightly
enlarged uterus and no palpable adnexal masses. Rectovaginal exam confirms.
Discussion Questions:
1. What is the next step in the management of this patient?
This patient is perimenopausal as she appears to be having anovulatory cycles. As
long as she is not experiencing nay bothering symptoms (such as hot flashes), there
is no need for any interventions. Since she has always had normal Pap smears and
her last one was a year ago, she does not need a Pap smear during this visit. She
needs to have a yearly mammogram, and needs to have a sigmoidoscopy or
colonoscopy if she has not had one yet. Her cholesterol levels need to be checked
if she has not had a check in last 3 years or they had been abnormal in the past.
Other laboratory studies such as glucose levels or thyroid function tests can be
checked if there are any related symptoms.
2. Does this patient need to continue using contraception?
In theory, this patient can still get pregnant since she is still menstruating. However,
the irregular cycles indicate perimenopausal status and the extremely low
likelihood of pregnancy.
3. Does this patient need a bone density scan?
At this age, the patient does not need a bone density scan as she does not have any
additional risk factors for osteoporosis. In addition to age, gender, and race, risk
factors for osteoporosis include: Family or personal history of fractures as an adult,
bone structure and body weight, menopause and menstrual history, lifestyle, certain
medications and chronic diseases.

4. What are your recommendations regarding the frequency of Pap Smears for this
patient?
This patient needs a Pap Smear every 3 years since she has never had an abnormal
Pap Smear and her last one was one year ago.

5. How would the recommendations for the Pap smear be different if this patient was
32 years of age?
They would not be any different if she did not have any additional risk factors.

References:
Obstetrics and Gynecology by Beckmann 5th Edition, 2006; Chapter 1 Health Care for Women. Pages 311.
Essentials of Obstetrics and Gynecology by Hacker and Moore 4th Edition,2004; Chapter 2 Clinical
Approach to the Patient. Pages 9-10.

UNIT TWO:
SECTION A

OBSTETRICS - Normal Obstetrics

7.

Maternal-Fetal Physiology

8.

Preconception Care

9.

Antepartum Care

10. Intrapartum Care


11. Immediate Care of the Newborn
12. Postpartum Care
13. Lactation

CLINICAL CASE 7

Maternal-Fetal Physiology
At the conclusion of this exercise, the student will be able to:
1.
Calculate the most accurate delivery date for a woman with regular
menstrual periods
2.
Explain the importance of preconceptional vitamins
3.
Describe the physiological reasons for amenorrhea, weight gain, GI
symptoms, lowered hematocrit, dyspnea and glucosuria during normal
gestation
4.
Educate the parturient about the onset of labor
5.
List 2 risks of the postdate pregnancy
A 22-year-old married woman calls your office because she is 4 days late for her
menstrual period. She has told your nurse that her menstrual periods are always
between 27-30 days apart, and usually last 4-5 days. She and her husband have not
been using contraception for 4 months. She has performed a home urine pregnancy test
and it is positive. How likely is it that the home test is incorrect?
She makes an appointment to be seen in your office for her first prenatal visit within the
next 2 weeks. You advise her to continue her prenatal vitamins that she has been taking
for the last 3 months. What component of the prenatal vitamins is the most important in
preventing birth defects?
Even though this is her first pregnancy, she understands that she will not have further
menstrual periods until after delivery. What is the physiological explanation for
amenorrhea during pregnancy?
She also understands that she will gain weight during pregnancy. What is a normal
weight gain for an average-sized woman and what are the components of that weight
gain?
Her initial prenatal visit is uneventful and a 6-week intrauterine gestation is confirmed.
Within the next two weeks, she notes the onset of early morning nausea, heartburn and
reflux. Later in the pregnancy, she experiences constipation and the development of
hemorrhoids. What hormone is most closely related to these pregnancy-related
concerns, and why?
Her initial hematocrit of 36% drops to 33% at 28 weeks gestation in spite of regular
prenatal vitamin and supplemental iron therapy. Why?
At her 20-week visit, she describes some dyspnea at rest, but no exercise intolerance,
saying she can still run 3-5 miles daily. What physiological process could explain this
dyspnea?

At her 24-week gestational age visit, she describes urinary frequency and her urinalysis
at that visit shows a trace of glucose. Can these changes be normal during pregnancy?
Should she be screened for diabetes at this time?
At that same visit, she wants to discuss the process of labor. She is not sure she
understands when to come to the hospital when contractions start, and wants to
confirm her due date. She also asks what will happen if she does not delivery by her
due date.

Teaching points:
It is important for health care workers to recognize the normal physiological changes
that occur to the woman during pregnancy. It is also critical to recognize those signs
and symptoms that may occur during pregnancy that are NOT normal physiological
changes. Because most pregnancies are straightforward and routine, any sign or
symptom that is not one with which you are familiar (if you treat many pregnant
patients), or which you do not find described in any standard treatise about pregnancy
(if you do not routinely treat pregnant patients) should be managed with great care and
with appropriate evaluation. Frequently, maternal or fetal catastrophes may be
prevented or ameliorated by the early detection and treatment begun by the alert
practitioner.

TEACHING CASE 8
Preconception Care
Student Handout

Clinical Case:
You have been Marys doctor for the past 3 years. She is a 39-year-old Caucasian woman
with a BMI of 32.9 who sees you primarily for her idiopathic chronic hypertension,
which is well controlled on an ACE inhibitor. She has smoked 1 pack of cigarettes per
day for the past 20 years. She is in today for her annual exam and mentions that she is
getting married in a few months and would like to start a family. She has never been
pregnant before. Her past medical history is otherwise unremarkable.
On physical exam, her BP=138/84, Ht=5 2, Wt=180 lbs. Otherwise, her exam is
unremarkable.

Discussion Questions:
1. What is the goal of counseling a woman about pregnancy prior to conception?
2. What are the major topics that should be discussed with any woman prior to
conception?
3. For the patient in this case, what specific topics need to be addressed?

References:
Obstetrics and Gynecology by 5th Edition, 2006; Chapter 3 Embryology, Anatomy, and Reproductive
Genetics. Pages 39-47.
Essentials of Obstetrics and Gynecology by Hacker and Moore 4th Edition,2004; Chapter 8 Antepartum
Care: Preconception and Prenatal care, Genetic Evaluation and Teratology, and Antenatal Fetal Assessment.
Pages 83-85.
ACOG Committee Opinion No. 313, September 2005 The Importance of Preconception Care in the
Continuum of Womens Health Care.

Preconception Care
Preceptor Handout

Rationale: The proven benefits of good health prior to conception include a significant
reduction in maternal and fetal morbidity and mortality.
The APGO Educational Objectives related to this topic are the following:
A. Describe how certain medical conditions affect pregnancy*
B. Describe how pregnancy affects certain medical conditions*
C. Counsel patients regarding a history of genetic abnormalities
D. Counsel patients regarding advanced maternal age
E. Counsel patients regarding substance abuse*
F. Counsel patients regarding nutrition and exercise*
G. Counsel patients regarding medications and environmental hazards*
H. Counsel patients regarding immunizations*

*Designated as Priority One in the APGO Medical Student Educational Objectives, 8 th


Edition

Preconception Care
Preceptor Handout
Clinical Case:
You have been Marys doctor for the past 3 years. She is a 39-year-old Caucasian woman
with a BMI of 32.9 who sees you primarily for her idiopathic chronic hypertension,
which is well controlled on an ACE inhibitor. She has smoked 1 pack of cigarettes per
day for the past 20 years. She is in today for her annual exam and mentions that she is
getting married in a few months and would like to start a family. She has never been
pregnancy before. Her past medical history is otherwise unremarkable.
On physical exam, her BP=138/84, Ht=5 2, Wt=180 lbs. Otherwise, her exam is
unremarkable.
Discussion Questions:
1. What is the goal of counseling a woman about pregnancy prior to conception?
This type of counseling is often referred to as preconception care or counseling. The
goal is to optimize, whenever possible, a womans health and knowledge before
planning and conceiving a pregnancy in order to eliminate, or at least reduce, the risk
associated with pregnancy for the women and her future baby. In addition, if pregnancy
is not desired, then current contraceptive use and options can be discussed to assist the
patient in identifying the most appropriate method for her and to reduce the potential
for an unplanned pregnancy.
2. What are the major topics that should be discussed or addressed with any woman
prior to conception?

Identify undiagnosed, untreated or poorly controlled medical conditions


Review immunization history and recommend appropriate
immunizations
Risks of medication and radiation exposure in early pregnancy
Nutritional issues
Family history and genetic history including racial/ethnic background
and specific genetic risks
Tobacco, alcohol, and substance abuse and other high-risk behaviors
(such as sexual activity and risk for STIs)
Occupational and environmental exposures
Social issues
Mental health issues

Counseling may be performed by a provider who is skilled in the care of obstetric

patients. However, the assistance of a maternal-fetal medicine specialist or genetic


specialist may be necessary in certain circumstances.
3. For the patient in this case, what specific topics need to be addressed?
Mary will need to be counseled regarding several preconception issues, including:
Weight loss and exercise (Marys BMI is 32.9 and she is considered to be
obese [BMI 30]; weight loss in obese non-pregnant women has proven
health benefits: for Mary, she may see improvement in her blood pressure
and decrease the need for antihypertensive therapy; obesity in pregnancy
is associated with increased risks including higher rates of gestational
diabetes, preeclampsia, cesarean delivery, anesthesia complications, postoperative complications)
The effect of chronic medical disease (idiopathic hypertension) on
pregnancy (increased risk of preeclampsia, fetal growth restriction,
abruption and recommendations for heightened maternal and fetal
surveillance in pregnancy)
Need to modify antihypertensive therapy (ACE inhibitors are
contraindicated in pregnancy due to risks for fetal renal dysgenesis and
dysfunction)
Effect of smoking on pregnancy (increased risk of fetal growth
restriction)
Offer CF carrier testing (carrier prevalence increased in Caucasians) and
discuss any family history of birth defects or genetic disorders: referral
for genetic counseling may be warranted if issues are identified
Discussion of increased risk of Downs Syndrome and other trisomies
based on current age of 39 and probable older age when she conceives.
Begin prenatal multivitamins or at least folic acid supplementation (0.4
mg per day) for the prevention of fetal neural tube defects
Accurate recording of LMP and cycle length in order to assist in dating
her pregnancy and allow her to present early for prenatal care when she
does conceive
Review immunization history; employment, medical or behavioral risk
factors for infections against which effective vaccines are available; and
test for evidence of immunity against rubella: recommend immunizations
based on your review

References:
Obstetrics and Gynecology by 5th Edition, 2006; Chapter 3 Embryology, Anatomy, and Reproductive
Genetics. Pages 39-47.
Essentials of Obstetrics and Gynecology by Hacker and Moore 4th Edition,2004; Chapter 8 Antepartum
Care: Preconception and Prenatal care, Genetic Evaluation and Teratology, and Antenatal Fetal Assessment.
Pages 83-85.
ACOG Committee Opinion No. 313, September 2005 The Importance of Preconception Care in the
Continuum of Womens Health Care.

TEACHING CASE 9
Antepartum Care
Student Handout
Clinical Case:
Your resident asks you to make a preliminary assessment of a 24-year-old woman who
has presented to the emergency room complaining of vaginal spotting for the past two
days, and which has become heavier today. She says that todays bleeding is more than
a usual period and she became concerned when she passed a large clot. When you enter
the cubicle where she is resting, you notice an anxious, though pleasant, woman sitting
upright on the gurney. She denies fever, chills, abdominal pain or cramping. She says
that she has been urinating more frequently than usual, without pain, and notes fatigue
that she attributes to stress at her work as a pastry chef. She is unable to tell you when
her last menstrual period was since she has had irregular menses since puberty, often
with two to three-month gaps between periods. She has never been pregnant. She tells
you that she and her boyfriend, who plan to marry in the next year, use condoms for
contraception. She has never been diagnosed with a sexually transmitted infection.
The patient is 170 pounds and is 55 tall. On physical exams, her vital signs are stable
and she is not orthostatic. Speculum exam reveals no active bleeding from the cervix,
although there is evidence of old blood in the vaginal vault. The cervical os is closed. No
lesions are present in the vagina or on the vulva. Bimanual exam reveals a slightly
enlarged and globular uterus in mid-position, slightly irregular in contour but nontender; the adnexae are without masses and tenderness.

Discussion Questions:
1. What are the next steps in the assessment of this patient?
2. What potential complications should you discuss with this patient?
3. What would you recommend as further management for this patient?

References:
Obstetrics and Gynecology by Beckmann 5th Edition, 2006; Chapter 5 Antepartum Care. Pages 63-78.
Essentials of Obstetrics and Gynecology by Hacker and Moore 4th Edition, 2004; Chapter 8 Antepartum
Care: Preconception and Prenatal care, Genetic Evaluation and Teratology, and Antenatal Fetal Assessment.
Pages 83-103.

Antepartum Care
Preceptor Handout
Rationale: Antepartum care promotes patient education and provides ongoing risk
assessment and development of an individualized patient management plan.
The APGO Educational Objectives related to this topic are the following:
A. Diagnose pregnancy*
B. Assess gestational age*
C. Distinguish an at-risk pregnancy*
D. Assess fetal growth, well-being, maturity and amniotic fluid volume
E. Describe appropriate diagnostic studies
F. Describe nutritional needs of pregnant women
G. Describe adverse effects of drugs and the environment
H. Perform a physical examination on obstetric patients*
I. Answer commonly asked questions concerning pregnancy and labor and
delivery
J. Counsel women with an unintended pregnancy

*Designated as Priority One in the APGO Medical Student Educational Objectives, 8 th


Edition

Antepartum Care
Preceptor Handout
Clinical Case:
Your resident asks you to make a preliminary assessment of a 24-year-old woman who
has presented to the emergency room complaining of vaginal spotting for the past two
days, and which has become heavier today. She says that todays bleeding is more than
a usual period and she became concerned when she passed a large clot. When you enter
the cubicle where she is resting, you notice an anxious, though pleasant, woman sitting
upright on the gurney. She denies fever, chills, abdominal pain or cramping. She says
that she has been urinating more frequently than usual, without pain, and notes fatigue
that she attributes to stress at her work as a pastry chef. She is unable to tell you when
her last menstrual period was since she has had irregular menses since puberty, often
with two to three-month gaps between periods. She has never been pregnant. She tells
you that she and her boyfriend, who plan to marry in the next year, use condoms for
contraception. She has never been diagnosed with a sexually transmitted infection.
The patient is 170 pounds and is 55 tall. On physical exams, her vital signs are stable
and she is not orthostatic. Speculum exam reveals no active bleeding from the cervix,
although there is evidence of old blood in the vaginal vault. The cervical os is closed. No
lesions are present in the vagina or on the vulva. Bimanual exam reveals a slightly
enlarged and globular uterus in mid-position, slightly irregular in contour but nontender; the adnexae are without masses and tenderness.
Discussion Questions:
1. What are the next steps in the assessment of this patient?
-Evaluate for pregnancy
-Determine whether pregnancy is intrauterine
-Determine whether pregnancy is viable
In a woman of childbearing age who presents with an unknown last menstrual period, it
is important to determine her pregnancy status. A history of irregular menses is not
uncommon, especially in teenagers or in overweight women and, in this setting, a
qualitative urine pregnancy test is crucial. The diagnosis of pregnancy should not be
made on nonspecific signs and symptoms.
Signs and symptoms of pregnancy:
History: Missed periods, Fatigue, Nausea and vomiting, Breast tenderness,
Urinary frequency
Physical Exam: Softening and enlargement of the uterus, congestion and bluish
discoloration of the vagina, softening of the cervix

Laboratory evaluation:
A blood sample (for CBC to determine whether anemia is present in the setting
of vaginal bleeding) and a urine specimen (for urinalysis and urine pregnancy
test) should be obtained.
Urine pregnancy tests measure human chorionic gonadotropin (hCG) and
standard tests become positive approximately 4 weeks following the first day of
the last menstrual period. Many emergency rooms have access to laboratory
facilities that can give rapid serum quantitative hCG (QhCG) results and this can
be important in the setting of a threatened miscarriage or possible ectopic
pregnancy where the measurement of QhCG levels over time can assist in
making the diagnosis.
The CBC and UA are normal. The urine pregnancy test is positive.
With a known pregnancy, the next steps are to determine if it is intrauterine or ectopic,
viable or non-viable. The physical examination is important in ruling out adnexal masses
or pain that indicate a possible ectopic pregnancy. Uterine size is a clue to the presence
of an intrauterine pregnancy, though fibroids (leiomyomata) or adenomyosis can also
cause uterine enlargement. Typically, pregnancy size is given in terms of the estimated
gestational age or the size of fruit, with a six to eight-week size uterus the size of a large
orange, 12-14 weeks that of a grapefruit, and 14-16 weeks the size of a cantaloupe. A
12-week size uterus can be felt at the symphysis pubis and a 20-week size pregnancy
reaches the level of the umbilicus.
The accepted norm for pregnancy dating in the absence of a firm last menstrual period
date is the ultrasound. In addition, the ultrasound will help determine whether the
pregnancy is viable and whether it is intrauterine or not. A rule of thumb is the earlier
the ultrasound, the more reliable as far as dating. In practice, many ultrasounds are
performed at about 16-20 weeks gestation, which ensures both accurate dating and the
opportunity of evaluating the fetal for developmental abnormalities. The later in
pregnancy an ultrasound is performed, the less reliable is its dating ability, due to the
variance is fetal size with advancing gestation.
In very early pregnancy and particularly in obese women, a transvaginal ultrasound
may be necessary to identify the pregnancy and fetal cardiac activity. Otherwise,
transabdominal ultrasonography may be performed. Fetal cardiac activity visualized
on ultrasound confirms viability and should be present when the embryo is 5mm in
length or more (by TV U/S, corresponds to approximately 5 weeks GA).
2. What potential complications should you discuss with this patient?
This patients history of painless vaginal bleeding is common. If the pregnancy test is
positive and an intrauterine pregnancy is not identified then the patient needs to be
further evaluated to rule out an ectopic pregnancy. With an early intrauterine
pregnancy, it is not always clear why this bleeding occurs; it may be due to bleeding
from a friable cervix or to bleeding from implantation of the pregnancy onto the uterine
wall. Painless vaginal bleeding early in pregnancy commonly resolves spontaneously

and does not recur. In those women who experience cramping as well as bleeding, the
risk of spontaneous abortion (miscarriage) is increased. Patients with cramping and
bleeding may require closer monitoring with repeat ultrasounds or perhaps repeat
QhCG levels until the symptoms resolve and the risk of miscarriage decreases, usually
after the first trimester. If a spontaneous abortion is diagnosed, appropriate steps are
taken to manage this situation.
3. What would you recommend as further management for this patient?
In anticipation of a normally developing, desired pregnancy, steps are taken to initiate
prenatal care by referring the patient for a first office visit to begin antenatal care. At
the first prenatal care visit, laboratory studies will be obtained, including blood and Rh
typing, serology studies, an HIV test, hepatitis and rubella titers, and an antibody
screen. She can be prescribed prenatal vitamins and iron from the emergency room,
otherwise this will be done at the first office visit. She should be cautioned to return to
the emergency room if her bleeding resumed and especially if it was accompanied by
the passage of tissue or severe cramping. A copy of the ultrasound photo should be
given to her.

References:
Obstetrics and Gynecology by Beckmann 5th Edition, 2006; Chapter 5 Antepartum Care. Pages 63-78.
Essentials of Obstetrics and Gynecology by Hacker and Moore 4th Edition, 2004; Chapter 8 Antepartum
Care: Preconception and Prenatal care, Genetic Evaluation and Teratology, and Antenatal Fetal Assessment.
Pages 83-103.

TEACHING CASE 10
Intrapartum Care
Student Handout
Clinical Case:
AJ is a 23-year old G1P0 currently at 38 weeks gestation who comes to Labor and
Delivery complaining of a 5 hour history of mild contractions occurring every 7 15
minutes and lasting 30-45 seconds in duration. Her antepartum course has been
uncomplicated. She denies leaking of fluid per vagina, bloody show or vaginal bleeding.
She reports normal fetal movement.
VS: BP= 105/65, pulse = 95, respirations=18. The patient is placed on a fetal heart rate
monitor which reveals a reassuring pattern. The tocodynamometer confirms irregular
contractions every 7-15 minutes. The patients cervix is 1-2 cm dilated, 50% effaced
with the fetal vertex at 0 station.

Discussion Questions:
1. What is your differential diagnosis at this time?
2. What do we mean when we talk about cervical dilation and effacement?
3. What are the stages and phases of normal labor?
4. What are your next steps in management?
5. Discuss the steps of vaginal delivery beginning with the cardinal movements.
6. What are other methods of delivery if AJ had not been able to push effectively or
if fetal intolerance of labor had developed?
7. What are indications and contraindications for each method of delivery?

References:
Obstetrics and Gynecology by Beckmann 5th Edition, 2006; Chapter 6 Intrapartum Care. Pages 79-94.
Essentials of Obstetrics and Gynecology by Hacker and Moore 4th Edition, 2004; Chapter 9 Normal
Labor, Delivery, and Postpartum Care. Pages 104-135.

Intrapartum Care
Preceptor Handout

Rationale: Understanding the process of normal labor and delivery allows optimal care
and reassurance for the parturient and timely recognition of abnormal events.
The APGO Educational Objectives related to this topic are the following:
A. List the signs and symptoms of labor*
B. Describe the three stages of labor and recognize common abnormalities*
C. Describe the steps of a vaginal delivery*
D. Describe different methods of delivery with the indications and
contraindications of each
E. Describe the evaluation of common puerperal complications*

*Designated as Priority One in the APGO Medical Student Educational Objectives, 8 th


Edition

Intrapartum Care
Preceptor Handout

Clinical Case:
AJ is a 23-year old G1P0 currently at 38 weeks gestation who comes to Labor and
Delivery complaining of a 5 hour history of mild contractions occurring every 7 15
minutes and lasting 30-45 seconds in duration. Her antepartum course has been
uncomplicated. She denies leaking of fluid per vagina, bloody show or vaginal bleeding.
She reports normal fetal movement.
VS: BP= 105/65, pulse = 95, respirations=18. The patient is placed on a fetal heart rate
monitor which reveals a reassuring pattern. The tocodynamometer confirms irregular
contractions every 7-15 minutes. The patients cervix is 1-2 cm dilated, 50% effaced
with the fetal vertex at 0 station.
Discussion Questions:
1. What is your differential diagnosis at this time?
True labor vs. false labor
Have students discuss characteristics of true and false labor and how the
they are differentiated
1. Painless irregular contractions vs. regular painful contractions
2. Definition of labor = progressive cervical effacement, dilation or
both resulting from regular painful uterine contractions
2. What do we mean when we talk about cervical dilation and effacement?
Have the student explain cervical dilation and effacement

3. What are the stages and phases of normal labor?


Have students discuss stages of normal labor
Have students explain the concept of fetal station
Have students discuss the Friedman curve
Latent phase
Active phase
1st stage (6-18 hrs in nulliparas, 2-10 hrs in multiparas)
2nd stage (0.5-3 hrs in nulliparas, 5-30 min in multiparas)
3rd stage (0-30 min)

4. What are your next steps in management?


Admission
Intravenous access
Fetal heart rate monitoring
Laboratory evaluation
Have student discuss why each of these steps are taken

5. Discuss the steps of vaginal delivery beginning with the cardinal movements.
Engagement
Descent
Flexion
Internal rotation
Extension

If possible have a bony pelvis model or plastic pelvic model and model
fetus available to have student demonstrate the steps of vaginal delivery
(shows how).

6. What are other methods of delivery if AJ had not been able to push effectively or if
fetal intolerance of labor had developed
Forceps
Vacuum
Cesarean section
7. What are indications and contraindications for each method of delivery?
Discuss the appropriate assessment of a patient before an operative
vaginal delivery is performed
Discuss the indications for a c-section

References:
Obstetrics and Gynecology by Beckmann 5th Edition, 2006; Chapter 6 Intrapartum Care. Pages 79-94.
Essentials of Obstetrics and Gynecology by Hacker and Moore 4th Edition, 2004; Chapter 9 Normal
Labor, Delivery, and Postpartum Care. Pages 104-135.

CLINICAL CASE 11

Immediate Care of the Newborn


C.C. is a term male newborn infant at 5 minutes of age. The Apgar score assigned by the
charge nurse at 1 minute was 4. Currently, he has a heart rate of 110, a vigorous cry,
active motion of all four extremities, bluish hands and feet, and a positive grimace.
Because of the low 1-minute Apgar score, the charge nurse sent cord gas. The following
umbilical arterial gas measurements were noted: pH 7.14, pCO2 69 mm Hg, HCO3 21.
Discussion
Apgar scores are a useful aid to evaluate the clinical status of the newborn and the need
for resuscitation. The 1-minute Apgar score is used to identify an infants need for
immediate resuscitation. An infant with a score of 4-6 at 1 minute demonstrates
depressed respiration, flaccidity and poor color. In general, these infants respond to
stimulation and do not need resuscitation efforts, i.e. ventilation. The Apgar Scoring
System is reflected in the following table:
Sign

0 Points

Heart rate
Absent
Respiratory effort
Absent
Muscle tone
Flaccid
Reflex irritability
No response
Color
Blue, pale
*From Williams Obstetrics, 21st Edition

1 Point

2 Points

Below 100
Slow, irregular
Some flexion of extremities
Grimace
Body pink, extremities blue

Over 100
Good, crying
Active motion
Vigorous cry
Completely pink

A recent investigation on the continuing value of the Apgar score studied greater than
13,000 infants born before term, i.e. 26-36 weeks gestation. The neonatal mortality rate
was 315 per 1000 for infants who had a 5-minute Apgar score of 0-3, compared to 5 per
1000 in infants with a 5-minute Apgar score of 7-10. In term infants, the neonatal
mortality was 0.2 per 1000 if the 5-minute Apgar score was 7-10 vs. 244 per1000 if the
Apgar score was 0-3. These investigators concluded that the Apgar score, as a means of
evaluating the physical condition of the infant shortly after delivery, is still useful in
predicting survival during the neonatal period.
In summary, the Apgar score is useful to assess the condition of the infant at birth and
to predict survival through the immediate neonatal period. It should not be used alone
as evidence to assess neurologic damage and/or hypoxia. Another means to evaluate
the newborn is the umbilical artery blood gas pattern. An umbilical arterial blood pH <
7.2 is equal to acidemia. If the pCO2 is > 65 and the HCO3 is > 22, then respiratory
acidosis is present. Metabolic acidosis is present if the pCO2 is < 65 and the HCO3 is < 17
and a mixed pattern is if the pCO2 is > 65 and the HCO3 is 17.
Attempts have been made to relate the Apgar score to long-term outcomes caused by
antenatal events. In 1986, the American College of Obstetricians and Gynecologists, as

well as the American Academy of Pediatrics, issued a committee opinion on the use and
misuse of the Apgar score. The summary of this committee opinion states emphatically
that, Apgar scores alone should not be used as evidence that neurologic damage is
caused by hypoxia or by inappropriate intrapartum management. Importantly, there
are several factors that may affect the Apgar score and these include prematurity,
maternal medications and infection, such as chorioamnionitis.
In 1995, the American College of Obstetricians and Gynecologists recommended that
cord blood gas analysis be used in neonates with low Apgar scores to distinguish
metabolic acidemia from hypoxia and other causes that might result in a low Apgar
score. Birth asphyxia is an imprecise term and acidemia alone is not sufficient to
establish that there has been a hypoxic injury. Hypoxic injury may be established if all
of the following are present:
1.
2.
3.
4.

An umbilical artery pH < 7.0


Apgar score of 0-3 for more than 5 minutes
Neonatal neurologic sequelae such as coma, hypotonia and/or seizures
Multiorgan system dysfunction

It is recommended that blood gases be sent on all neonates with low Apgar scores to
distinguish acidemia from hypoxia or other causes that might result in low scores.

Teaching points:
1.

The Apgar score is an excellent means of assessment of the newborn for


resuscitation efforts and is predictable of survival in the neonatal period.

2.

Hypoxic encephalopathy requires the presence of


a.
An umbilical artery pH < 7.0
b.
Apgar score of 0-3 for more than 5 minutes
c.
Neonatal neurologic sequelae such as coma, hypotonia, and/or
seizures
d.
Multiorgan system dysfunction

CLINICAL CASE 12

Postpartum Care
At the conclusion of this exercise, the student will be able to:
1.
Describe normal physiologic changes of the postpartum period
2.
Identify and discuss management of common complications in the early
postpartum period
3.
Discuss contraceptive counseling of the postpartum, breastfeeding
patient
4.
Discuss postpartum depression: diagnosis and management

Case study #1
A 22-year-old multigravida delivered her third healthy child without complications and is ready
for discharge from the hospital. She is breastfeeding, as she has with all of her children.
Although she is not currently married, she is in a stable relationship. She is interested in
permanent sterilization and wants to discuss this with you at her postpartum check-up. She
does not want anything for contraception at discharge since she is breastfeeding and is
protected.

Teaching points
1.

Begin consideration of contraceptive options while the patient is in the


hospital
Unless women are breastfeeding every 3-4 hours around the clock, they may
be fertile as soon as 6 weeks post-partum

2.

Case study #2
A 36-year-old married woman has just delivered her first child, a healthy baby girl weighing 7
lb. Her labor and delivery were uncomplicated. She and her husband, who had been trying to
have a baby for years, are thrilled about this child. She is discharged on postpartum day 2. Two
weeks after discharge, your nurse reports that the patients husband has called and is
concerned about his wifes behavior. She is crying all of the time and last night she said, she did
not love our daughter and wished she had not been born. He thinks she is probably just
exhausted and wants something to help her sleep.

Teaching points
1.

Mood disorders that persist for >10 days may represent post partum
depression and need to be addressed by the patients health care
provider.

TEACHING CASE 13

Lactation
Student Handout
Clinical Case:
A 22 year-old G1P1 comes to the office for an urgent office visit 4 days postpartum. She
states that she has not been feeling well, has had a fever at home, and has a tender
swollen area on her left breast. She has no problems with her right breast. Her
previous medical history is significant for severe depression for which she was taking
Lithium prior to pregnancy, but currently she is taking Zoloft. On examination, she is in
no distress but appears tired. Her temperature is 100.7C and BP is 130/70. On breast
examination, she has an erythematous tender 4 cm size area on the left breast. Her left
nipple is also tender and has some cracks. The right breast is normal. She really would
like to breast feed, because she has heard it is good for her baby. However it has been
causing her a lot of stress, and she feels he has not been having adequate milk
production. In addition, she is concerned about how it will affect her sleep and how
often she would have to do it especially at night. She also would like to resume taking
Lithium as her symptoms of severe depression are returning.
Discussion Questions:
1. Does this patient have engorgement, a blocked milk gland (galactocele), or
mastitis? What symptoms and signs led to your diagnosis?
2. What treatment do you recommend for her problem?
3. Discuss the benefits of breastfeeding.
4. How do you address this patients concern about inadequate milk production?
5. What do you tell her about how frequently and how long to breastfeed?
6. What do you tell her about the safety of using lithium while breastfeeding?

References:
Obstetrics and Gynecology by Beckmann 5th Edition, 2006; Chapter 10 Postpartum Care. Pages 128131.
Essentials of Obstetrics and Gynecology by Hacker and Moore 4th Edition,2004; Chapter 9 Normal
Labor, Delivery, and Postpartum Care. Pages 125-127.

Lactation
Preceptor Handout

Knowledge of the physiology and function of the breast during lactation allows
appropriate counseling to the pregnant and postpartum woman.
The APGO Educational Objectives related to this topic are the following:
A. List the normal physiologic and anatomic changes of the breast during
pregnancy and the postpartum periods*
B. Recognize and treat common postpartum abnormalities of the breast*
C. List the reasons why breastfeeding should be encouraged*
D. Identify commonly used medications which are appropriate and inappropriate
to use while breastfeeding*
E. Counsel the lactating patient about commonly asked questions, such as
frequency, duration, inadequate production of milk, etc.

*Designated as Priority One in the APGO Medical Student Educational Objectives, 8th
Edition

Lactation
Preceptor Handout

Clinical Case:
A 22 year-old G1P1 comes to the office for an urgent office visit 4 days postpartum. She
states that she has not been feeling well, has had a fever at home, and has a tender
swollen area on her left breast. She has no problems with her right breast. Her
previous medical history is significant for severe depression for which she was taking
Lithium prior to pregnancy, but currently she is taking Zoloft. On examination, she is in
no distress but appears tired. Her temperature is 100.7C and BP is 130/70. On breast
examination, she has an erythematous tender 4 cm size area on the left breast. Her left
nipple is also tender and has some cracks. The right breast is normal. She really would
like to breast feed, because she has heard it is good for her baby. However it has been
causing her a lot of stress, and she feels he has not been having adequate milk
production. In addition, she is concerned about how it will affect her sleep and how
often she would have to do it especially at night. She also would like to resume taking
Lithium as her symptoms of severe depression are returning.

Discussion Questions:
1. Does this patient have engorgement, a blocked milk gland (galactocele), or
mastitis? What symptoms and signs led to your diagnosis?
This patient has mastitis: tender mass, fever, and systemic symptoms.
Engorgement is bilateral.
A plugged duct should not produce a fever or systemic symptoms.

2. What treatment do you recommend for her breast problem?


Antibiotics for mastitis with coverage for staph aureus, B-hemolytic streptococci,
and haemophilus influenzae. Dicloxacillin if no penicillin allergy.
She should continue nursing from the affected breast. If she resumes the
Lithium, she should stop breastfeeding.
Warm compresses to the breast and a water based cream such as lanolin or A
and D ointment for the cracked nipple would be appropriate.

3. List the benefits of breastfeeding.


Improved infant nutrition
Protection against infection
Protection against allergic reactions
Convenience
Decreased cost
4. How do you address this patients concern about inadequate milk production?
You reassure her that colostrum is produced in the first 5 days and is gradually
replaced by milk on the third to sixth day of infant life. You further tell her that
colostrum protects the baby against infection and that the infant will usually get
adequate fluids and nutrition during this transition
5. What do you tell her about how frequently and how long to breastfeed?
There is no definite frequency or length of time that she should breast feed. She
should base this on infant demand.
6. What do you tell her about the safety of using lithium while breastfeeding?
You tell her that lithium is probably unsafe to use while breastfeeding.
It is very important to address her depression and assess her for any suicidal
ideation. Risks and benefits of depression versus breastfeeding need to be
carefully discussed with this patient.

References:
Obstetrics and Gynecology by Beckmann 5th Edition, 2006; Chapter 10 Postpartum Care. Pages 128131.
Essentials of Obstetrics and Gynecology by Hacker and Moore 4th Edition,2004; Chapter 9 Normal
Labor, Delivery, and Postpartum Care. Pages 125-127.

UNIT TWO:
SECTION B

OBSTETRICS - Abnormal Obstetrics

14.
15.
16.
17.
18.
19.
20.
21.
22.
23.
24.
25.
26.
27.
28.
29.
30.

Ectopic Pregnancy
Spontaneous Abortion
Medical & Surgical Conditions in Pregnancy
Preeclampsia-Eclampsia Syndrome
Isoimmunization
Multifetal Gestation
Fetal Death
Abnormal Labor
Third-Trimester Bleeding
Preterm Labor
Premature Rupture of Membranes
Intrapartum Fetal Surveillance
Postpartum Hemorrhage
Postpartum Infection
Anxiety and Depression
Postterm Pregnancy
Fetal Growth Abnormalities

TEACHING CASE 14
Ectopic Pregnancy
Student Handout
Clinical Case:
A 36-year-old G1P0010 female presents to your preceptors office with onset today of
light vaginal bleeding, which she feels is not her menstrual period, and mild right lower
quadrant pain which she rates as 2/10. The pain is intermittent and crampy, and is not
associated with urination. There is no nausea or vomiting. The patients last bowel
movement was yesterday and was normal in consistency without blood or black color.
Her past medical history shows no allergies, no medications, and two hospitalizations.
The first was eight years ago for lower abdominal pain which was thought to be due to
Pelvic Inflammatory Disease and which resolved with antibiotics. The second was for a
left ectopic pregnancy which required surgical removal of her left tube. Review of
Systems and Family History are unremarkable. Social History shows that she is
mutually monogamous with a male partner without birth control.
Physical Exam shows an anxious appearing female with a temperature of 99.2 degrees
orally, a BP of 105/62, and a pulse of 95. Examination of her abdomen shows normal
bowel sounds, no masses, organomegaly, distention, or rebound, and 2/10 right lower
quadrant pain. Pelvic examination reveals 2/10 right adnexal tenderness without
adnexal masses. Uterus is normal size. Rectal exam is negative with heme negative
stool.
Discussion Questions:
1. What is the differential diagnosis for this patient? Which specific symptoms and
signs does this patient have that are suspicious for an ectopic pregnancy?
2. What risk factors predispose patients to ectopic pregnancy and which of these
risk factors does this patient have?
3. Which is the most important test to do next in order to narrow down your
diagnosis?
4. If this patients pregnancy test is positive, what next tests could be helpful in
making a more definitive diagnosis?
5. What treatment options are available?
References:
Obstetrics and Gynecology by Beckmann 5th Edition, 2006; Chapter 15 Ectopic Pregnancy. Pages 160170.
Essentials of Obstetrics and Gynecology by Hacker and Moore 4th Edition, 2004; Chapter 25 Ectopic
Pregnancy. Pages 325-333.

Ectopic Pregnancy
Preceptor Handout

Ectopic pregnancy is a leading cause of maternal morbidity and mortality in the United
States. Early diagnosis and management may not only save lives, but may also preserve
future fertility.
The APGO Educational Objectives related to this topic are the following:
A. Develop a differential diagnosis for bleeding and abdominal pain in the first
trimester*
B. Identify risk factors for ectopic pregnancy*
C. Be able to evaluate a patient suspected of having an ectopic pregnancy*
D. Diagnose an ectopic pregnancy
E. Describe treatment options for patients with ectopic pregnancy

*Designated as Priority One in the APGO Medical Student Educational Objectives, 8 th


Edition

Ectopic Pregnancy
Preceptor Handout

Clinical Case:
A 36-year-old G1P0010 female presents to your preceptors office with onset today of
light vaginal bleeding, which she feels is not her menstrual period, and mild right lower
quadrant pain which she rates as 2/10. The pain is intermittent and crampy, and is not
associated with urination. There is no nausea or vomiting. The patients last bowel
movement was yesterday and was normal in consistency without blood or black color.
Her past medical history shows no allergies, no medications, and two hospitalizations.
The first was eight years ago for lower abdominal pain which was thought to be due to
Pelvic Inflammatory Disease and which resolved with antibiotics. The second was for a
left ectopic pregnancy which required surgical removal of her left tube
Review of Systems and Family History are unremarkable. Social History shows that she
is mutually monogamous with a male partner without birth control.
Physical Exam shows an anxious appearing female with a temperature of 99.2 degrees
orally, a BP of 105/62, and a pulse of 95. Examination of her abdomen shows normal
bowel sounds, no masses, organomegaly, distention, or rebound, and 2/10 right lower
quadrant pain. Pelvic examination reveals 2/10 right adnexal tenderness without
adnexal masses. Uterus is normal size. Rectal exam is negative with heme negative
stool.

Discussion Questions:
1. What is the differential diagnosis for this patient? Which specific symptoms and
signs does this patient have that are suspicious for an ectopic pregnancy?
Differential Diagnosis:
-ectopic pregnancy
-incomplete, completed, or missed abortion
-threatened abortion
-ovarian cyst
-adnexal torsion
-pelvic inflammatory disease
-endometriosis
-appendicitis and GI etiologies
-urinary tract infections or stones

Symptoms or clinical presentation may include:


-abdominal pain(95-100%)*
-abnormal uterine bleeding(65-85%)*
-amenorrhea(75-95%).
Note that the LNMP was not given in the clinical case to see if the students ask
about this. You can tell them that the LNMP was 7 weeks ago.
Signs or physical findings include:
-abdominal tenderness*(80-90%)
-adnexal tenderness* (75-90%)
-normal uterine size*( 70%)
-adnexal mass(30-50%)
However, ectopic pregnancies often have an atypical presentation and can have
extensive overlap with other abdominopelvic disorders.
The key learning point for students is that any sexually active women in the
reproductive age group who presents with pain, irregular bleeding, and/or
amenorrhea should have ectopic pregnancy as part of the initial differential
diagnosis. Pain is common but is not always the presenting symptom in ectopic
pregnancies.
2. What risk factors predispose patients to ectopic pregnancy and which of these risk
factors does this patient have (indicated with an *)?
-previous ectopic pregnancy(approx 10 times increase)*
-history of pelvic inflammatory disease, gonorrhea, or chlamydia infections*
-history of previous gyn or abdominal surgery*
-sterilization failure
-endometriosis
-congenital uterine malformation
-older age(35-44 y/o are 3 times higher risk than younger women)*
Often, diagnosing and managing a patient with an ectopic pregnancy can be
difficult. Knowledge of the risk factors can heighten the clinicians suspicion and
may determine the direction of the investigation.
3. Which is the most important test to do next in order to narrow down your
diagnosis?
HCG - Most important test. Sometimes skipped which can lead to mortality or
morbidity

4. If this patients pregnancy test is positive, what next tests could be helpful in
making a more definitive diagnosis?
STAT CBC (to check for anemia that may indicate intra-abdominal bleeding).
You could say that this patient had a Hematocrit of 37%.
Ultrasound to look for:
-Intrauterine pregnancy
-Extra-uterine mass
-Extra-uterine fetal cardiac activity (which would rule out medical treatment).
However, students should be aware that false-positive intrauterine gestational
sacs may be seen in as many as 20% of ectopic pregnancies.
You could say that this patients ultrasound showed no evidence of intra or
extra-uterine pregnancy and was normal.
Quantitative HCG
Initial to see if level is >1500mIU/ml so that lack of intrauterine pregnancy on
transvaginal ultrasound can lead to a high suspicion of extra-uterine pregnancy.
You could say that this patients HCG level was 830mIU/ml
Serial HCG levels
Useful when ultrasound is not helpful and initial HCG is <1500mIU/ML.
In viable early intra-uterine pregnancy, HCG levels will usually rise by at least
66% in 48 hours. Less than 66% should cause suspicion of ectopic or non-viable
intrauterine pregnancy.
Patients who are stable where the diagnosis is unclear can be followed by serial
HCG levels and, when levels have reached high enough for ultrasound to be
effective, can have repeat ultrasounds.
You could say that this patients repeat HCG level was 1050mIU/ml 48 hours
after the first which was less than a 66% rise.
5. What treatment options are available?
This could lead to a discussion of whether to handle a patient like this with serial
repeat HCG levels, with methotrexate, or by surgery.
o Methrotrexate medical treatment requirements:
- Hemodynamically stable.
- No fetal heart beat seen outside of the uterus.
- Ectopic gestation that is not too big (usually <3.5cm).
- Cooperative patient who will be sure to return for follow-up and serial
HCGs and will report increased pain.

o Laparoscopy or Laparotomy with or without conservation of the fallopian


tube.
o Check for Rh status and give Rh negative women Rho-GAM to prevent
isoimmunization.

References:
Obstetrics and Gynecology by Beckmann 5th Edition, 2006; Chapter 15 Ectopic Pregnancy. Pages 160170.
Essentials of Obstetrics and Gynecology by Hacker and Moore 4th Edition, 2004; Chapter 25 Ectopic
Pregnancy. Pages 325-333.

TEACHING CASE 15
Spontaneous Abortion
Student Handout
Clinical Case:
A 32 year-old G4P0030 (three previous miscarriages) presents with a positive urine
pregnancy test at 9 weeks 4 days from start of last normal menstrual period. She
reports 5 days of moderate painless vaginal bleeding and chills. Physical examination
shows a temperature of 101.5 orally, pulse of 95, and BP of 95/60 with normal bowel
sounds, no rebound, and 5/10 suprapubic tenderness. Pelvic exam shows moderate
amount of blood in vagina with a closed 5/10 tender cervix and an 8/10 tender uterus.
No adnexal masses or tenderness.
Lab data shows a serum HCG level of 6,500 mIU/ml and ultrasound shows a gestational
sac in the uterus with no fetus seen. The ovaries and tubes appear normal.
Discussion Questions:
1. What are the different types of spontaneous abortion?
2. Which type or types is most likely in this case?
3. What is your reasoning for which type of abortion this is?
4. Why does this patient have a fever and tenderness and what needs to be done
about it?
5. If this patient did not have fever and tenderness, what complications could she
develop?
6. If this patient was 6 weeks pregnant and had an HCG level of 700 mIU/ml and a
negative ultrasound, what would be your differential diagnosis if she had a small
amount of bleeding and no fever or tenderness?
7. How would you make the diagnosis in question 6?
8. For a patient with any type of abortion, what blood test is essential to do?

References:
Obstetrics and Gynecology by Beckmann 5th Edition, 2006; Chapter 14 Spontaneous Abortion. Pages
153-159.
Essentials of Obstetrics and Gynecology by Hacker and Moore 4th Edition, 2004; Chapter 8 Antepartum
Care. Pages 85-89.

Spontaneous Abortion
Preceptor Handout

Bleeding is common in early pregnancy. A logical approach to its evaluation may not only
affect the outcome of the pregnancy, but also will help to reassure the patient.

The APGO Educational Objectives related to this topic are the following:
A. Develop a differential diagnosis for first trimester vaginal bleeding *
B. Differentiate the types of spontaneous abortion*
C. Define recurrent abortion
D. List the complications of spontaneous abortion
E. Identify the causes and complications of septic abortion *

*Designated as Priority One in the APGO Medical Student Educational Objectives, 8 th


Edition

Spontaneous Abortion
Preceptor Handout
Clinical Case:
A 32 year-old G4P0030 (three previous miscarriages) presents with a positive urine
pregnancy test at 9 weeks 4 days from start of last normal menstrual period. She
reports 5 days of moderate painless vaginal bleeding and chills. Physical examination
shows a temperature of 101.5 orally, pulse of 95, and BP of 95/60 with normal bowel
sounds, no rebound, and 5/10 suprapubic tenderness. Pelvic exam shows moderate
amount of blood in vagina with a closed 5/10 tender cervix and an 8/10 tender uterus.
No adnexal masses or tenderness.
Lab data shows a serum HCG level of 6,500 mIU/ml and ultrasound shows a gestational
sac in the uterus with no fetus seen. The ovaries and tubes appear normal.

Discussion Questions:
1. What are the different types of spontaneous abortion?
-threatened abortion
-incomplete abortion
-complete abortion
-inevitable abortion
-missed abortion (subtype is blighted ovum)
-septic abortion
-recurrent abortion
2. Which type or types is most likely in this case?
-missed abortion
-septic abortion
3. What is your reasoning for which type of abortion this is?
-missed because no tissue passed and nonviable pregnancy in the uterus
-septic because of fever and tenderness
4. Why does this patient have a fever and tenderness and what needs to be done about it?
-fever is most likely from infected non-viable products of conception
-patient needs immediate evacuation of the uterus and antibiotics in order to
prevent sepsis and possible septic shock

5. If this patient did not have fever and tenderness, what complications could she
develop?
-fever, sepsis, septic shock
-hemorrhage and DIC
6. If this patient was 6 weeks pregnant with no fever or tenderness, had an HCG level of
700 mIU/ml and a negative ultrasound with no evidence of a gestational sac, what
would be your differential diagnosis if she had a small amount of bleeding and no fever
or tenderness?
-threatened abortion
-incomplete abortion
-complete abortion
-missed abortion
7. How would you make the diagnosis in question 6?
Order serial HCGs since HCG level is too low for ultrasound to show an IUP
(which usually is seen on vaginal ultrasound at 1500-2000 mIU/ml HCG. If this
is a threatened abortion, the HCG level usually will increase at least 66% when
repeated in 48 hours. If it does not, a incomplete, complete, or missed abortion
is very likely, and a stable patient can either be followed down to a negative
quantitative HCG, have a D and C, or have a medical evacuation of the pregnancy.
8. For a patient with any type of abortion, what blood test is essential to do?
Blood typing for Rh factor is essential followed by RHoGAM injection if patient is
Rh negative. This is vital to prevent Rh sensitization in a subsequent pregnancy
which can endanger the fetus.

References:
Obstetrics and Gynecology by Beckmann 5th Edition, 2006; Chapter 14 Spontaneous Abortion. Pages
153-159.
Essentials of Obstetrics and Gynecology by Hacker and Moore 4th Edition, 2004; Chapter 8 Antepartum
Care. Pages 85-89.

TEACHING CASE 16
Medical and Surgical Conditions of Pregnancy
HIV in Pregnancy
Student Handout
Clinical Case:
A 30 year-old white female P0000 who is infected with human immunodeficiency virus
(HIV, diagnosed 4 years ago) presents to your office inquiring about future pregnancy.
She is recently married to a man who is not infected with HIV and they want to have a
child. She is concerned about whether her baby could be infected with HIV and whether
pregnancy could make her develop AIDS. She is asymptomatic and is taking no HIV
medications. Her pap smears have all been normal. Pertinent ROS: She has felt well, no
recent fevers, chills, cough, shortness of breath, abdominal pain, vaginal discharge,
night sweats, diarrhea, weight loss, or other symptoms. On examination she has normal
external genitalia, no vaginal discharge or cervical lesions, an anteverted uterus of
normal shape and size, nontender, with no adnexal masses palpable.
Laboratory tests: CD4+ lymphocyte count (per mm3) 489; HIV RNA (copies/ml) 6520.
Discussion Questions:
1. What are the major issues to discuss with a woman who is HIV+ and wants to have a
baby?
2. What are some of the risk factors for mother to child transmission (MTCT) of HIV?
3. What are the recommendations for treatment of HIV in pregnant women? How may
these differ from recommendations for non-pregnant adults?
4. Are there any antiretroviral drugs that should be avoided or administered in altered
doses in pregnant women with HIV?
References:
1.

2.

Public Health Service Task Force. Recommendations for use of antiretroviral drugs in pregnant
HIV-1-infected women for maternal health and interventions to reduce perinatal HIV-1
transmission in the United States and Safety and toxicity of individual antiretroviral agents in
pregnancy. February 24, 2005. Available at http://AIDSinfo.nih.gov. Accessed November 1, 2005.
Riley LE and Yawetz S. Case Records of the Massachusetts General Hospital: Case 32-2005: A 34year-old HIV-positive woman who desired to become pregnant. N Engl J Med 2005; 353:1725-32.

Medical and Surgical Conditions of Pregnancy


HIV in Pregnancy
Preceptor Handout

Medical and surgical conditions may alter the course of pregnancy and pregnancy may
have an impact on the management of these conditions.
At the end of this case, the student will be able to:
A. Discuss the potential impact of human immunodeficiency virus (HIV) infection
on the gravid patient and the fetus/newborn, as well as the impact of pregnancy
on this condition, if any.
B. Describe the initial management of HIV infection in the pregnant patient.

Medical and Surgical Conditions of Pregnancy


HIV in Pregnancy
Preceptor Handout
Clinical Case:
A 30 year-old white female P0000 who is infected with human immunodeficiency virus
(HIV) presents to your office inquiring about future pregnancy. She is recently married
to a man who is not infected with HIV and they want to have a child. She is concerned
about whether her baby could be infected with HIV and whether pregnancy could make
her develop AIDS.
History: The patient was diagnosed with HIV infection 4 years ago as part of a routine
annual visit at her gynecologists office. She believes that she acquired HIV through
heterosexual contact from a previous boyfriend. She has had no other known STIs. She
is asymptomatic and is taking no HIV medications. Her husband is HIV negative and
they use condoms regularly. Her pap smears have all been normal.
Pertinent ROS: She has felt well, no recent fevers, chills, cough, shortness of breath,
abdominal pain, vaginal discharge, night sweats, diarrhea, weight loss, or other
symptoms.
Physical Examination: Pelvic: Normal external genitalia, no vaginal discharge or cervical
lesions, anteverted uterus, normal shape and size, nontender, no adnexal masses
palpable.
Laboratory tests: CD4+ lymphocyte count (per mm3) 489; HIV RNA (copies/ml) 6520.
Discussion Questions:
1. What are the major issues to discuss with a woman who is HIV+ and wants to have a
baby?
Three major issues in this case:
a. Management of HIV in a pregnant woman (or, does pregnancy alter the course of
HIV infection?). Answer: Fortunately, pregnancy does not appear to accelerate the
course of HIV infection. For the most part, pregnant women with HIV should be
treated in the same manner as non-pregnant adults with HIV (exceptions discussed
below).
b. Management of pregnancy in an HIV infected woman (or, what are the risks to
the fetus and are there ways to prevent mother to child transmission [MTCT] of
HIV?). Answer: The major risk to the fetus is MTCT of HIV. In the absence of
antiretroviral therapy, the MTCT rate for HIV is ~25%. With effective therapy, this

rate can be reduced to 1% or less. Additional risk factors for MTCT of HIV are
discussed below.
c. Safe sex practices (or, prevention of HIV infection for the HIV-negative partner in
discordant couples). Answer: Of course, abstinence or the regular use of condoms
are the best methods of prevention. The risk of transmission to the uninfected
husband in this case during each act of unprotected intercourse would be low (but
not eliminated) once the womans viral load becomes undetectable on therapy.
However, in the event of desired pregnancy such as in this case, artificial
insemination should be recommended.
2. What are some of the risk factors for MTCT of HIV?
a. Maternal disease status: CD4+ T-cell count, HIV viral load, AIDS (more
advanced disease is associated with an increased rate of MTCT).
b. Co-existing STIs (need to screen and treat).
c. Drug abuse (need to screen and offer treatment).
d. Labor-related factors: the majority of MTCT of HIV during pregnancy occurs
at the time of labor and delivery; major risk factors during this time include:
length of time membranes are ruptured (>4 hours associated with increased
risk), chorioamnionitis (infection associated with increased risk), fetal scalp
electrode (invasive monitoring associated with theoretically increased risk),
vaginal versus cesarean delivery (in the absence of antiretroviral therapy, in
the presence of ZDV monotherapy, or in the presence of HIV viral load >1000
on therapy women should be offered elective C/S at 38 weeks GA prior to
labor or ROM).
3. What are the recommendations for treatment of HIV in pregnant women? How may
these differ from recommendations for non-pregnant adults?
Current recommendations (November 2005) for starting antiretroviral therapy in nonpregnant adults include a CD4+ T-cell count <200-350 and an HIV viral load >100,000.
Therefore, for this patient in the non-pregnant state, we would not recommend starting
antiretroviral therapy for her own health based on her laboratory values. However, we
should recommend antiretroviral therapy to prevent transmission of HIV to her fetus.
The primary goal of antiretroviral therapy in pregnancy is to maximally suppress viral
load by the time of delivery. Unfortunately, even in the presence of undetectable viral
load on antiretroviral therapy, there is the possibility of transmission (i.e., there is no
threshold viral load below which transmission does not occur). However, for any given
viral load, the rate of transmission is lower among women on antiretroviral therapy
compared to women not receiving therapy. Pregnant women with HIV should be
treated with combination antiretroviral therapy (ZDV monotherapy is suboptimal). The
choice of drugs should be made on the basis of clinical experience of individual agents
in pregnancy, their known and suspected toxic effects on mother and fetus,
pharmacokinetic data in pregnancy, long-term efficacy and patients concerns (see
below). Women may start on therapy prior to pregnancy (optimal) or at 14 weeks
gestation (some women prefer to defer starting therapy until after the first trimester).

Oral therapy should be continued in labor and delivery and, unless contraindicated for
some reason, intravenous ZDV should be administered to the mother during labor or
prior to elective C/S. The neonate should receive oral antiretroviral therapy for up to
six weeks (follow updated recommendations, coordinate care with pediatricians).
4. Are there any antiretroviral drugs that should be avoided or administered in altered
doses in pregnant women with HIV?
The field of HIV is a rapidly advancing one and the drug regimens available can be
complicated with multiple possible side effects. These factors emphasize the need for
consultation with a provider who is experienced in the care of HIV infected adults.
There is a web site that is updated regularly where providers can access current
guidelines as well as obtain assistance by phone and by e-mail
(http://AIDSinfo.nih.gov). Discussing all of the potential nuances of therapeutic options
and concerns in pregnancy is beyond the scope of this case. However, selected examples
follow: There are certain drugs that should be avoided or monitored differently in
pregnancy: 1) teratogenic effects (efavirenz or Sustiva); 2) potential maternal or
neonatal mitochondrial toxicity (certain combinations of nucleoside reverse
transcriptase inhibitors, NRTIs); 3) potential maternal hepatic toxicity (nevirapine or
Viramune); increased risk for glucose intolerance (protease inhibitors); and altered
pharmacokinetics (indinavir or Crixivan).

Teaching Points:
Basic Science: pharmacology and pharmacokinetics of antiretroviral drugs (see
discussion question D).
Clinical: 1) antiretroviral therapy for HIV infected pregnant women (see discussion
question C) and 2) importance of preconception counseling for HIV-infected women
who wish to become pregnant.
Epidemiology: 1) transmission (timing and rates) of HIV from mother to fetus; 2) risk
factors for MTCT of HIV; and 3) safe sex practices.
References:
Public Health Service Task Force. Recommendations for use of antiretroviral drugs in pregnant HIV-1infected women for maternal health and interventions to reduce perinatal HIV-1 transmission in the
United States and Safety and toxicity of individual antiretroviral agents in pregnancy. February 24,
2005. Available at http://AIDSinfo.nih.gov . Accessed November 1, 2005.
Riley LE and Yawetz S. Case Records of the Massachusetts General Hospital: Case 32-2005: A 34year-old HIV-positive woman who desired to become pregnant. N Engl J Med 2005; 353:1725-32.

TEACHING CASE 17
Preeclampsia
Student Handout
Clinical Case:
An 18 year old G1P0 currently at 38 0/7 weeks presents for her routine prenatal visit.
She has had an uncomplicated pregnancy up to this point, with the exception of a late
onset of prenatal care and obesity (BMI of 35 kg/m2). She reports that during the past
week, she has noted some swelling of her hands and feet. She also has been feeling a bit
more fatigued and has had a headache on and off. She reports good fetal movement. She
has had some contractions on and off, but nothing persistent. Her blood pressure is
147/92 and her urine dip has 1+ protein/no ketones/no glucose. The fundal height
measures 36 cm, the fetus is cephalic with a heart rate of 144 bpm. On physical exam
you note that the patient has 3+ pre-tibial edema, and trace edema of her hands and
face. She has 2+ deep tendon reflexes and 2 beats of clonus. You review her blood
pressures up to this point and note that at the time of her first prenatal visit at 18
weeks, her blood pressure was 130/76 and she had no protein in her urine. However,
since that visit, her blood pressures seem to have been climbing higher with each visit.
Her last visit was one week ago, and she had a blood pressure of 138/88 with trace
protein in the urine and she has gained 5 pounds.
Discussion Questions:
1. What is the appropriate method for assessing blood pressure in the ambulatory
setting and what is considered a hypertensive blood pressure during pregnancy?
2. What types of hypertensive syndromes can occur during pregnancy?
3. What are the symptoms and physical findings of preeclampsia-eclampsia
syndrome?
4. What are the laboratory findings that support a diagnosis of preeclampsiaeclampsia syndrome?
5. What types of maternal and fetal complications are associated with preclampsiaeclampsia syndrome?

References:
Obstetrics and Gynecology by Beckmann 5th Edition, 2006; Chapter 17 Hypertension in Pregnancy.
Pages 188-196. Essentials of Obstetrics and Gynecology by Hacker and Moore 4th Edition, 2004;
Chapter 15 Hypertensive Disorders of Pregnancy. Pages 197-207.

Preeclampsia
Preceptor Handout

Preeclampsia-eclampsia syndrome accounts for significant morbidity and mortality in


both the mother and newborn.
The APGO Educational Objectives related to this topic are the following:
A. Define hypertension in pregnancy*
B. Classify hypertension in pregnancy
C. Describe the pathophysiology of preeclampsia-eclampsia syndrome
D. Enumerate the symptoms of preclampsia-eclampsia*
E. Summarize the physical findings of preeclampsia-eclampsia*
F. Diagnose preeclampsia-eclampsia syndrome
G. Manage a patient with preeclampsia-eclampsia syndrome
H. Counsel a patient concerning the indications of MgSO4
I. Counsel a patient regarding the maternal-fetal complications associated with
preeclampsia-eclampsia syndrome

*Designated as Priority One in the APGO Medical Student Educational Objectives, 8 th


Edition

Preeclampsia
Preceptor Handout
Clinical Case:
An 18 year old G1P0 currently at 38 0/7 weeks presents for her routine prenatal visit.
She has had an uncomplicated pregnancy up to this point, with the exception of a late
onset of prenatal care and obesity (BMI of 35 kg/m2). She reports that during the past
week, she has noted some swelling of her hands and feet. She also has been feeling a bit
more fatigued and has had a headache on and off. She reports good fetal movement. She
has had some contractions on and off, but nothing persistent. Her blood pressure is
147/92 and her urine dip has 1+ protein/no ketones/no glucose. The fundal height
measures 36 cm, the fetus is cephalic with a heart rate of 144 bpm. On physical exam
you note that the patient has 3+ pre-tibial edema, and trace edema of her hands and
face. She has 2+ deep tendon reflexes and 2 beats of clonus. You review her blood
pressures up to this point and note that at the time of her first prenatal visit at 18
weeks, her blood pressure was 130/76 and she had no protein in her urine. However,
since that visit, her blood pressures seem to have been climbing higher with each visit.
Her last visit was one week ago, and she had a blood pressure of 138/88 with trace
protein in the urine and she has gained 5 pounds.
Discussion Questions:
1. What is the appropriate method for assessing blood pressure in the ambulatory
setting and what is considered a hypertensive blood pressure during pregnancy?
Blood pressure assessment in the ambulatory setting should be conducted using
the following principles in order to reduce inaccurate readings:

Use an appropriate cuff size: 1.5 times upper arm circumference or a cuff
with a bladder that encircles 80% or more of the arm
Position the patient in the upright position
Allow the patient a 10 minute or longer rest period
No tobacco or caffeine 30 minutes preceding the measurement
Manual sphygmomanometer preferred

In pregnancy, hypertension is defined as either a systolic blood pressure 140


or diastolic blood pressure 90 or both.
2. What types of hypertensive syndromes can occur during pregnancy?

Chronic hypertension: Requires that the patient have documented


hypertension preceeding 20 weeks gestation, or where hypertension is
first noted during pregnancy and persists for longer than 12 weeks
postpartum.

Preeclampsia-eclampsia: Development of new onset hypertension and


proteinuria after 20 weeks of pregnancy. Is stratified into mild and severe
forms.

Preeclampsia superimposed on chronic hypertension: Superimposed


preeclampsia should be reserved for those women with chronic
hypertension who develop new-onset proteinuria ( 300 mg in a 24-hour
collection) after the 20th week of pregnancy. In pregnant women with
preexisting hypertension and proteinuria, the diagnosis of superimposed
preeclampsia should be considered if the patient experiences sudden
significant increases in blood pressure or proteinuria or any of the other
signs and symptoms consistent with severe preeclampsia.

Gestational Hypertension: Hypertension without proteinuria which first


appears after 20 weeks gestation or within 48 to 72 hours after delivery
and resolves by 12 weeks postpartum.

3. What are the symptoms and physical findings of preeclampsia-eclampsia


syndrome?
Symptoms:
Headache
Scotoma,
Pain in the right upper quadrant
Physical Findings:
Elevated blood pressure
Abnormal weight gain
Clonus
Generalized edema (e.g. hands and face in addition to lower extremity)
4. What are the laboratory findings that support a diagnosis of preeclampsiaeclampsia syndrome?

Proteinuria (> 300 mg on a 24 hour urine collection)


Elevated hematocrit
Hemolysis
Thrombocytopenia (< 100,000 cells/mm3)
Elevated liver enzymes
Elevated serum uric acid concentration

References:
Obstetrics and Gynecology by Beckmann 5th Edition, 2006; Chapter 17 Hypertension in Pregnancy.
Pages 188-196.
Essentials of Obstetrics and Gynecology by Hacker and Moore 4th Edition, 2004; Chapter 15
Hypertensive Disorders of Pregnancy. Pages 197-207.

CLINICAL CASE 18

Isoimmunization
At the conclusion of this exercise, the student will be able to demonstrate knowledge of
the following:
1.
2.
3.
4.

Red Cell Antigens


Use of immunoglobulin prophylaxis during pregnancy
Clinical situations under which D isoimmunization are likely to occur
Management of the at-risk pregnancy in a woman with a history of Rh
isoimmunization

A 32-year-old woman, P1101, and her new husband present for prenatal care at 20
weeks gestation. Her past obstetric history is significant for a first child delivered at
term following an abruption. Her second child died of complications of prematurity
following in utero transfusions for Rh isoimmunization. Her initial prenatal labs this
pregnancy indicate her blood type as A negative and an antibody screen positive for
anti-D with a titer of 1:64. You discuss any additional evaluation needed, her risks in
this pregnancy, and the plan of management with her and her husband.
Teaching points:
1.
2.
3.
4.
5.
6.
7.
8.
9.

What is Rh isoimmunization and what are the red cell antigens involved?
What are the risk factors for Rh isoimmunization?
What is the mechanism for RhoGAM prophylaxis against Rh disease?
What is the dose of RhoGAM?
What is the recommended schedule for RhoGAM administration?
Could this patients Rh isoimmunization have been prevented?
Is there any further blood work that should be obtained before you
counsel this patient on her risks in this pregnancy?
Discuss the management of the Rh-sensitized mother in an at-risk
pregnancy.
What are some ultrasound findings that may suggest Rh disease?

CLINICAL CASE 19

Multifetal Gestation
At the conclusion of this exercise, the student will be able to:
1.
2.
3.

Discuss the etiology of multiple gestation


Discuss the zygosity of twin gestation, understanding diagnosis and
complications unique to each
Discuss antepartum, intrapartum and postpartum management and
complications of twin gestation.

A 32-year-old African-American nullipara at 33 weeks gestation, who has been


followed in your practice with a known twin gestation, presents for an office visit. She
has no known medical problems. Her pregnancy was the result of ovulation induction
and, other than having twins and anemia, has been uncomplicated to date. She has had
normal weight gain during the pregnancy and her BP and urine dipstick are normal
today. She complains of decreased fetal movement for both babies and her fundal
height is smaller than expected for this gestation. She also complains of abdominal
cramping, but denies leaking fluid. Fetal heart tones are documented for both babies.
Her last fetal ultrasound was five weeks ago and the technician thought she saw a
dividing membrane between the two babies. The estimated fetal weight at that time
was appropriate for gestational age and the babies were similar in size, with the upper
twin slightly smaller.
Ultrasound evaluation reveals normal cardiac activity for both twins; however, there is
a 28% difference in weights with Twin A (presenting, vertex) larger than Twin B
(upper, breech). There is normal amniotic fluid volume and a thin dividing membrane is
seen. An NST is performed and both twins have a reactive fetal heart rate pattern with
no decelerations. However, she is noted to be contracting every 3 minutes on the
tocodynamometer. Vaginal examination showed the cervix to be 3 cm dilated and 75%
effaced with the vertex of Twin A at 1 station. The membranes appeared to be intact.
She is admitted to labor and delivery and treated for preterm labor. Her contractions do
not slow down and she progresses to complete dilation. She is taken to the operating
room and Twin A delivers by spontaneous vaginal delivery without complications. Twin
B is delivered by total breech extraction and also does well. After delivery, she does well
and is discharged home after postpartum day #2.
Teaching points:
1.
2.
3.
4.

Etiology of multiple gestation


Preterm labor and its associated morbidities
Utility of tocolytic medications
Delivery of twins

CLINICAL CASE 20

Fetal Death
At the conclusion of this exercise, the student will be able to:
Discuss common causes of fetal death in each trimester of pregnancy.
Discuss symptoms and diagnosis of fetal death.
Discuss management of the patient with a fetal death or a history of fetal death in a
prior pregnancy.

Case study #1
A 30-year-old Hispanic woman Para 0020 has been followed in your practice with
insulin dependent diabetes and presents for a routine prenatal visit at 36 weeks
gestation. She had a normal ultrasound at 20 weeks gestation with a normal fetal
anatomic survey. She reports no problems and good fetal movement. Her blood sugar
control has been reasonable. Her BP and urine dipstick were normal. Unfortunately, no
fetal heart tones were heard by Doppler and an ultrasound evaluation confirmed no
fetal cardiac activity. She is very upset and you spend time with her counseling her
regarding causes of fetal death and options for management.

Case study #2
A 26-year-old woman Para 0100 presents to your office for her first prenatal care visit.
She has no known medical problems. In her last pregnancy, she had an intrauterine
fetal demise at 28 weeks gestation. She reports that she began itching two weeks before
the baby died and her doctor did some blood tests and said everything was okay. At the
time of her delivery, she said the obstetrician taking care of her told her that the babys
umbilical cord was wrapped around its neck several times. She wants to discuss plans
for managing this pregnancy.

Teaching points:
-Risk factors and causes of fetal death
-Counseling and management of fetal death in the third trimester
-Counseling and evaluation of a woman with a history of a fetal death in a prior
pregnancy

CLINICAL CASE 21

Abnormal Labor
At the conclusion of this exercise, the student will be able to:
1.
Recognize abnormal labor patterns
2.
Describe fetal and maternal complications resulting from abnormal labor
3.
Discuss indications for oxytocin administration
4.
Describe indications for cesarean delivery
Clinical Case:
Charlene is a 31-year-old G1P0 attorney who has been treated in the infertility clinic for
several years to achieve this very desired pregnancy. Her prenatal care has been
uncomplicated. Her only problem has been heartburn for which she takes antacid
tablets with some relief. She comes to labor and delivery today at 40 6/7 wk. gestation.
She says the baby has been very active and she could not sleep well last night because of
discomfort. She states that she has been having contractions about every 5-7 minutes
for about 10 hours and she is exhausted. She wants the pregnancy to be over. She
denies leaking of fluid or vaginal bleeding, but did notice some bloody, sticky discharge
at about 3:00 am.
A vaginal exam reveals her cervix to be about 2cm dilated, 100% effaced, with a vertex
presentation at 3 station. Fetal heart tones are in the 140s with an external monitor.
Another external monitor confirms uterine contractions about every 5 minutes. Her
vital signs are stable and her physical examination is otherwise unremarkable.
Dipstick urinalysis reveals no protein, glucose or ketones. A blood clot to type and hold
is sent to the laboratory per hospital policy. A CBC is normal.
Charlene is admitted to labor and delivery where an IV line is placed. Two hours later,
there has been no change in the cervical exam and she asks for pain medicine. Narcotic
medication is ordered and she soon falls asleep. The fetal heart tones remain stable and
the contraction frequency has decreased.
Charlene is awakened about two hours after the narcotic dose by painful contractions
that appear on the monitor about every 3 minutes. A cervical exam is done revealing a
change to 5cm dilation, 100% effaced, and 2 station with some caput noted. The
membranes are artificially ruptured, revealing copious amounts of clear fluid; internal
fetal scalp and pressure monitors are placed. Charlene requests an epidural, and the
anesthetist is called.
The epidural functions well and the internal uterine pressure catheter show uterine
contractions every 7-10 minutes measuring 30-40 cmHg. The fetal heart tracing
remains reassuring. After another two hours, the cervix is unchanged and the station

has remained at 2. Oxytocin is started by intravenous pump. The uterine contractions


become more frequent, to every 2-4 minutes, and reach 50-60 cmHg. Her temperature
has climbed slightly, to 99.8 F. Two hours later the cervix is still 5 cm dilated and she
has a temperature of 101 F; the fetal heart rate is reassuring, but the baseline has
increased to the 160s. An operative delivery is planned.
Diagnosis:
-Failure to progress due to arrest of dilation
-Probable chorioamnionitis
Teaching points:
1. The Friedman curve is a tool to measure the rate of cervical change over time and
can be used to monitor the progress of labor in its latent and active phases. The
Friedman curve can be used to identify the patient who falls off the curve and
thus demonstrates an abnormality of labor. Such abnormalities can be due to a
lack of contractions, inadequate force of contractions, dystocia or cephalopelvic
disproportion. Some labor abnormalities, such as inadequate force or frequency
of contractions, can be remedied by the administration of oxytocin or other
contraction-inducing medications. Others, such as cephalopelvic disproportion,
require surgical intervention.
2. The Friedman curve relies on measurements of cervical change and fetal
position, station, the position of the fetal head relative to the ischial spines of the
maternal pelvis, dilation of the cervix measured in centimeters, and cervical
effacement measured in centimeters or as a percentage of the cervix that has
thinned. These measurements assume that the presentation of the fetus is vertex;
the presentation can also be breech or the lie may be transverse. These latter
types of fetal presentation or lie affect the ability of the uterus to contract and of
the fetus to descend through the maternal pelvis and thus may predispose to
abnormal labor.
3. Oxytocin is the most widely used pharmaceutical agent to initiate or augment
uterine contraction in order to produce the repetitive, strong contractions that
allow for cervical change and descent of the fetus through the maternal pelvis. Its
efficacy can best be measured by internal pressure monitors that allow accurate
observation, not only of the frequency of contractions, but also of their adequacy.
These measures are reflected in Montevideo units. When Montevideo units have
been adequate, when sufficient time has been allowed to observe cervical change
and when no change has been forthcoming, alternative delivery methods must be
considered. If the cervix has been completely dilated and effaced, and maternal
pushing efforts have not resulted in delivery, and if the fetal vertex is at a +2 or
greater station, vacuum or forceps delivery may be considered. In any other
situation, cesarean delivery is the alternative.

4. Pain medication in labor is used both for patient comfort and as a treatment to
aid the progress of labor. The use of intravenous or intramuscular narcotic
medication can allow maternal relaxation, including of the cervix and uterus, that
may result in a progression of labor. Epidural anesthesia can accomplish similar
results, though there are data to suggest that labor can be prolonged after its use
and that perhaps there is an association with increased cesarean delivery for
failed labor progress.
5. The fetus is in a sterile environment until membranes are ruptured, when
bacteria that are normally present in the vagina may ascend into the uterus and
cause an infection. In addition to temperature elevation, other signs of an intrauterine infection include a tender uterus, elevated white blood count and, in
extreme cases, pus emerging from the uterus. The fetal response to infection and
its consequent temperature elevation is tachycardia and a decrease of long and
short-term variability (see Objective 27). Administration of antibiotics to the
mother is often instituted, especially if delivery is not imminent. Given clinical
signs and symptoms, broad-spectrum antibiotics may be continued for 24 to 48
hours after the patient becomes afebrile (see Objective 29).

TEACHING CASE 22
Third Trimester Bleeding
Student Handout
Clinical Case:
A 25-year-old G2P1 female at 32 weeks gestation is brought to labor and delivery by her
husband. About an hour before, she was watching television when she noted a sudden
gush of bright red blood vaginally. The bleeding was heavy and soaked through her
clothes, and she has continued to bleed since then. She denies any cramps or abdominal
pain. She says that her last sexual intercourse was a week ago. A review of her prenatal
chart finds nothing remarkable other than a borderline high blood pressure from her
first prenatal visit that has not required medication. There is no mention of bleeding
prior to this episode. She had an ultrasound to confirm pregnancy at 14 weeks, but none
since.
Physical examination reveals an extremely pale woman whose blood pressure is 98/60,
pulse 130, respirations 30, temperature 99 F. Her abdomen is soft without guarding or
rebound to palpation, and the uterus is nontender and firm, but not rigid. Fundal height
is 33cm. Fetal heart tones are in the 140s with good variability. The external monitor
reveals uterine irritability, but no discrete contractions are seen. There is a steady
stream of bright red blood coming from her vagina.
Discussion Questions:
1. What is your differential diagnosis for potential causes of bleeding for this
patient?
2. What steps would you take to evaluate this patient and determine the etiology of
the bleeding?
3. What signs and symptoms would help you differentiate the potential causes of
the bleeding?
4. What steps would you take to manage the low blood pressure and tachycardia
that the patient is displaying?

References:
Obstetrics and Gynecology by Beckmann 5th Edition, 2006; Chapter 20 Third-Trimester Bleeding. Pages
209-215.
Essentials of Obstetrics and Gynecology by Hacker and Moore 4th Edition, 2004; Chapter 11 Obstetric
Hemorrhage and Puerperal Sepsis. Pages 146-150.

Third Trimester Bleeding


Preceptor Handout

Bleeding in the third trimester requires immediate patient evaluation. Thoughtful, prompt
evaluation and management is necessary to reduce the threat to the lives of the mother
and fetus.
The APGO Educational Objectives related to this topic are the following:
A. List the causes of third trimester bleeding*
B. Describe the initial evaluation of a patient with third trimester bleeding
C. Differentiate the signs and symptoms of third trimester bleeding*
D. State the maternal and fetal complications of third trimester bleeding
E. Describe the initial management for shock secondary to acute blood loss*
F. List the indications and potential complications of blood products

*Designated as Priority One in the APGO Medical Student Educational Objectives, 8 th


Edition

Third Trimester Bleeding


Preceptor Handout

Clinical Case:
A 25-year-old G2P1 female at 32 weeks gestation is brought to labor and delivery by her
husband. About an hour before, she was watching television when she noted a sudden
gush of bright red blood vaginally. The bleeding was heavy and soaked through her
clothes, and she has continued to bleed since then. She denies any cramps or abdominal
pain. She says that her last sexual intercourse was a week ago. A review of her prenatal
chart finds nothing remarkable other than a borderline high blood pressure from her
first prenatal visit that has not required medication. There is no mention of bleeding
prior to this episode. She had an ultrasound to confirm pregnancy at 14 weeks, but none
since.
Physical examination reveals an extremely pale woman whose blood pressure is 98/60,
pulse 130, respirations 30, temperature 99 F. Her abdomen is soft without guarding or
rebound to palpation, and the uterus is nontender and firm, but not rigid. Fundal height
is 33cm. Fetal heart tones are in the 140s with good variability. The external monitor
reveals uterine irritability, but no discrete contractions are seen. There is a steady
stream of bright red blood coming from her vagina.
Discussion Questions:
1. What is your differential diagnosis for potential causes of bleeding for this patient?
The differential diagnosis of third trimester bleeding should include:

Placental abruption
Placenta Previa
Vasa Previa
Genital lacerations/trauma (e.g. labial, vaginal or cervical)
Foreign body
Cervical cancer
Bloody show

2. What steps would you take to evaluate this patient and determine the etiology of
the bleeding?
Confirm placental location
Avoid digital cervical exam
Sonographic evaluation of placental location

Assess maternal hemodynamic status:


Serial vital signs
Hemotologic studies to assess for acute anemia and DIC
Kleihauer-Betke test for maternal-fetal hemorrhage
Assess fetal status:
Continuous external heart rate monitor or sonographic biophysical
assessment
3. What signs and symptoms would help you differentiate the potential causes of the
bleeding?
Placental abruption: Separation of the placenta from the uterine wall. It occurs
in about 20% of all third trimester bleeders and has a 25% recurrence risk in a
subsequent pregnancy. Risk factors for placental abruption include chronic
hypertension, cocaine use, abdominal trauma, sudden uterine compression (as
with rupture of membranes), and high parity. Physical findings include frequent
uterine contractions or hypertonicity, vaginal bleeding (sometimes
catastrophic), and fetal distress. Disseminated intravascular coagulation occurs
in 10% of cases, in 30% if the bleeding is severe. If the fetal heart tracing is
reassuring, expectant management and vaginal delivery may be considered. If
there are signs of maternal or fetal deterioration, an immediate cesarean
delivery is required. Perinatal mortality approaches 50% in severe cases.
Placenta previa: Occurs when placental tissue covers the cervical os. A central
or total placenta previa covers the os completely; as its name implies, a partial
placenta previa partially covers the os. In a marginal previa, the placental edge is
at the margin of the internal os while, with a low-lying placenta, the placenta
approaches the os, but is not at its edge. At 24 weeks, about 1 pregnancy in 20
will demonstrate ultrasound evidence of a placenta previa, while, at 40 weeks,
the incidence decreases to 1 in 200. Risk factors include prior cesarean delivery,
history of myomectomy, previous abortion, increased parity, multiple gestation,
advanced maternal age and smoking. Bleeding is usually painless and may occur
after intercourse. Management includes observation in labor and delivery, IV
access, continuous fetal monitoring and steroids for fetal lung maturation if
needed. Cesarean delivery is the method of choice with hysterectomy backup if
intraoperative bleeding cannot be controlled. Perinatal mortality can read 40%
Vasa previa: A rare condition where the fetal vessels of a velamentous cord
insertion cover the cervical os. The incidence is less than 1% of all pregnancies,
though it is increased in multiple gestations: up to 11% in twins and up to 95%
in triplets. The diagnosis is suggested by painless vaginal bleeding in the absence
of evidence of placenta previa or abruption. Treatment is delivery by cesarean
section.

Other causes: Causes of 3rd trimester bleeding such as cervicitis, cervical


erosions, trauma, cervical cancer, foreign body or even bloody show can usually
be differentiated on physical exam once the preceeding etiologies are ruled out.
4. What steps would you take to manage the low blood pressure and tachycardia that
the patient is displaying?
The most concerning feature of this presentation is the persistent bleeding and
vital signs suggestive of evolving shock related to acute blood. Tenets of
management should include:

Establish adequate IV access


i. 2 large bore IVs or central venous line
Monitor blood and coagulation profiles
i. Serial CBC and platelet counts
ii. Serial prothrombin time, partial thromboplastin time, and
fibrinogen
Volume resuscitation
i. Crystalloid
ii. Packed red blood cells
iii. Platelets, fresh frozen plasma and cryoprecipitate as indicated
Monitor vitals and response to therapy:
i. Serial blood pressure, heart rate, and respirations
ii. Continuous oxygen saturation monitor
iii. Continuous urine output assessement via indwelling Foley
catheter

Management of the patient with significant 3rd trimester hemorrhage, when the
fetus is mature, is hemodynamic stabilization and delivery. Vaginal delivery is
generally precluded in the setting of persistent hemodynamic instability.

References:
Obstetrics and Gynecology by Beckmann 5th Edition, 2006; Chapter 20 Third-Trimester Bleeding. Pages
209-215.
Essentials of Obstetrics and Gynecology by Hacker and Moore 4th Edition, 2004; Chapter 11 Obstetric
Hemorrhage and Puerperal Sepsis. Pages 146-150.

TEACHING CASE 23
Preterm Labor
Student Handout
Clinical Case:
An 18-year-old African-American, G2P0101 who is 12 weeks pregnant, presents to your
prenatal clinic for a new patient visit. Before you walk into the room to see the patient,
you look through her records and note that she delivered her last pregnancy just 12
months ago. Beginning at 24 weeks, the patient presented numerous times to Labor and
Delivery, reporting contractions, and was sent home each time with a diagnosis of
Braxton-Hicks contractions. She eventually presented at 28 weeks gestation and was
diagnosed with preterm labor. She eventually delivered at 29 weeks. The neonates
course was complicated by intra-ventricular hemorrhage and respiratory distress
syndrome. The child now appears to have cerebral palsy and chronic lung disease due
to bronchopulmonary dysplasia. The patients intake vitals are: Temp 36.5 C, HR 64,
RR 20, BP 100/60, wt 49 kg.
Your nurse tells you that the patient appears very anxious about this pregnancy and
that she has a lot of questions about why she had a premature baby in the first place.
She is concerned that she might have this type of complication with this current
pregnancy, but wishes that she could distinguish Braxton-Hicks contractions from true
labor better so that she can avoid coming to Labor & Delivery repeatedly like she did
last time.
Discussion Questions:
1. What are risk factors for preterm labor, and which if any does this patient have?
2. What characteristics distinguish Braxton-Hicks contractions from true labor
contractions?
3. What should you counsel the patient regarding the signs and symptoms of
preterm labor?

References:
Obstetrics and Gynecology by Beckmann 5th Edition, 2006; Chapter 22 Preterm Labor. Pages 221-227.
Essentials of Obstetrics and Gynecology by Hacker and Moore 4th Edition, 2004; Chapter 13 Obstetric
Complications. Pages 167-175.

Preterm Labor
Preceptor Handout
Prematurity is the most common cause of neonatal mortality and morbidity. The
reduction of preterm births remains an important goal in obstetric care. Understanding
the causes and recognizing the symptoms of preterm labor provides the basis for
management decisions.
The APGO Educational Objectives related to this topic are the following:
A. Cite the risk factors for preterm labor*
B. Distinguish preterm labor from Braxton-Hicks contractions
C. Identify the causes of preterm labor
D. Counsel the patient regarding the signs and symptoms of preterm labor*
E. Describe the initial management of preterm labor
F. List indications and contraindications of medications used to treat preterm
labor
G. Describe the adverse events associated with the management of preterm
labor
H. Counsel the patient who has experienced prior preterm birth
I. Describe cervical incompetence

*Designated as Priority One in the APGO Medical Student Educational Objectives, 8 th


Edition

Preterm Labor
Preceptor Handout
Clinical Case:
An 18-year-old African-American, G2P0101 who is 12 weeks pregnant, presents to your
prenatal clinic for a new patient visit. Before you walk into the room to see the patient,
you look through her records and note that she delivered her last pregnancy just 12
months ago. Beginning at 24 weeks, the patient presented numerous times to Labor and
Delivery, reporting contractions, and was sent home each time with a diagnosis of
Braxton-Hicks contractions. She eventually presented at 28 weeks gestation and was
diagnosed with preterm labor. She eventually delivered at 29 weeks. The neonates
course was complicated by intra-ventricular hemorrhage and respiratory distress
syndrome. The child now appears to have cerebral palsy and chronic lung disease due
to bronchopulmonary dysplasia. The patients intake vitals are: Temp 36.5 C, HR 64,
RR 20, BP 100/60, wt 49 kg.
Your nurse tells you that the patient appears very anxious about this pregnancy and
that she has a lot of questions about why she had a premature baby in the first place.
She is concerned that she might have this type of complication with this current
pregnancy, but wishes that she could distinguish Braxton-Hicks contractions from true
labor better so that she can avoid coming to Labor & Delivery repeatedly like she did
last time.
Discussion Questions:
1. What are risk factors for preterm labor, and which if any does this patient have?
Preterm birth related to spontaneous preterm labor should be distinguished
from preterm birth that is iatrogenic due to maternal or fetal complications such
as Preeclampsia. Risk factors for spontaneous preterm labor include:
Factors associated with Preterm
Labor
African-American race
Prior history of preterm birth
Preterm uterine contractions
Premature rupture of membranes (PROM)
Incompetent cervix
Shortenened cervix on transvaginal ultrasound
Infections
Urinary
Vaginal (BV)
Intra-amniotic

Excessive uterine enlargement


Polyhydramnios
Multiple gestation
Uterine distortion
Leiomyomas
Septate uterus, uterine didephis, and
other anomalies
Placental abnormalities
Abruption placentae
Placenta previa
Maternal smoking (associated with PROM)
Substance abuse
Adapted from Beckman, 5th Edition, Table 22.1, page 223.

This patient history is notable for a number of risk factors for preterm labor,
namely:

Prior history of preterm birth


African-American race
Low maternal weight

2. What characteristics distinguish Braxton-Hicks contractions from true labor


contractions?
Braxton-Hicks contractions: The uterus undergoes irregular and sporadic
contractions that are usually painless or of mild intensity beginning in the first
trimester. The frequency of these contractions increases in the last 4 to 8 weeks
of pregnancy. The contractions are not associated with progressive cervical
dilation and effacement. The contractions often resolve with sedation.
True Labor Contractions: As compared to Braxton-Hicks contractions, true labor
contractions occur at regular intervals, progressively increase in frequency and
intensity, and are associated with cervical dilation. True labor contractions will
not resolve with sedation.
3. What should you counsel the patient regarding the signs and symptoms of preterm
labor?
Distinguishing true preterm labor from false labor or Braxton-Hicks contractions
is challenging and depends primarily on prompt patient evaluation. Efforts to
develop models or tests to predict preterm labor have thus far been
unsuccessful. The primary method for identifying preterm labor is by screening
for maternal symptoms and signs as summarized below:
Symptoms and Signs of Preterm Labor
Menstrual-like cramps
Low, dull backache
Abdominal pressure
Pelvic pressure
Abdominal cramping (with or without diarrhea)
Increase or change in vaginal discharge (mucous, watery, light bloody discharge)
Uterine contractions, often painless
Adapted from Beckmann, 5th Edition, Table 22.2, page 223

References:
Obstetrics and Gynecology by Beckmann 5th Edition, 2006; Chapter 22 Preterm Labor. Pages 221-227.
Essentials of Obstetrics and Gynecology by Hacker and Moore 4th Edition, 2004; Chapter 13 Obstetric
Complications. Pages 167-175.

TEACHING CASE 24
Premature Rupture of Membranes
Student Handout

Clinical Case:
A 26-year-old Hispanic woman, who is 31 weeks pregnant, presents to the labor unit
complaining of ruptured membranes. She notes that, for the last two days, she has had
increased vaginal discharge and some lower back pain. She reports a gush of fluid about
2 hours ago. The fluid ran down her leg and appeared clear and with no noticeable odor.
Her prior pregnancy was complicated by preterm labor and premature ruptured
membranes at 26 weeks gestation. The neonates course was complicated by
necrotizing enterocolitis, respiratory distress, and death at 28 days of life. Her past
obstetric history includes two prior preterm deliveries at 26 weeks gestation.
Discussion Questions:
1. What factors are associated with PROM?
2. What should be the next step in diagnosis?
3. What should be the next step in management?
4. What are the risks associated with preterm PROM?

References:
Obstetrics and Gynecology by Beckmann 5th Edition, 2006; Chapter 23 Premature Rupture of
Membranes. Pages 228-232.
Essentials of Obstetrics and Gynecology by Hacker and Moore 4th Edition, 2004; Chapter 13 Obstetric
Complications. Pages 172-174.

Premature Rupture of Membranes


Preceptor Handout

Rupture of membranes prior to labor is a problem for both term and preterm pregnancies.
Careful evaluation of this condition may improve fetal and maternal outcome.
The APGO Educational Objectives related to this topic are the following:
A. Summarize the history, physical findings and diagnostic methods to
confirm
rupture of the membranes*
B. Cite the factors predisposing to premature rupture of membranes
C. List the risks and benefits of expectant management versus immediate
delivery
D. Describe the methods to monitor maternal and fetal status during
expectant
management
E. Counsel the patient with premature rupture of membranes

*Designated as Priority One in the APGO Medical Student Educational Objectives, 8 th


Edition

Premature Rupture of Membranes


Preceptor Handout

Clinical Case:
A 26-year-old Hispanic, who is 31 weeks pregnant, presents to the labor unit
complaining of ruptured membranes. She notes that, for the last two days, she has had
increased vaginal discharge and some lower back pain. She reports a gush of fluid about
2 hours ago. The fluid ran down her leg and appeared clear and with no noticeable odor.
Her prior pregnancy was complicated by preterm labor and premature ruptured
membranes at 26 weeks gestation. The neonates course was complicated by
necrotizing enterocolitis, respiratory distress, and death at 28 days of life. Her past
obstetric history includes two prior preterm deliveries at 26 weeks gestation.
Discussion questions:
1. What factors are associated with PROM?
Need to discuss the risk factors for PROM.
o Vaginal and cervical infections
o Abnormal membrane physiology
o Incompetent cervix
o Nutritional deficiencies
Vitamin C
Zinc
Copper
o Smoking
2. What should be the next step in diagnosis?
Need to discuss the work up and differential diagnosis for PROM.
Sterile speculum examination to confirm the diagnosis
o Fluid per cervical os
o Fern cervical mucus broad fern vs. amniotic fluid narrow fern
o pH (Nitrazine) turns blue
False positive Nitrazine due to
Alkaline urine
Blood
Cervical mucus
Antiseptic solutions
Vaginitis - trichomonas
o Cervicovaginal fetal fibronectin > 50 ng/ml

3. What should be the next step in management?


Need to discuss different management plans for PROM as well as the
pros and cons.
Monitor the fetus (EFM)
Antibiotics
o Ampicillin for prevention of GBS in neonate
o Steroids (to enhance fetal lung maturation and decrease RDS; however,
magnitude of reduction of RDS is not as great as when membranes are
intact)
4. What are the risks associated with preterm PROM?
Outline the maternal and fetal risk associated with PROM preterm and
at term.
Maternal
Chorioamnionitis
Cesarean delivery for malpresentation and failed induction
Abruption

Fetal risks
Cord prolapse
RDS
NEC
Infection (sepsis)
Pulmonary hypoplasia especially if < 19 weeks when PROM occurs
(rare after 26 weeks gestation)
Skeletal deformities

Teaching points:
The etiology of Preterm PROM is unknown, however, infection may play a
role
Complications of Preterm PROM include preterm delivery and all the risks
associated with prematurity, sepsis, cord prolapse. (Pulmonary hypoplasia
and skeletal anomalies are rare unless members rupture extremely early (<
22 weeks) and there is a concomitant prolonged latency period).
Management is dictated by the gestational age, fetal presentation, and
presence of chorioamnionitis.

References:
Obstetrics and Gynecology by Beckmann 5th Edition, 2006; Chapter 23 Premature Rupture of
Membranes. Pages 228-232.
Essentials of Obstetrics and Gynecology by Hacker and Moore 4th Edition, 2004; Chapter 13 Obstetric
Complications. Pages 172-174.

CLINICAL CASE 25
Intrapartum Fetal Surveillance
At the conclusion of this exercise, the student will be able to:
1.
List 4 ways by which the fetus may be evaluated during labor
2.
Describe the appropriate times when each method may be used
3.
List 3 periodic fetal heart rate changes and their significance
Clinical Case:
A 27-year-old gravida 3 para 2 married female is admitted to the labor and delivery unit
in early labor. She has had an uncomplicated pregnancy similar to her other two
pregnancies, both of which delivered vaginally. Her last labor lasted 4 hours and the
infants birth weight was 3900 grams after an uncomplicated delivery.
At the time of admission to the labor and delivery unit, her physical examination reveals
a healthy appearing woman in moderate distress with contractions every 4-6 minutes,
described as 7 on a pain scale of 1-10, with 10 being most severe. Her weight is 165
pounds, blood pressure is 135/82, and fundal height is 37 cm. The estimated fetal
weight is around 4000 grams, the fetus is in the vertex presentation and her pelvic
examination reveals a normal-sized pelvis with cervix dilated to 5cm/80%
effacement/-1 station. Fetal heart rate is noted to be 120 beats per minute when the
external monitor is applied at 1600.
This patient appears to be having a normal labor at term. The fetal heart rate is normal
and the fetus is having accelerations of the fetal heart rate, also a reassuring finding.
Her contraction pattern appears normal, and we should expect a vaginal delivery in the
next few hours.
Two hours later, the nurse calls you to the labor suite to review the fetal heart tracing.
She expresses concern about the changed appearance of the fetal heart tracing and asks
for your opinion. What do you think is the most likely etiology for the appearance of the
fetal heart rate on the tracing below?
Discussion:
Intrapartum fetal surveillance is an important tool in the management of labor. In spite
of concerns that the intensive electronic fetal monitoring used today leads to an
increase in surgical intervention and operative deliveries, most women in labor in the
United States receive some type of electronic fetal monitoring during labor. Care must
be taken to carefully evaluate the electronic fetal heart rate patterns in order to allow
labor to progress normally, but always being ready to intervene when necessary.

CLINICAL CASE 26
Postpartum Hemorrhage
At the conclusion of this exercise the student will be able to:
A. Describe risk factors for postpartum hemorrhage
B. Describe the differential diagnosis of postpartum hemorrhage
C. Understand the management of the patient with postpartum hemorrhage
Clinical Case:
Renee is a 33-year-old G3P2 patient who is scheduled for an elective repeat cesarean
delivery. She had a cesarean section two years ago for fetal intolerance to labor and has
been counseled about vaginal birth after cesarean section. She was ambivalent about
another surgery but, ultimately, decided for this since she also wanted a tubal ligation
and this could be done at the same time. She presents to labor and delivery at 38 wk.
gestation with spontaneous rupture of membranes an hour earlier. She is experiencing
intermittent, moderately painful contractions. She has called her husband at work, but
he has not yet arrived. She doesnt want to proceed to a cesarean delivery without him
beside her.

Physical examination:
Renee is quite uncomfortable with contractions that an external monitor reveals to be
occurring every three to four minutes. The fetal heart tones are in the 130s with
reassuring short and long-term variability. Her blood pressure is 110/80, pulse
80/minute, respirations 20/minute. The remainder of the physical examination is
unremarkable. However, you notice clear fluid emerging from the vagina and a
speculum exam reveals fluid pouring from the cervical os. Her most recent cervical
examination, yesterday in the office, revealed 2cm dilation, 50% effacement and a fetal
presentation high in the pelvis. Because of the ruptured membranes, you are concerned
about the possibility of a cord prolapse and you re-examine the cervix, finding that it
has changed to 4 cm dilation, 80% effacement with a vertex presentation at 3 station.
When given this information, Renee states that she prefers to avoid a cesarean delivery
and wishes to proceed with a vaginal delivery, if possible. Renees husband arrives and
is relieved to discover that she has declined a repeat cesarean section. Labor
progresses normally and, after pushing for about an hour, she delivers a male infant
with Apgar 8/9 weighing 7 pounds 13 ounces. The placenta delivers about 15 minutes
after the baby and the perineum appears intact. Thirty minutes later you are called to
the recovery room because Renee has experienced brisk vaginal bleeding that did not
respond to uterine massage by her nurse.

Laboratory:
Per hospital protocol for previous cesarean section patients, a blood sample is sent for a
type and crossmatch, a CBC reveals WBC 8.8, hematocrit 37%, hemoglobin 9 and
platelets 280,000. A urine dipstick is negative for sugar, ketones or protein.

Diagnosis:
Postpartum hemorrhage

Assessment/plan
You order Pitocin to be placed in the IV solution and check her blood pressure, which is
100/65. You ask that Methergine .20 mg be given intramuscularly. This is done but the
uterus continues to bleed. You return her to the delivery room where bimanual
exploration of the uterus is done after her epidural is re-dosed. You discover a fragment
of retained placenta and remove it manually. There is no evidence of a dehiscence of the
prior uterine incision. Gentle curettage of the uterine cavity fails to produce further
tissue. Careful vaginal exam reveals no cervical or vaginal lacerations. The bleeding
slows markedly, but is still a trickle; the uterus is moderately firm. You ask that a dose
of PGF2a be given intramuscularly and this is done. Further uterine massage succeeds
in reducing the bleeding to scant amounts. You order a CBC to be done in 4 hours and
for her intravenous line to be maintained. She is taken, once again, to the recovery room
for observation. Her blood pressure is stable, she is afebrile and she is alert, oriented
and comfortable.
Teaching points:
1.

Postpartum hemorrhage is defined as excessive bleeding in the


puerperium, after delivery of the baby and placenta. For vaginal
deliveries, this means over 500cc of blood loss; while in cesarean
sections, the blood loss should be over 1000cc to qualify as a postpartum
hemorrhage.

2.

Uterine atony is the most common cause and is, itself, caused by
precipitous labor; an overly distended uterus caused by such things as
macrosomia, multiple gestations or hydramnios; general anesthesia with
consequent uterine relaxation; amnionitis; oxytocin use in labor,
especially with prolonged inductions.

3.

Lacerations of the vagina, cervix, vulva or perineum can lead to significant


blood loss and are particularly common in instrumented deliveries
(forceps, vacuum), macrosomia or precipitous labor where the tissues
have no time to stretch.

4.

Retained placenta can result from a friable placenta, common in post


dates gestations; succenturiate placental lobes; prior cesarean section or
prior uterine curettage.

5.

Less common causes include coagulation defects, amniotic fluid


embolism, uterine inversion or hematomas in the vagina or vulva.

6.

Management includes evaluation for all items in the differential


diagnosis: inspection for lacerations, treatment of an atonic uterus with
massage, oxytocin, Methergine, prostaglandin. If medical maneuvers fail,
surgical procedures can be employed. These include uterine artery
ligation, hypogastric artery ligation, uterine compression sutures,
selective arterial embolization or hysterectomy.

7.

Circulation must be maintained with 2 large bore IV sites, IV fluid


infusion, and blood typing and crossmatch. Serial hematocrit will guide
plans for transfusion. Evaluation of coagulopathy may be needed if
bleeding persists, with the administration of whole blood or packed red
blood cells plus fresh frozen plasma or cryoprecipitate and platelets, if
necessary.

CLINICAL CASE 27
Postpartum Infection
Patient 1
A 19-year-old G1 now P1 African-American, two day post op from a c-section is
evaluated for a fever of 39o C. She denies nausea or vomiting, but has noticed increased
lower abdominal pain since last evening. Her pregnancy was uneventful and she
presented to the hospital at 38 6/7 days with rupture of membranes. 12 hours later, she
is given Pitocin to induce labor. 9 hours later, she is 5 cm/completely effaced and
vertex at zero station. Despite adequate contractions (240 Montevideo units per hour),
she has had no progress for 3 hours. At this time, a primary low transverse cesarean
section is performed. She delivered a viable male, 3750 grams, with Apgar 9/9 at one
and five minutes respectively. At the time of birth, she is given Keflex 1 gm for
intrapartum prophylaxis.
Past Medical History:
Med: neg
Surgeries: none
No known drug allergies
Medications: Prenatal vitamins, Iron, Folate
Lab:
WBC: 16.9 w/ 70% PMNs
Hematocrit: 34 vol. %
UA: negative

Patient 2
You are part of the obstetrical team caring for a 32-year-old primipara who has
presented with a term pregnancy and spontaneous rupture of membranes. The fluid is
clear, the baby is active and has a heart rate of 125, with good reactivity and variability.
The patient has had an unremarkable prenatal course. Pitocin induction is begun 24
hours after SROM.
Ten hours after beginning Pitocin induction, the patient is noted to be doing well with
her epidural. Her cervix is 7 cm dilated, 100% effaced. The fetus is in the vertex
position, seemingly OP, at -2 station. Her exam is unchanged 2 hours later despite Q 3
min. contractions that measure 60 mm Hg by an internal pressure catheter. She is
advised to have a cesarean section.

She delivers a 9 1b. 2 oz. boy, Apgars 7 and 9, by low transverse cesarean section. The
procedure is uncomplicated. Blood loss is estimated at 500 cc. She receives 2 grams of
a cephalosporin for prophylaxis after the umbilical cord is clamped and 20 units of
Pitocin after the placenta is removed.
On the second full day postpartum/post op, she complains of generally not feeling well.
She feels tired and achy, and wonders if she has a fever. She has no appetite and has not
yet passed flatus. The nurse reports that the patients pulse is 88, BP 110/70, and
temperature 38.5 C. Lower abdominal and uterine tenderness are the only positive
findings on exam.

Lab:
19,000 WBC, Hct 34, VA few epithelial cells, 1-5 WBCs, rare bacteria
Assessment/Plan:
Postpartum endometritis
Parenteral antibiotics
Discussion:
Postpartum endometritis remains a common and potentially serious complication of
abdominal delivery. This infection is also termed endometritis, metritis,
endomyometritis and endomyoparametritis. Of these, endometritis is the most
commonly used term to describe postpartum uterine infection.
The route of delivery that is vaginal vs. cesarean section is the single most important
risk factors. The incidence of endometritis following vaginal delivery rarely exceeds 2
3%; however, after cesarean section frequency ranges from 10% in low risk patients
who have received prophylactic antibiotics to as high as 95% in a high risk population
without prophylactic antibiotics. In the later group, i.e. cesarean section, if the
membranes have been ruptured for a prolonged period of time (> than 6 hours) and the
patient has had prolonged labor, then the likelihood of endometritis is markedly
increased. There are few data to support a direct increase in endometritis following the
use of electronic fetal monitoring or on the number of vaginal examinations. It is true
and related that with slow progress of labor there are more vaginal exams performed.
It is well established that the pathogenesis of postpartum endometritis involves both
anaerobic and aerobic organisms. This infection is an ascending infection and is caused
by the organisms found in the normal vaginal flora. These included the aerobic
organisms of Group A and B Streptococcus, Enterococcus, as well as Staphylococcus,
Gram-negative aerobic organisms include E.coli, Klebsiella pneumoniae, and Proteus
mirabilis, as well as a whole host of anaerobic organisms. The precise pathogenesis is a

complex interaction among the host defense mechanisms, the size of bacterial inoculum
and virulence of the bacteria involved. Obviously, the number of bacteria and the size of
the inoculum are primarily influenced by the length since rupture of the membranes,
the duration of labor, as well as, potentially, the number of vaginal examinations.
The most common reported clinical signs and symptoms of postpartum endometritis
include fever, leukocytosis, lower abdominal pain, uterine tenderness and foul-smelling
vaginal discharge. Clearly, the most important sign and symptom is that of fever. This
diagnosis is based on clinical findings alone and there has been no laboratory and/or
culture techniques used to increase the likelihood of this diagnosis.
At the time diagnosis of endometritis, parenteral antibiotic therapy is begun. Single
agent therapy, such as broad spectrum second and third-general cephalosporins offer
the advantage of less toxicity and theoretically less pharmacy and nursing time for
administration. When compared to the gold standard of clindamycin-gentamicin, the
cure rates and efficacy rates are identical. Failure to respond to the antibiotic therapy
within 48-72 hours may be due to pelvic abscess, septic pelvic thrombophlebitis and/or
the emergence of a resistant organism. The treatment should be continued until the
patient is afebrile, as well as asymptomatic, for 24-36 hours. Patient may be discharged
from the hospital at this time with no antibiotic therapy, as follow up oral antibiotics
are generally unnecessary.

Teaching points:
1.

Postpartum infection is common especially in high-risk groups (preterm


delivery, prolonged ruptured membranes, prolonged labor, women
whose delivery via cesarean section).

2.

This is an ascending polymicrobial infection involving both gram-positive


anaerobes and gram-negative anaerobes.

3.

Prophylactic administration of antibodies will decrease the likelihood of


endometritis after a cesarean section by > 50%.

4.

Sequelae of endometritis include pelvic abscess, septic pelvic vein


thrombophlebitis or phlegmon.

5.

Treatment should include broad-spectrum coverage for the polymicrobial


organisms most commonly involved in endometritis.

TEACHING CASE 28
Anxiety and Depression
Student Handout
Clinical Case:
Ms. Davis is a 22-year-old G3P2Ab1 African American reports that besides being
overwhelmed by having a newborn baby, her 2 -year-old daughter recently
experienced a severe illness. Since that time, she intermittently has thoughts about
hurting herself and her children. Her mother is aware of her concerns and is currently
caring for the children.
During her visit, Ms. Davis describes feeling depressed, sleep deprived, guilty and
hopeless. She also states that she has had crying spells and a decreased appetite for the
last two months. She can go a day or two without being hungry or eating, and she
reports feeling like her children would be better off without me or if they werent
here. She has made no plans to act on these feelings, although she notes that these
feelings have increased in frequency over the past two months.
Ms. Davis states that sometimes when she hears the newborn cry, she thinks she hears a
voice in her head telling her to just shake him until he stops crying. When she has
these kinds of thoughts, she says, she calls her mother or husband or reads the Bible
until these thoughts and feelings subside. She worries, however, that one day she will
not be placated by these means alone. She admits that occasionally she acts on impulse.
Ms. Davis notes that her sister has depression and is treated with fluoxetine.
She is a stay-at-home mom who has been married five years. Her pregnancy was
uncomplicated and she had a normal vaginal delivery at term. She initially tried to
breast feed, but stopped after 3 days due to sore nipples.
Discussion Questions:
1. What are the most important symptoms demonstrated by Ms. Davis?
2. How do her symptoms compare with other women at this postpartum stage?
3. If you were to develop a treatment protocol for Ms. Davis, what would it
comprise?
4. Would you elect to use pharmacologic or hormonal therapy?
5. Is a referral or consult needed for Ms. Davis and what would be your
justification?

References:
Obstetrics and Gynecology by Beckmann 5th Edition, 2006; Chapter 10 Postpartum Care. Pages 129131.

Anxiety and Depression


Preceptor Handout

Pregnancy, as with any significant life event, may be accompanied by anxiety and
depression. Recognition of psychological disturbance is essential for early intervention.

The APGO Educational Objectives related to this topic are the following:
A. Identify risk factors and diagnose postpartum blues, postpartum
depression and postpartum diagnosis*
B. Describe treatment options for postpartum blues, postpartum depression
and postpartum psychosis
C. Identify appropriate treatment options for depression in pregnancy

*Designated as Priority One in the APGO Medical Student Educational Objectives, 8 th


Edition

Anxiety and Depression


Preceptor Handout

Clinical Case:
Ms. Davis is a 22-year-old G3P2Ab1 African American reports that besides being
overwhelmed by having a newborn baby, her 2 -year-old daughter recently
experienced a severe illness. Since that time, she intermittently has thoughts about
hurting herself and her children. Her mother is aware of her concerns and is currently
caring for the children.
During her visit, Ms. Davis describes feeling depressed, sleep deprived, guilty and
hopeless. She also states that she has had crying spells and a decreased appetite for the
last two months. She can go a day or two without being hungry or eating, and she
reports feeling like her children would be better off without me or if they werent
here. She has made no plans to act on these feelings, although she notes that these
feelings have increased in frequency over the past two months.
Ms. Davis states that sometimes when she hears the newborn cry, she thinks she hears a
voice in her head telling her to just shake him until he stops crying. When she has
these kinds of thoughts, she says, she calls her mother or husband or reads the Bible
until these thoughts and feelings subside. She worries, however, that one day she will
not be placated by these means alone. She admits that occasionally she acts on impulse.
Ms. Davis notes that her sister has depression and is treated with fluoxetine.
She is a stay-at-home mom who has been married five years. Her pregnancy was
uncomplicated and she had a normal vaginal delivery at term. She initially tried to
breast feed, but stopped after 3 days due to sore nipples.
Discussion Questions:
1. What are the most important symptoms demonstrated by Ms. Davis?
Early signs of depression include sleeplessness, loss of self-esteem, irritability,
and mood swings. More serious symptoms are anorexia, obsessive behavior,
panic and delusions. Ms. Davis shows definite signs of postpartum depression.
2. How do her symptoms compare with other women at this postpartum stage?
There is a wide range of response to pregnancy/delivery from mild depression
(maternity blues) to postpartum depression and to the extreme response of
postpartum psychosis.

Symptoms for maternity blues: mild insomnia, tearfulness, fatigue, poor


concentration, depressed affect, irritability. There is a 50% to 80% incidence
rate with resolution in 10 days.
Symptoms for Postpartum depression: Labile mood, irritabililty, difficulty falling
asleep, phobias, anxiety that increases in the evening. Incidence is 10% or
greater with resolution in 3 to 14 months.
Symptoms for Postpartum psychosis: similar to organic brain syndrome,
confusion, attention deficit, distractibility, clouded sensorium. Incidence is 0.1 to
0.2 percent with resolution variable.
3. If you were to develop a treatment protocol for Ms. Davis, what would it comprise?
Treatment must be tailored to the patients situation. Most cases of mild
depression are handled by the physician. Treatment may include pharmacologic
or hormonal, and cognitive-behavioral therapy.
4. Would you elect to use pharmacologic or hormonal therapy?
Use drugs such as Sertraline and Amitriptyline, since they are safe in lactating
women. One study suggests that several commonly prescribed and efficacious
agents can be safely given to the breast feeding mother suffering from
postpartum depression (PPD). Quantifiable amounts of the maternal medication
are not found in the infants blood. Based on this guideline, the authors
recommend using the following drugs in cases where the mother desires to
continue breast feeding: Amitriptyline, Nortriptyline, Desipramine,
Clomipramine or Sertraline. SSRIs preferred as tricyclic antidepressants have
more side effects and an overdose could be lethal.
Fluoxetine (Prozac) is not a good drug for this disorder. Fluoxetine, although a
very useful and popular antidepressant, has been found to permeate breast milk
to levels approximately 20-25% of maternal plasma. This has prompted the drug
manufacturer to advise against the use of this product in the breast feeding
woman. In a case report, severe colic, fussiness and crying were identified in a
parturient taking fluoxetine. It has also been associated with reduced growth
that may be of clinical importance in situations in which the infant weight gain is
already of concern. Shorter acting SSRIs, such as Paroxetine, preferred.
Hormonal therapy is a newer option aimed at the potential causes of PPD. Some
investigators believe that mood disorders postpartum are precipitated by
changes in estrogen levels. The hypothalamic-pituitary-adrenal axis is greatly
suppressed directly after childbirth, and some researchers in the field of
endocrinology believe that estrogen will normalize CRH secretion and reactivate
the H-P-A axis to mitigate depressive symptoms in the postpartum woman.

5. Is a referral or consult needed for Ms. Davis and what would be your justification?
Group treatment through a cognitive-behavioral program, plus appropriate
pharmacotherapy, will yield a better outcome than either treatment alone.
Pharmacotherapy is a quick, efficient answer to the problem of PPD in most
cases but it is not all of the answer. Group treatment through a cognitivebehavioral program has been shown to be effective treatment, as well. Other
forms of psychotherapy have been studied with fairly conclusive results and
these forms of treatment augment the antidepressant effects of these
medications.

References:
Obstetrics and Gynecology by Beckmann 5th Edition, 2006; Chapter 10 Postpartum Care. Pages 129131.

TEACHING CASE 29
Postterm Pregnancy
Student Handout
Clinical Case:
A 35-year-old, G1P0, woman presents to your office for a routine prenatal exam. She is
5 days past her due date that was determined by her last menstrual period and a
second trimester ultrasound. While reviewing her chart, you note that she has gained
32 pounds during this uncomplicated pregnancy. Todays exam reveals a weight gain of
1/2 pound since last weeks visit. Her BP is 110/65. She has no glycosuria or
proteinuria. The fundal height measures 38 cm and fetal heart tones are auscultated at
120 bpm in the left lower quadrant. The fetus has cephalic presentation and an
estimated weight of 8 lbs.
Just before you go into the room, your nurse pulls you to the side, and tells you, She has
a lot questions! Once you walk into the room, the patient expresses her
disappointment that she has not had the baby yet. She assumed that she will be having
the baby on her due date. She asks you about potential harm to her and the baby from
going past her due date, and she would like to know her options.
Discussion Questions:
1. Define term, postdates, and postterm pregnancy and the prevalence of
each.
2. What are the causes of prolonged pregnancy?
3. What are the common antenatal, intrapartum, and neonatal complications
associated with prolonged pregnancy?
4. What are the features of fetal postmaturity syndrome?
5. What management plan would you recommend for this patient?

References:
Obstetrics and Gynecology by Beckmann 5th Edition, 2006; Chapter 21 Postterm Pregnancy. Pages 216220.
Essentials of Obstetrics and Gynecology by Hacker and Moore 4th Edition, 2004; Chapter 13 Obstetric
Complications. Pages 179-182.

Postterm Pregnancy
Preceptor Handout

Perinatal mortality and morbidity may be increased significantly in a prolonged


pregnancy. Prevention of complications associated with postterm pregnancy is one of the
goals of antepartum and intrapartum management.

The APGO Educational Objectives related to this topic are the following:
A. Identify the normal period of gestation*
B. Discuss the complications of post-maturity

*Designated as Priority One in the APGO Medical Student Educational Objectives, 8 th


Edition

Postterm Pregnancy
Preceptor Handout

Clinical Case:
A 35-year-old, G1P0, woman presents to your office for a routine prenatal exam. She is
5 days past her due date that was determined by her last menstrual period and a
second trimester ultrasound. While reviewing her chart, you note that she has gained
32 pounds during this uncomplicated pregnancy. Todays exam reveals a weight gain of
1/2 pound since last weeks visit. Her BP is 110/65. She has no glycosuria or
proteinuria. The fundal height measures 38 cm and fetal heart tones are auscultated at
120 bpm in the left lower quadrant. The fetus has cephalic presentation and an
estimated weight of 8 lbs.
Just before you go into the room, your nurse pulls you to the side, and tells you, She has
a lot questions! Once you walk into the room, the patient expresses her
disappointment that she has not had the baby yet. She assumed that she will be having
the baby on her due date. She asks you about potential harm to her and the baby from
going past her due date, and she would like to know her options.
Discussion Questions:
1. Define term, postdates, and postterm pregnancy and the prevalence of each.
The duration of pregnancy is reported in gestational weeks, calculated from the
first day of the last normal menses. Normal pregnancy lasts 40 2 weeks.

Preterm: Gestational age less than 37 completed weeks


Term: Gestational age greater than or equal to 37 completed weeks and less
than 42 completed weeks
Postterm: Gestational age greater than or equal to 42 completed weeks
Postdates: A common synonym for postterm, but also often used to indicate a
pregnancy beyond 40 weeks.

In the United States, approximately 12% of pregnancies deliver preterm.


Postterm pregnancy is estimated to have an incidence of 6% - 12%.
Approximately 80% of pregnancies are delivered at a term gestation.
2. What are the causes of prolonged pregnancy?
The most common etiology of post-term pregnancy is inaccurate pregnancy
dating. Other less common causes all relate to a presumed defect in fetal labor
initiation.

Factors Associated with Post-term Pregnancy


Factor

Discussion

Inaccurate or unknown dates

Most common cause; high association with and


major risk factor of late or no prenatal care

Irregular ovulation; variation in


length of follicular phase

Results in overestimation of gestational age

Altered estrogen:progesterone
ratio
Anencephaly

Decreased production of 16hydroxydehydroepiandrosterone sulfate, a


precursor of estriol

Fetal adrenal
hypoplasia

Decreased fetal production precursors of estriol

Extrauterine pregnancy

Pregnancy not in uterus; no labor

Adpated from Beckman, 5th Edition, Table 21.1, page 217.

3. What are the common antenatal, intrapartum, and neonatal complications


associated with prolonged pregnancy?
Most of the complications related to prolonged pregnancy are associated with
the underlying fetal growth abnormalities (either macrosomia or decreased
growth rate) and/or placental dysfunction.
Antenatal Concerns:
Macrosomia estimated prevalence of 25% in prolonged pregnancy
Postmaturity syndrome (see discussion below)
Perinatal death rate increases steadily after 37 weeks, approaching 1 in
300 at 42 weeks.
Intrapartum Concerns:
Labor dystocia
Infant Birth trauma
Maternal perineal trauma
Cesarean delivery
Postpartum hemorrhage
Meconium passage
Neonatal Concerns:
Meconium aspiration syndrome
Hypoglycemia
Hyperbilirubinemia

1. What are the features of fetal postmaturity syndrome?


Although the true incidence of the fetal postmaturity (dysmaturity) syndrome is
unknown, it has been estimated to occur in 10% of pregnancies between 41 and
43 weeks. The syndrome results from placental insufficiency due to aging and
infarction. Typical features include:

Loss of subcutaneous fat resulting in a long, thin body


Long fingernails
Dry, peeling, wrinkled skin
Abundant hair

Postmature infants have an increased risk of perinatal mortality, as compared to


other post-term infants.
2. What management plan would you recommend for this patient?

Confirm accuracy of gestational age estimate


Once a patient approaches 41 weeks of gestation:
Option 1:
Induce labor
Option 2:
Expectant management in anticipation of spontaneous labor
Fetal surveillance

There is no defined cut-off for when labor must be induced


A variety of antenatal testing options are currently employed, none of
which is clearly superior
There does not appear to be a difference in cesarean delivery rates
following induction at 41 versus 42 weeks.

References:
Obstetrics and Gynecology by Beckmann 5th Edition, 2006; Chapter 21 Postterm Pregnancy. Pages 216220.
Essentials of Obstetrics and Gynecology by Hacker and Moore 4th Edition, 2004; Chapter 13 Obstetric
Complications. Pages 179-182.

CLINICAL CASE 30
Fetal Growth Abnormalities
Cherie is a 26-year-old G2PO female who presents to your office for her first prenatal
visit. She states that the pregnancy has been uncomplicated, except for one episode of
the flu. History reveals that her LMP was 35 weeks ago. Her periods are irregular. She
smokes 2 packs of cigarettes a day and has gained 8 lbs. during this pregnancy.
Physical Exam:
BP 110/70; fundal height is 30 cm. Fetal heart tones are present.
Prenatal Lab:
Toxoplasmosis - <1:16
Herpes - <0.25
Rubella - Immune
CMV - .29
CBC - Hgb -10.1 gm/dl
WBC - 8.6
VDRL = non-reactive
Blood Type - 0+
Antibody Screen - negative
Obstetrical Ultrasonography Report:
Fetal number: Single
Position: Cephalic
Placenta: Anterior, grade II
Amniotic fluid volume: Normal
Fetal dating:
BPD:

82.9 mm = 33.3 3.1 weeks

HC:

299.7 mm = 33.2 3.0 weeks

AC:

274.0 mm = 31.5 3.0 weeks

FL:

58.0 mm = 30.3 3.0 weeks

Humerus:

51.2 mm = 29.9 2.8 weeks

Menstrual age = 34.9 weeks


Composite sonar age = 31.6 2.4 weeks
Estimated fetal weight = 1700 308 grams, less than the 10th percentile at 34.9 weeks

Fetal Organ Imaging


Kidneys:
+
Bladder:
+
Stomach:
+
Spine: Appears grossly normal
Cardiac motion: + (normal four chamber view)
Fetal Breathing: +
Limb motion: +
Umbilical Artery Doppler Flow Study
S/D ratio = 2.66 (average of 3 measurements). This is within normal limits for this
gestational age.
Comments/ Teaching points
This patients gestational age was established by an early ultrasound examination at 10
weeks plus 6 days. That examination confirmed menstrual dating. Therefore, there is a
3.3-week lag in expected growth from menstrual dating and early ultrasound
examination.
The amniotic fluid volume is 13.8 cm.
Clear views of the cerebellum and cord insertion are not possible today due to fetal
positions.
1. What do you tell the patient about her diagnosis, causes and treatment options?
2. How do you follow this patient?
A follow-up visit indicates a reactive NST with oligohydramnios. (AFI=2)
3. Now what?

UNIT TWO:
SECTION C

OBSTETRICS - Procedures

31. Obstetric Procedures

TEACHING CASE 31
Obstetric Procedures
Student Handout
Clinical Case:
A 26 year old, G3P2002 presents to Labor and Delivery with a complaint of frequent,
painful uterine contractions and leaking of fluid. She has been getting prenatal care
through your clinic and review of her records shows her to be 36 6/7 weeks with a
spontaneous dichorionic/diamniotic twin pregnancy. She has had a completely
uncomplicated pregnancy to date, with the exception of obesity. She is 5 4 tall and
weighs 220 pounds, giving her a BMI of 37.8 kg/m2. Your evaluation reveals the patient
to be 8 cm dilated, fully effaced, and +1 station. Although, the membranes are ruptured,
you are unsure of the presenting part. The fetal heart rate tracings for both twins are
reassuring. During your evaluation, the patient repeatedly tells you that she really
wants to deliver these twins vaginally because she delivered both of her prior babies
vaginally, and doesnt want to be slowed down by the recovery from a cesarean.
Discussion Questions:
1. Is this patient a candidate for a vaginal delivery of her twins? What additional
information do you need to make that decision?
2. You confirm that the patient is indeed a good candidate for vaginal delivery. What are
the complications that this patient may encounter during her delivery?
3. What pre-delivery preparations can you make to minimize these risks for the patient?
4. The patient achieves the 2nd stage of labor and progresses well to deliver the first infant
without complication. You perform an assessment for the presentation of the 2nd twin
and find it to be breech. What are your options for delivering the 2nd twin?
5. You proceed with attempting vaginal delivery of the 2nd twin. While waiting for the 2nd
fetus to progress in labor, you notice the onset of heavy vaginal bleeding. The fetal heart
rate tracing begins to deteriorate and you perform a cesarean delivery. What measures
can you take intra-operatively to prevent complications from the cesarean?
6. You complete the cesarean successfully, but note that the patient had an estimated
blood loss of 1500 cc probably due to an abruption. What measures can you take postoperatively assess for and diagnose complications?
References:
Obstetrics and Gynecology by Beckmann 5th Edition, 2006; Chapter 18 Multifetal Gestation. Pages 197201 & Chapter 24 Obstetric Procedures. Pages 233-238.
Essentials of Obstetrics and Gynecology by Hacker and Moore 4th Edition, 2004; Chapter 14 Multifetal
Gestation and Malpresentation. Pages 183-196 & Chapter 18 Obstetric Procedures. Pages 247-255.

Obstetric Procedures
Preceptor Handout

Knowledge of obstetric procedures is basic to management and counseling of the


pregnancy patient.
The APGO Educational Objectives related to this topic are the following:
A. Describe the key components of pre-operative evaluation and planning,
including the complete medical histories, the informed consent process and
working with consultants*
B. Describe the common peri-operative prophylactic measures, including steps
taken to reduce infections and deep vein thrombosis*
C. Describe the components of routine post-operative care*
D. List common post-operative complications*
E. Describe each procedure and list the indications and complications of each of
the following:
1. Ultrasound
2. Chorionic villous sampling
3. Amniocentesis and cordocentesis
4. Antepartum fetal assessment
5. Intrapartum fetal surveillance
6. Induction and augmentation of labor
7. Episiotomy
8. Spontaneous vaginal delivery
9. Vacuum-assisted delivery
10. Forceps delivery
11. Breech delivery
12. Cesarean delivery
13. Vaginal birth after cesarean delivery

*Designated as Priority One in the APGO Medical Student Educational Objectives, 8 th


Edition

Obstetric Procedures
Preceptor Handout

Clinical Case:
A 26 year old, G3P2002 presents to Labor and Delivery with a complaint of frequent,
painful uterine contractions and leaking of fluid. She has been getting prenatal care
through your clinic and review of her records shows her to be 36 6/7 weeks with a
spontaneous dichorionic/diamniotic twin pregnancy. She has had a completely
uncomplicated pregnancy to date, with the exception of obesity. She is 5 4 tall and
weighs 220 pounds, giving her a BMI of 37.8 kg/m2. Your evaluation reveals the patient
to be 8 cm dilated, fully effaced, and +1 station. Although, the membranes are ruptured,
you are unsure of the presenting part. The fetal heart rate tracings for both twins are
reassuring. During your evaluation, the patient repeatedly tells you that she really
wants to deliver these twins vaginally because she delivered both of her prior babies
vaginally, and doesnt want to be slowed down by the recovery from a cesarean.
Discussion Questions:
1. Is this patient a candidate for a vaginal delivery of her twins? What additional
information do you need to appropriately counsel this patient?
The key factors which should be considered in determining delivery approach
are:
Gestational age / Fetal size
Fetal presentation
Operator experience
Assuming adequate operator experience, the fetal presentation can be
determined by:
Vaginal exam
Leopolds maneuvers
Ultrasound
2. You confirm that the patient is indeed a good candidate for vaginal delivery. What
are the complications that this patient may encounter during her delivery?

Intrapartum fetal distress


Umbilical cord prolapse
Cesarean delivery of one or both twins (possibly emergent)
Postpartum hemorrhage

3. What pre-delivery preparations can you make to minimize these risks for the
patient?
Prerequisites for the intrapartum management of multiple
gestations

A secondary or tertiary care center to assure availability of anesthesia and


neonatal services
A delivery room equipped for immediate cesarean delivery, if needed.
A well-functioning large-bore intravenous line for rapid administration of fluids
and blood.
Blood availability for transfusion.
The capability to continuously monitor the fetal hart rates simultaneously.
An anesthesiologist who is immediately available to administer general
anesthesia should intrauterine manipulation or cesarean section be necessary
for delivery of the second twin.
Two obstetricians scrubbed and gowned for the delivery, one of whom is skilled
in intrauterine manipulation and delivery of the second twin.
Imaging techniques (i.e. sonography) for determining the precise presentations
of the twins.
Adequate newborn staff to assist in immediate resuscitation and care of the
infants.
Adapted from Hacker & Moore, 4th Edition, Chapter 14, page 187

4. The patient achieves the 2nd stage of labor and progresses well to deliver the first
infant without complication. You perform an assessment for the presentation of the
2nd twin and find it to be transverse. What are your options for delivering the 2 nd
twin?
If adequate fetal surveillance can be maintained and is reassuring, there is no
urgency in accomplishing delivery of the 2nd twin. Options for management
include:
Option 1: Cesarean delivery
Option 2: External cephalic version and vaginal delivery
Option 3: Internal podalic version and vaginal breech extraction

5. The fetus spontaneously converts to a cephalic presentation and you proceed with
attempting vaginal delivery of the 2nd twin. While waiting for the 2nd fetus to
progress in labor, you notice the onset of heavy vaginal bleeding. The fetal heart
rate tracing begins to deteriorate and you perform a cesarean delivery. What
measures can you take intra-operatively to prevent potential complications of the
cesarean?

Intra-operative antibiotics for endometritis prophylaxis


IV oxytocin for uterine atony prophylaxis
Have additional uterotonics available for treatment of persistent atony

6. You complete the cesarean successfully, but note that the patient had an estimated
blood loss of 1500 cc probably due to an abruption. What measures can you take
post-operatively to assess for and diagnose complications?

Immediate post-operative Hematocrit, and serial assessments if clinically


indicated
Urine output assessment
Physical exam

References:
Obstetrics and Gynecology by Beckmann 5th Edition, 2006; Chapter 18 Multifetal Gestation. Pages 197201 & Chapter 24 Obstetric Procedures. Pages 233-238.
Essentials of Obstetrics and Gynecology by Hacker and Moore 4th Edition, 2004; Chapter 14 Multifetal
Gestation and Malpresentation. Pages 183-196 & Chapter 18 Obstetric Procedures. Pages 247-255.

UNIT THREE:
SECTION A

GYNECOLOGY - General Gynecology

32.
33.
34.
35.
36.
37.
38.

Contraception and Sterilization


Abortion
Vulvar and Vaginal Disease
Sexually Transmitted Infections & Urinary Tract Infections
Pelvic Relaxation & Urinary Incontinence
Endometriosis
Chronic Pelvic Pain

TEACHING CASE 32
Contraception and Sterilization
Student Handout
Clinical Case:
A 17 year old G0 presents to clinic desiring information about contraceptive methods.
She reports that she is sexually active with her boyfriend, using condoms occasionally,
when she needs them. She has never used any other methods. She has had 2 lifetime
partners. She became sexually active at age 15 and had sex with her first partner 3-4
times but didnt use contraception. She has been sexually active with her current
partner for the last year. She came today because she last had unprotected intercourse
3 days ago and is worried she might get pregnant. She has decided its time for a more
reliable method of contraception. She has never had a Pap smear. She has history of
well controlled seizure disorder and had appendicitis at age 11. She is taking Valproic
Acid. She smokes one-half pack of cigarettes per day, drinks alcohol socially, and uses
occasional marijuana. Her blood pressure is 100/60 and pulse is 68.
Discussion questions:
5. What history is required for recommending appropriate contraception?
6. What physical exam and studies are required for prescribing hormonal
contraceptives?
7. What contraceptives should the patient be counseled about and what are the
advantages and disadvantages of each method?
8. When/how to start the contraceptive method?

References:
Obstetrics and Gynecology by Beckmann 5th Edition, 2006; Chapter 25 Contraception. Pages 241-257.
Essentials of Obstetrics and Gynecology by Hacker and Moore 4th Edition, 2004; Chapter 27 Family
Planning: Contraception, Sterilization, and Abortion. Pages 341-351.
A Pocket Guide to Managing Contraception, 2005-2007, Hatcher, Zieman, Creinin, et al.

Contraception and Sterilization


Preceptor Handout

An understanding of the medical and personal issues involved in decisions regarding


contraceptive methods is necessary to adequately advise patients requesting
contraception. In the process of deciding whether to have a sterilization procedure, men
and women often seek the advice of their physicians. Providing accurate information will
allow patients to make an informed decision regarding this elective surgery.

The APGO Educational Objectives related to this topic are the following:
A. Describe the physiologic basis of contraception (OCPs, emergency
contraception, patches, rings, IUD, sterilization, etc.) *
B. Describe the effectiveness of each form of contraception *
C. Counsel the patient regarding the benefits and risks for each form of
contraception *
D. Cite the financial considerations of the various forms of contraception
E. Describe the methods of male and female surgical sterilization *
F. List the risks and benefits of procedures, including: *
1. Potential surgical complications
2. Failure rates
3. Reversibility (lack of)

*Designated as Priority One in the APGO Medical Student Educational Objectives, 8th
Edition

Contraception and Sterilization


Preceptor Handout
Clinical Case:
A 17 year old G0 presents to clinic desiring information about contraceptive methods.
She reports that she is sexually active with her boyfriend, using condoms occasionally,
when she needs them. She has never used any other methods. She has had 2 lifetime
partners. She became sexually active at age 15 and had sex with her first partner 3-4
times but didnt use contraception. She has been sexually active with her current
partner for the last year. She came today because she last had unprotected intercourse
3 days ago and is worried she might get pregnant. She has decided its time for a more
reliable method of contraception. She has never had a Pap smear. She has history of
well controlled seizure disorder and had appendicitis at age 11. She is taking Valproic
Acid. She smokes one-half pack of cigarettes per day, drinks alcohol socially, and uses
occasional marijuana. Her blood pressure is 100/60 and pulse is 68.
Discussion Questions:
1. What history is required for recommending appropriate contraception?
o Sexual history
Onset of sexual activity
Number of partners since onset
History of STDs
o Medical history contraindications to estrogen-containing hormonal
contraceptives
Migraines with aura
DVT
Hypertension
Smoking age > 35
o Menstrual history
LMP (pregnancy)
Irregular menses
2. What physical exam and studies are required for prescribing hormonal
contraceptives?
a. Pap and pelvic exam have typically been bundled services, i.e., these
exams are required to prescribe contraceptives. There is no rationale for
this bundling.
b. Paps should be initiated 3 years after onset of sexual activitythis
patient became sexually active 2 years ago
c. STD screening for a sexually active teenager should include chlamydia
which may be tested with urine. Other STDs should be screened for
based on individual risk assessment.

d. A blood pressure should be obtained in patients who desire estrogencontaining contraceptives to R/O hypertension, rare in this age group, but
easy to obtain, non-sensitive and low cost.
3. What contraceptives should the patient be counseled about?

Combination hormonal methods: Pills, patch, ring


i. Advantages
b.
Cycle control
c.
Non-contraceptive benefits
ii. Disadvantages
b.
Nuisance side effects bloating, H/A, breast tenderness,
nausea
c.
No STD protection
d.
Need to remember daily, weekly, month
e.
Seizure medications may decrease effectiveness

Condoms
i. Advantages
b.
STD protection
ii. Disadvantages
b.
Need to use every time
c.
Decreased sensation

Depo-provera
i. Advantages
b.
4 shots per year
c.
Highly effective
ii. Disadvantages
b.
Irregular bleeding
c.
Weight gain
d.
No STD protection

IUD

Plan B
i. Advantages
b.
Backs up regular birth control
c.
Useful for accidents condom breaking, discontinued
methods
ii. Disadvantages
b.
Not as effective as regular methods
c.
May be difficult to obtain

i. Advantages
b.
Longterm contraception with single act motivation
c.
Highly effective
d.
High continuation
ii. Disadvantages
b.
No STD protection

4. When/how to start the contraceptive method?

Consider contraception an emergency


Best if patient leaves with a method
Advance prescriptions of Plan B to all patients (except those with an IUD)
Best if method begins that day if negative pregnancy test
i. Combination methods Quick start: First pill on day of visit
regardless of cycle, preferably in clinic
ii. Depo-provera Same day shot
iii. IUD Same day insertion

References:
Obstetrics and Gynecology by Beckmann 5th Edition, 2006; Chapter 25 Contraception. Pages 241-257.
Essentials of Obstetrics and Gynecology by Hacker and Moore 4th Edition, 2004; Chapter 27 Family
Planning: Contraception, Sterilization, and Abortion. Pages 341-351.
A Pocket Guide to Managing Contraception, 2005-2007, Hatcher, Zieman, Creinin, et al.

TEACHING CASE 33

Abortion

Student Handout
Clinical Case:
The patient is a 14-year-old G1P0 who presents to a private clinic requesting
termination of pregnancy. Her last menstrual period began 9 weeks prior to arrival.
She has been experiencing intermittent nausea and vomiting. She is sexually active
with her 21-year-old partner and reports she is having consensual sex with him. She
has been using condoms for contraception. She has no history of sexually transmitted
infections. She has mild asthma for which she uses an inhaler as needed. She drinks
socially 3-5 drinks a day on the weekends. She does not use drugs or tobacco and
lives with her mother and 17-year old brother. Her physical exam is unremarkable,
uterine size is approximately 8 weeks. Ultrasound confirms intrauterine pregnancy at 8
weeks with fetal heart motion present.
Discussion questions:
1. What is options counseling and how would you counsel this patient about her
options?
2. Is this patient a victim of sexual abuse? Should this case be reported to
authorities?
3. Can the patient consent for termination herself or must she have parental
consent since she is a minor?
4. What types of abortion is this patient eligible for, given her gestational age of 8
weeks?
5. How is a surgical abortion performed?
6. How is a medication abortion performed?
7. What are the potential complications of abortion?
8. How should this patient be counseled about contraception?
9. How should this patient be counseled and managed regarding STD prevention?

References:
Obstetrics and Gynecology by Beckmann 5th Edition, 2006; Chapter 2 Ethics in Obstetrics and
Gynecology. Pages 28-29 & Chapter 14 Abortion. Pages 158-159.
Essentials of Obstetrics and Gynecology by Hacker and Moore 4th Edition, 2004; Chapter 27 Family
Planning: Contraception, Sterilization, and Abortion. Pages 350-351.

Abortion
Preceptor Handout

Induced abortion is a reproductive option considered by some patients. Regardless of


ones personal views, the practitioner should be aware of the techniques, management and
complications of induced abortions.

The APGO Educational Objectives related to this topic are the following:
A. Explain surgical and non-surgical methods of pregnancy termination*
B. Identify potential complications of induced abortion*
C. Provide non-directive counseling to patients surrounding pregnancy options*

*Designated as Priority One in the APGO Medical Student Educational Objectives, 8 th


Edition

Abortion

Preceptor Handout
Clinical Case:
The patient is a 14-year-old G1P0 who presents to a private clinic requesting
termination of pregnancy. Her last menstrual period began 9 weeks prior to arrival.
She has been experiencing intermittent nausea and vomiting. She is sexually active
with her 21-year-old partner and reports she is having consensual sex with him. She
has been using condoms for contraception. She has no history of sexually transmitted
infections. She has mild asthma for which she uses an inhaler as needed. She drinks
socially 3-5 drinks a day on the weekends. She does not use drugs or tobacco and
lives with her mother and 17-year old brother. Her physical exam is unremarkable,
uterine size is approximately 8 weeks. Ultrasound confirms intrauterine pregnancy at 8
weeks with fetal heart motion present.
Discussion questions:
1. What is options counseling and how would you counsel this patient about her
options?
-

Counseling should focus on the options of abortion, adoption or


continuation of the pregnancy
If the patient has already made a decision, less time is required for this
counseling. For the undecided patient, this counseling may take place
over more than one visit

2. Is this patient a victim of sexual abuse? Should this case be reported to


authorities?
-

States have different definitions of sexual coercion/rape. In some states,


a 5 year discrepancy in age (such as in this case) for a girl under 16 years
old constitutes statutory rape and would need to be reported. It is
important to have access to state laws for sexually active young minors.
Rape must be reported to authorities

3. Can the patient consent for termination herself or must she have parental consent
since she is a minor?
-

States have different regulations regarding parental notification/consent


of abortion for minors. Some states have no notification/consent laws: an
abortion can occur at the request of the minor. In some states, one parent
or legal guardian must be notified by phone or in writing and/or consent
to the procedure. It is important to have access to state laws for minors
who request abortion.
Other laws that are state dependent involve mandatory waiting times
(from 12-48 hours) before abortion and state directed counseling

(discussing material chosen by the state to all women undergoing


abortion). It is important to know your own states laws.

4. What types of abortion is this patient eligible for, given her gestational age of 8
weeks?
-

Medication abortion is available up to 7 weeks gestational age per the


FDA protocol and up to 9 weeks with the evidence-based protocol used
by most abortion clinics. The patient is also eligible for surgical abortion,
a D&C. Surgical abortion may be performed throughout the second and
early third trimester.

5. How is a surgical abortion performed?


-

Surgical abortion at 8 weeks gestation is a D&C (dilation and curettage).


Typically, the procedure is done in an outpatient setting under
paracervical block with or without mild IV sedation. The cervix is serially
dilated to accommodate a suction curette which is then attached either to
an MVA (manual vacuum aspiration) syringe or to an electric suction
machine. The tissue obtained is then inspected to ensure the presence of
a gestational sac.

6. How is a medication abortion performed?


-

The patient takes 1-3 tablets of mifepristone PO (RU-486, an antiprogestin), then 6-72 hours later inserts 4 tablets of misoprostol
intravaginally. Other protocols use oral or buccal misoprostol. The
pregnancy is usually passed within 4-6 hours of misoprostol placement.
The patient is given a follow-up appointment where an ultrasound or
beta hcg is obtained to confirm completion of the abortion.

7. What are the potential complications of abortion?


-

Complications include bleeding, infection and perforation of the uterus.


All these complications are rare. Most recently, 5 women who underwent
medication abortion in the US developed a rare and fatal infection with
clostridium sordelli. The CDC is currently investigating these deaths. The
overall mortality of first-trimester abortion remains lower than that of
bringing a pregnancy to term.

8. How should this patient be counseled about contraception?


-

She should be counseled about all forms of contraception and, regardless


of method chosen, she should be counseled about and given an advance
prescription for Plan B, emergency contraception.

9. How should this patient be counseled and managed regarding STD prevention?
-

She should be counseled about condom use and STD prevention. Selfesteem/education counseling: An evaluation of this patients social
situation and the reasons for her having a sexual relationship with a 21year old should be explored. She should be offered an STD screen,
including gonorrhea and chlamydia cultures and blood tests for HIV,
syphilis and hepatitis B.

References:
Obstetrics and Gynecology by Beckmann 5th Edition, 2006; Chapter 2 Ethics in Obstetrics and
Gynecology. Pages 28-29 & Chapter 14 Abortion. Pages 158-159.
Essentials of Obstetrics and Gynecology by Hacker and Moore 4th Edition, 2004; Chapter 27 Family
Planning: Contraception, Sterilization, and Abortion. Pages 350-351.

TEACHING CASE 34
Vulvar and Vaginal Disease
Student Handout
Clinical Case:
A 20-year-old female college student G2P2 comes to see you because of a persistent
vaginal discharge and also seeking contraceptive advice. She and her boyfriend have
been sexually active for 6 months. They use condoms most of the time, but she is
interested in using something with a lower failure rate for birth control. She has
regular menses and no significant past medical or gynecologic history. She describes
the discharge as yellowish and also notes mild vulvar irritation. On physical exam, she
has normal external female genitalia without lesions or erythema, a gray/yellow
discharge on the vaginal walls and pooled in the posterior fornix. Her cervix is grossly
normal but bleeds easily with manipulation. The bimanual exam is unremarkable.
Laboratory testing reveals Vaginal fluid pH = 7 and vaginal wet prep positive for mobile
flagellated organisms.

Discussion Questions:
1. What is your differential diagnosis?
2. What is the most likely diagnosis?
3. What is your management plan for this patient?
4. What are the additional issues you would want to discuss with this patient?
5. What contraceptive options would be appropriate for this patient?
6. Would you recommend screening for additional sexually transmitted infections
in this patient and if so, how?

References:
Obstetrics and Gynecology by Beckmann 5th Edition, 2006; Chapter 27 Vulvitis and Vaginitis. Pages 265272.
Essentials of Obstetrics and Gynecology by Hacker and Moore 4th Edition, 2004; Chapter 23 Pelvic
Infections. Pages 296-308.

Vulvar and Vaginal Disease


Preceptor Handout

Vaginal and vulvar symptoms are frequent patient concerns. In order to provide
appropriate care, the physician must understand the common etiologies of these
problems, as well as appropriate diagnostic and management options.
The APGO Educational Objectives related to this topic are the following:
A. Diagnose and manage a patient with vaginitis*
B. Interpret a wet mount microscopic examination*
C. Describe dermatologic disorders of the vulva*
D. Evaluate a patient with vulvar symptoms*

*Designated as Priority One in the APGO Medical Student Educational Objectives, 8th
Edition

Vulvar and Vaginal Disease


Preceptor Handout
Clinical Case:
A 20-year-old female college student G2P2 comes to see you because of a persistent
vaginal discharge and also seeking contraceptive advice. She and her boyfriend have
been sexually active for 6 months. They use condoms most of the time, but she is
interested in using something with a lower failure rate for birth control. She has regular
menses and no significant past medical or gynecologic history. She describes the
discharge as yellowish and also notes mild vulvar irritation. On physical exam, she has
normal external female genitalia without lesions or erythema, a gray/yellow discharge
on the vaginal walls and pooled in the posterior fornix . Her cervix is grossly normal but
bleeds easily with manipulation. The bimanual exam is unremarkable.
Laboratory testing reveals Vaginal fluid pH = 7 and vaginal wet prep positive for mobile
flagellated organisms.
Discussion Questions:
1. What is your differential diagnosis?
BV
Trichomonas
GC
Chlamydia
2. What is the most likely diagnosis?
Trichomonas
3. What is your management plan for this patient?
Treatment with a 2 gram single oral dose of metronidazole or 500 mg
oral metronidazole twice daily for 7 days
Sexual partner must be treated simultaneously and treatment of both
partners should be completed before resumption of sexual activity
Side effects of metronidazole treatment including a disulfiram-like
reaction should be discussed with the patient and patient should be
encouraged to abstain from alcohol during and for 3 days after treatment
with metronidazole.
4. What are the additional issues you would want to discuss with this patient?
STI protection
contraception

5. What contraceptive options would be appropriate for this patient?


A variety of contraceptive methods may be appropriate to achieve
effective contraception in this patient including oral contraceptives,
patches, ring or injection.
She is not a good candidate for sterilization (due to age), IUD (due to STI
history) and condom/diaphragm/spermicide (due to non-compliance).
The effectiveness, precautions, contraindications and method
administration should be discussed with the patient so that she can make
an informed choice.
It is important for the student to realize that a patient may have more
than one clinical issue to discuss.
6. Would you recommend screening for additional sexually transmitted infections in
this patient and if so, how?
Yes; with serologic testing for Hepatitis B, Syphilis, HIV and cervical
cultures for Gonorrhea and Chlamydia.
She should also have cervical cytology if not done recently.

References:
Obstetrics and Gynecology by Beckmann 5th Edition, 2006; Chapter 27 Vulvitis and Vaginitis. Pages 265272.
Essentials of Obstetrics and Gynecology by Hacker and Moore 4th Edition, 2004; Chapter 23 Pelvic
Infections. Pages 296-308.

TEACHING CASE 35
Sexually Transmitted Infections
Student Handout
Clinical Case:
A 16-year-old G1P1, LMP one week ago, presents with a one-week history of severe
lower abdominal pain. Pain is constant, bilateral and accompanied by fever and chills.
She has had some nausea and several episodes of vomiting. She has been sexually active
for 3 years and has had unprotected intercourse with several partners. She denies
irregular bleeding, dysmenorrhea or dyspareunia. Past medical history is negative
except for childhood illness. Past surgical history is remarkable for tonsillectomy as a
child and an uncomplicated vaginal delivery one year ago.
Physical exam reveals an ill appearing 16-year-old who is afebrile and has a pulse of 94
bpm, BP 124/82 and a respiratory rate 22 breaths/minute. On examination of the
abdomen, there is bilateral lower abdominal tenderness and the abdomen is slightly
distended with rebound, negative psoas and Murphys signs. Pelvic exam reveals the
BUS negative and the vagina pink, moist. There is a purulent discharge from the cervical
os and the cervix appears indurated. The uterus is in the midline position and is soft
and tender to palpation. There is bilateral adnexal fullness and moderate tenderness.
Laboratory evaluation includes positive GC, negative RPR and WBC 17.6 with a left shift.
Urinalysis is remarkable for few WBCs, no bacteria, 3+ ketones and negative urine HCG.

Discussion questions:
1. What is your differential diagnosis?
2. What is the most likely diagnosis?
3. What are the most likely organisms responsible for this condition?
4. What are the common presenting signs and symptoms for this condition?
5. What is the definitive diagnostic tool for equivocal cases?
6. What criteria will you use to determine inpatient vs. outpatient treatment?
7. What is your management and followup plan?
8. If this condition went untreated, what would be the possible sequelae?

References:
Obstetrics and Gynecology by Beckmann 5th Edition, 2006; Chapter 28 Sexually Transmitted Diseases.
Pages 273-287.
Essentials of Obstetrics and Gynecology by Hacker and Moore 4th Edition, 2004; Chapter 23 Pelvic
Infections. Pages 296-308.
Centers for Disease Control and Prevention. Sexually Transmitted Disease Treatment Guideline 2006.
www.cdc.gov/std/treatment/

Sexually Transmitted Infections


Preceptor Handout
To prevent sexually transmitted infections and urinary tract infections, the physician
should understand their basic epidemiology, diagnosis and management. The potential
impact of acute or chronic salpingitis is significant. Early recognition and optimal
management may help prevent the long-term sequelae of tubal disease.
The APGO Educational Objectives related to this topic are the following:
A. Differentiate the signs and symptoms of the following STIs:*
1. Gonorrhea
2. Chlamydia
3. Herpes simplex virus
4. Syphilis
5. Human papillomavirus infection
6. Human immunodeficiency virus (HIV) infection
7. Hepatitis B virus infection
B. List the physical and clinical finding in the following STIs:*
1. Gonorrhea
2. Chlamydia
3. Herpes simplex virus
4. Syphilis
5. Human papillomavirus infection
6. Human immunodeficiency virus (HIV) infection
7. Hepatitis B virus infection
C. Describe the methods of evaluation for the following STIs:*
1. Gonorrhea
2. Chlamydia
3. Herpes simplex virus
4. Syphilis
5. Human papillomavirus infection
6. Human immunodeficiency virus (HIV) infection
7. Hepatitis B virus infection
D. Describe the management of the following STIs:*
1. Gonorrhea
2. Chlamydia
3. Herpes simplex virus
4. Syphilis
5. Human papillomavirus infection
6. Human immunodeficiency virus (HIV) infection
7. Hepatitis B virus infection

E. Describe the pathogenesis of salpingitis *


F. List the signs of symptoms of salpingitis *
G. Describe the management of salpingitis *
H. Identify the long-term sequelae of salpingitis including: tuboovarian abscess, chronic salpingitis, ectopic pregnancy, infertility*
I. Counsel the patient about the public health concerns for STIs,
including screening programs, costs, prevention and
immunizations, and partner evaluation and treatment
J. Describe the diagnosis and management of UTIs *

*Designated as Priority One in the APGO Medical Student Educational Objectives, 8 th


Edition

Sexually Transmitted Infections


Preceptor Handout
Clinical Case:
A 16-year-old G1P1, LMP one week ago, presents with a one-week history of severe
lower abdominal pain. Pain is constant, bilateral and accompanied by fever and chills.
She has had some nausea and several episodes of vomiting. She has been sexually active
for 3 years and has had unprotected intercourse with several partners. She denies
irregular bleeding, dysmenorrhea or dyspareunia. Past medical history is negative
except for childhood illness. Past surgical history is remarkable for tonsillectomy as a
child and an uncomplicated vaginal delivery one year ago.
Physical exam reveals an ill appearing 16-year-old who is afebrile and has a pulse of 94
bpm, BP 124/82 and a respiratory rate 22 breaths/minute. On examination of the
abdomen, there is bilateral lower abdominal tenderness and the abdomen is slightly
distended with rebound, negative psoas and Murphys signs. Pelvic exam reveals the
BUS negative and the vagina pink, moist. There is a purulent discharge from the cervical
os and the cervix appears indurated. The uterus is in the midline position and is soft
and tender to palpation. There is bilateral adnexal fullness and moderate tenderness.
Laboratory evaluation includes positive GC, negative RPR and WBC 17.6 with a left shift.
Urinalysis is remarkable for few WBCs, no bacteria, 3+ ketones and negative urine HCG.
Discussion questions:
1. What is your differential diagnosis?
Salpingitis, appendicitis, ruptured ovarian cyst, UTI
2. What is the most likely diagnosis?
Acute pelvic inflammatory disease (PID)
3. What are the most likely organisms responsible for this condition?
Likely pathogens include N gonorrhoeae, C trachomatis, anaerobes, gramnegative bacteria and streptococci. All pelvic inflammatory disease (PID)
treatment regimens must therefore provide broad-spectrum coverage.
4. What are the common presenting signs and symptoms for this condition?
The most common presenting complaint of women with PID is lower
abdominal pain.

Associated symptoms include vaginal discharge, irregular bleeding,


dysmenorrhea, dyspareunia, dysuria, nausea, vomiting and fever.

Pelvic pain, fever and vaginal discharge are the most common findings if PID
is secondary to gonococcal infection. Patients may be asymptomatic if
chlamydia is the causative organism. Women who have gonococcal infection
have evidence of more acute inflammation (peritoneal signs, fever,

leukocytosis) than those who have nongonococcal infection because of the


endotoxin produced by N gonorrhoeae.

The clinical criteria necessary for the diagnosis of PID include:


o Abdominal tenderness +/- rebound
o Adnexal tenderness
o Cervical motion tenderness
o Plus one or more of the following: Gram stain of endocervix positive
for Gram negative intracellular diplococci, temperature >38 degrees C,
WBC>10,000, pus on culdocentesis or laparoscopy, pelvic abscess on
bimanual exam or ultrasound

Most women with acute PID present during the first half of the menstrual
cycle. Presentation later in the cycle indicates an infection of longer duration
and increases the likelihood of a tuboovarian abscess (TOA).

Atypical presentations of PID are common and complicate the differential


diagnosis. For example, the symptoms of Fitz-Hugh-Curtis syndrome may
mimic hepatitis or cholecystitis.

5. What is the definitive diagnostic tool for equivocal cases?


Laparoscopic findings of inflamed, dilated fallopian tubes, with purulent
discharge
6. What criteria will you use to determine inpatient vs. outpatient treatment?
Although broad-spectrum antibiotic coverage results in symptomatic
improvement in most patients, the risk of long-term sequelae remains
high.
Adolescents are at high risk for future reproductive complications due to
non-compliance. The CDC recommends serious consideration of
hospitalization for adolescents with PID.
Hospitalization is recommended if compliance is unpredictable, if the
diagnosis is uncertain, or if pelvic abscess is suspected.
Hospitalization is essential if the patient is pregnant, has HIV infection, is
too ill to tolerate (if vomiting, for example) or has failed to respond to
outpatient therapy within 48 hours.
7. What is your management and followup plan?
Treatment for both Chlamydia and Gonorrhea
Different regimens are adequate
o outpatient treatment: single dose IM Rocephin 250 mg plus 14
days of Doxycycline 100 mg po BID.
o inpatient treatment: Cefotetan 2 gms IV q 12 hours plus
Doxycycline 100 mg IV/po q 12 hours until > 24 hours of clinical
improvement, then continue outpatient regimen for a total of 14
treatment days

o All patients who are managed as outpatients must be reevaluated within


72 hours of the initiation of antibiotics.
o Sex partners of PID patients should be examined and treated if they had
sexual contact with the patient in the 60 days prior to the onset of
symptoms.
o Partners should be treated empirically for N gonorrhoeae and C
trachomatis, regardless of the apparent etiology of the PID or pathogens
isolated from the infected woman. Without treatment of infected
partners, risk of reinfection is high.
o Repeat cultures are recommended about 3-4 weeks after treatment of
Chlamydia infection to confirm treatment efficacy and rule out
asymptomatic reinfection.
8. If this condition went untreated, what would be the possible sequelae?
o Pelvic adhesions, tubal occlusion, chronic pelvic pain, ectopic pregnancy,
infertility

References:
Obstetrics and Gynecology by Beckmann 5th Edition, 2006; Chapter 28 Sexually Transmitted Diseases.
Pages 273-287.
Essentials of Obstetrics and Gynecology by Hacker and Moore 4th Edition, 2004; Chapter 23 Pelvic
Infections. Pages 296-308.
Centers for Disease Control and Prevention. Sexually Transmitted Disease Treatment Guideline 2006.
www.cdc.gov/std/treatment/

TEACHING CASE 36
Pelvic Relaxation and Urinary Incontinence
Student Handout
Clinical Case:
A 75-year-old woman G5P5 presents for an annual exam and reports having a fullness
in the vaginal area. The symptom is more noticeable when she is standing for a long
period of time. She does not complain of urinary or fecal incontinence. She has no other
urinary or gastrointestinal symptoms. There has been no vaginal bleeding. Her past
medical history is significant for well-controlled hypertension and chronic bronchitis.
She has never had surgery.
Pelvic exam reveals normal appearing external genitalia except for generalized atrophic
changes. The vagina and cervix are without lesions. A cystocele and rectocele are noted.
The cervix descends to the introitus with the patient in an upright position. Uterus is
normal size. Right and left ovaries are not palpable. No rectal masses are noted. Rectal
sphincter tone is slightly decreased. The patient prefers non-surgical treatment. A
pessary is placed and you prescribe vaginal estrogen to address atrophic changes.
Discussion Questions:
1. What increases this patients risk for pelvic organ prolapse?
2. What are the symptoms of pelvic organ prolapse?
3. What are the different types of pelvic organ prolapse?
4. What are the different types of urinary incontinence?
5. What is the role of vaginal estrogen in patients with pelvic relaxation?
6. When is surgery indicated?
7. What are nonsurgical treatments?

References:
Obstetrics and Gynecology by Beckmann 5th Edition, 2006; Chapter 29 Pelvic Relaxation, Urinary
Incontinence, and Urinary Tract Infection. Pages 288-298.
Essentials of Obstetrics and Gynecology by Hacker and Moore 4th Edition, 2004; Chapter 24
Genitourinary Dysfunction: Pelvic Organ Prolapse, Urinary Incontinence, and Infections. Pages 309-324.

Pelvic Relaxation and Urinary Incontinence


Preceptor Handout

Patients with conditions of pelvic relaxation and urinary incontinence present in a variety
of ways. The physician should be familiar with the types of pelvic relaxation and
incontinence, and the approach to management of these patients.
The APGO Educational Objectives related to this topic are the following:
A. Incorporate screening questions for urinary incontinence when eliciting a
patient history*
B. Discuss the difference between stress, urge and overflow incontinence
C. Obtain pertinent history components to differentiate between incontinence
types
D. Identify the following elements on physical exam: cystocele, rectocele, vaginal
vault/uterine prolapse
E. List behavioral, medical and surgical methods to appropriately treat
incontinence and pelvic organ prolapse

*Designated as Priority One in the APGO Medical Student Educational Objectives, 8 th


Edition

Pelvic Relaxation and Urinary Incontinence


Preceptor Handout
Clinical Case:
A 75-year-old woman G5P5 presents for an annual exam and reports having a fullness
in the vaginal area. The symptom is more noticeable when she is standing for a long
period of time. She does not complain of urinary or fecal incontinence. She has no other
urinary or gastrointestinal symptoms. There has been no vaginal bleeding. Her past
medical history is significant for well-controlled hypertension and chronic bronchitis.
She has never had surgery.
Pelvic exam reveals normal appearing external genitalia except for generalized atrophic
changes. The vagina and cervix are without lesions. A cystocele and rectocele are noted.
The cervix descends to the introitus with the patient in an upright position. Uterus is
normal size. Right and left ovaries are not palpable. No rectal masses are noted. Rectal
sphincter tone is slightly decreased. The patient prefers non-surgical treatment. A
pessary is placed and you prescribe vaginal estrogen to address atrophic changes.
Discussion Questions:
1. What increases this patients risk for pelvic organ prolapse?

Pelvic fascia, ligaments, and muscles may become attenuated from


pregnancy, labor and delivery.
Increased intraabdominal pressure (from a chronic cough or habitual
straining or heavy lifting) may predispose to prolapse
Atrophy may play a role in initiating or worsening of pelvic relaxation

2. What are the symptoms of pelvic organ prolapse?

Heaviness or fullness in the pelvis or something falling out are


common complaints
Most symptoms worsen after prolonged standing and are relieved by
lying down.
Urinary frequency, urgency, incontinence or retention are associated
with anterior vaginal wall prolapse

3. What are the different types of pelvic organ prolapse?

Anterior vaginal prolapse cystourethrocele, cystocele


Apical vaginal prolapse uterovaginal, vaginal vault (post surgery)
Posterior vaginal prolapse enterocele, rectocele
Complete procidentia refers to complete prolapse of the uterus

4. What are the different types of urinary incontinence?

Stress urinary incontinence involuntary loss of urine due to a sudden


increase in intraabdominal pressure in the absence of a bladder
contraction
Urge incontinence involuntary detrusor contractions
Mixed urinary incontinence is the combination of stress and urge types
Overflow incontinence hypotonic bladder or detrusor areflexia or
outflow obstruction and retention are possible causes; neurogenic
bladder
Total incontinence-can result from vesicovaginal fistulas from pelvic
surgery and/or radiation
Urethral syndrome (urinary tract symptoms without infection or
bladder and urethral pathology) can cause incontinence

5. What is the role of vaginal estrogen in patients with pelvic relaxation?

Pessaries may cause vaginal irritation and ulceration. They are better
tolerated when the vaginal epithelium is well estrogenized
Exogenous estrogen may be required in the hypoestrogenic patient
Oral estrogen is not recommended to prevent or treat urinary
incontinence

6. When is surgery indicated?

Desire for definitive surgical correction


Recurrent vaginal ulcerations with a pessary
Stress incontinence that the patient finds unacceptable

7. What are nonsurgical treatments?

Pelvic muscle exercises


Fitting the patient with a vaginal pessary
Medical therapy
Behavioral therapy
Injection of bulking agents
Vaginal cones and electronic devices are also described

References:
Obstetrics and Gynecology by Beckmann 5th Edition, 2006; Chapter 29 Pelvic Relaxation, Urinary
Incontinence, and Urinary Tract Infection. Pages 288-298.
Essentials of Obstetrics and Gynecology by Hacker and Moore 4th Edition, 2004; Chapter 24
Genitourinary Dysfunction: Pelvic Organ Prolapse, Urinary Incontinence, and Infections. Pages 309-324.

TEACHING CASE 37

Endometriosis
Student Handout
Clinical Case:
A 28-year-old woman G0P0 makes an appointment because of the inability to conceive
for the past two years. She has never used oral contraceptives and she and her husband
have not used any form of birth control for over two years. Her menarche occurred at
the age of 12 and her menses became very painful in her late teens. Since, she has had
chronic cyclical pelvic pain, which has been getting progressively worse over the years.
This pain is incapacitating at time and is usually located in multiple areas (midline
abdomen, pelvis and lower back). In addition to the pain, her menstrual periods have
become increasingly frequent and heavy. She experiences pain with intercourse that
has progressively gotten worse. She denies any non-cyclical vaginal bleeding, discharge
and weight loss. She states that her 22-year-old younger sister has always had very
painful menses.
On physical exam you notice a 4mm hyperpigmented, raised, non-tender nodule in the
umbilical area. The pelvic exam showed a fixed, retroverted uterus. The uterosacral
ligaments on both sides are nodular and mildly tender. A 5 cm right adnexal mass was
palpated and tender. The left ovary was slightly enlarged. Ultrasound of the abdomen
showed an echogenic cystic mass in the right ovary, which measured 4 cm in diameter.
The left ovary was reported as normal.
Discussion Questions:
1. What are the symptoms of endometriosis?
2. What are some of the physical exam findings associated with endometriosis?
3. What is the differential diagnosis in a patient you suspect has endometriosis?
4. How do you make the diagnosis of endometriosis?
5. What staging protocols are used?
6. What are the treatment options?

References:
Obstetrics and Gynecology by Beckmann 5th Edition, 2006; Chapter 30 Endometriosis. Pages 299-307.
Essentials of Obstetrics and Gynecology by Hacker and Moore 4th Edition, 2004; Chapter 26
Endometriosis and Adenomyosis. Pages 334-340.

Endometriosis
Preceptor Handout

Endometriosis is a common problem in women of reproductive age, which may result in


pelvic pain, infertility and menstrual dysfunction.
The APGO Educational Objectives related to this topic are the following:
A. Describe the theories of the pathogenesis of endometriosis
B. List the common sites of endometriosis implants
C. List the chief complaints a patient with endometriosis may present *
D. Identify the symptoms of endometriosis in a female patient presenting with
abdominal pain *
E. List the physical exam findings a patient with endometriosis may have *
F. Describe how endometriosis is diagnosed
G. Outline a plan for managing endometriosis

*Designated as Priority One in the APGO Medical Student Educational Objectives, 8 th


Edition

Endometriosis
Preceptor Handout
Clinical Case:
A 28-year-old woman G0P0 makes an appointment because of the inability to conceive
for the past two years. She has never used oral contraceptives and she and her husband
have not used any form of birth control for over two years. Her menarche occurred at
the age of 12 and her menses became very painful in her late teens. Since, she has had
chronic cyclical pelvic pain, which has been getting progressively worse over the years.
This pain is incapacitating at time and is usually located in multiple areas (midline
abdomen, pelvis and lower back). In addition to the pain, her menstrual periods have
become increasingly frequent and heavy. She experiences pain with intercourse that
has progressively gotten worse. She denies any non-cyclical vaginal bleeding, discharge
and weight loss. She states that her 22-year-old younger sister has always had very
painful menses.
On physical exam you notice a 4mm hyperpigmented, raised, non-tender nodule in the
umbilical area. The pelvic exam showed a fixed, retroverted uterus. The uterosacral
ligaments on both sides are nodular and mildly tender. A 5 cm right adnexal mass was
palpated and tender. The left ovary was slightly enlarged. Ultrasound of the abdomen
showed an echogenic cystic mass in the right ovary, which measured 4 cm in diameter.
The left ovary was reported as normal.
Discussion Questions:
1. What are the symptoms of endometriosis?

Wide variety of symptoms independent of location and severity of disease


Classic symptoms include dysmenorrhea, dyspareunia, infertility, abnormal
bleeding, dyschezia, and pelvic pain
Symptoms often most severe just prior to or during menstruation
Can cause hemothorax, cyclic nose bleeds, low back pain, bladder pain
and/or frequency, and fatigue

2. What are some of the physical exam findings associated with endometriosis?

Tender uterosacral nodularity on rectovaginal exam


Fixed, retroflexed uterus
Ovaries may be palpable (enlarged), tender, mobile or fixed
The physical exam may not have any specific findings

3. What is the differential diagnosis in a patient you suspect has endometriosis?

Chronic pelvic pain-consider chronic pelvic inflammatory disease, adhesions,


gastrointestinal conditions.
Dysmenorrhea-consider causes of primary and secondary dysmenorrhea
Dyspareunia-consider chronic pelvic inflammatory disease, ovarian cysts,
and symptoms related to a retroverted uterus

4. How do you make the diagnosis of endometriosis?

History and physical exam are first steps due to variety of presentations
Direct visualization is needed for establishing a diagnosis
Tissue biopsy makes definitive diagnosis
Pelvic sonogram cannot make diagnosis, but can exclude other conditions

5. What staging protocols are used?

The revised American Fertility Societys (AFS) staging system is generally


used to stage endometriosis in the infertile patient. In the AFS system, points
are assigned for size and depth of implants and for the severity of adhesions
in various locations. Stages I through IV are assigned on the basis of points.
Management of endometriosis can be guided by the stage of disease and the
desire for fertility.
The American Society of Reproductive Medicine (ASRM) protocol correlates
fertility potential with a quantified stage of disease. The staging includes the
color of lesions, the percentage of surface involved and a detailed description
of endometriomas.

6. What are the treatment options?

Depend on presenting symptoms and severity, location and severity, desire


for future childbearing, age, and possible gastrointestinal or urinary tract
involvement
Expectant management may be considered for patients with minimal
symptoms and disease and/or patients who are trying to conceive
Medical therapy includes combined estrogen and progestin oral
contraceptives, progestins alone, danazol (17-alpha ethinyl testosterone
derivative), and gonadotropin-releasing hormone (GnRH)
Surgical management ranges from conservative to definitive approaches

References:
Obstetrics and Gynecology by Beckmann 5th Edition, 2006; Chapter 30 Endometriosis. Pages 299-307.
Essentials of Obstetrics and Gynecology by Hacker and Moore 4th Edition, 2004; Chapter 26
Endometriosis and Adenomyosis. Pages 334-340.

CLINICAL CASE 38
Chronic Pelvic Pain
A 24-year old woman presents to you as a self-referral for pelvic pain. She describes a
four-year history of intermittent lower abdominal and pelvic pain that is now constant
in nature. The pain is always present, sometimes sharper in the left lower quadrant
and not related to menses. She has occasional nausea and is sometimes constipated.
Nothing makes the pain better or worse. Over the years, she has used acetaminophen
and ibuprofen, and has not found any relief. She began her menses at age 13 and they
have come on a regular monthly basis. She experiences some premenstrual bloating
and has cramps with her periods, and reports discomfort at other times of the month.
She had a trial of oral contraceptives and then a subsequent laparoscopy by a prior
gynecologist. She was told that everything looked normal. She is otherwise a healthy
non-smoker, but reports that this pain is making her life miserable.
She has a bachelors degree from a local college, works as computer processor and lives
at home with her parents. She has never been sexually active. Upon further questioning,
she reports that her oldest brother sexually abused her as a child.
Physical exam
Somewhat flattened affect, but smiles occasionally. 5 feet 4 inches; 142 pounds.
Trapezius and paraspinous muscles tender on palpation. No costovertebral angle
tenderness. Abdomen is soft with 2 well-healed pelviscopy incisions. There is no
rebound or guarding or mass. Tenderness is elicited with deep palpation of the lower
quadrants. External genitalia, vagina and cervix are normal. Uterus is mid-position,
mobile and the adnexa are mildly tender. The rectal vault is palpably normal with soft
stool that is heme negative.
Differential diagnosis of chronic pelvic pain

Gynecologic origin
Gastrointestinal disorders
Urinary problems
Musculoskeletal disease
Pain processing disorders
Psychiatric and psychological

Management plan
The patient was counseled about the multiple possible causes of chronic pelvic pain.
The provider was empathic and sensitive in regards to this challenging problem. A plan

of care was devised jointly and she was scheduled for a follow-up. The patients
previous records and operative report was obtained and reviewed. On a subsequent
visit, the patient did note that the pain worsens when her older brother returns home
for family holidays. She reports that she has never mentioned this to the therapist that
she has recently started seeing. A trial of low dose tricyclic antidepressants was
initiated which helped the patient with sleeping, but did not make the pain go away.
The patient continued to follow up at regularly scheduled intervals with her
gynecologist and therapist, and had less emergency room visits.
Teaching points
Chronic pelvic pain can be defined as cyclic pain of 6 months duration or non-cyclic pain
of 3 months duration and the pain interferes with normal activities. The problem of
chronic pelvic pain is under-recognized. It may affect 15% to 24% of American women
and accounts for a large proportion of office visit time and many invasive surgical
procedures.
Chronic pelvic pain can be derived from a variety of sources, including gynecologic,
gastrointestinal, rheumatologic, musculoskeletal, urologic or psychiatric. It can be
difficult to diagnose the etiology and can be challenging to treat. The health care
provider must perform a thorough history and physical exam, which are often much
more valuable in making a diagnosis than any laboratory or radiologic tests.
Patients present to different specialists based on their belief of what is causing the pain.
Gastrointestinal diseases may cause symptoms such as nausea, vomiting, bloating or
changes in bowel habits. Urinary tract disorders my cause dysuria, urgency or vague
pelvic discomfort. Patients need to be asked about fatigue, sleep disturbances, or mood
disorders and fibromyalgia and depression considered. Patients also need to be queried
about physical and sexual abuse, or any history of substance abuse. Musculoskeletal
disorders can be determined by a thorough motor and sensory examination, with
attention to the back, hips and legs.
Possible gynecologic causes of chronic pelvic pain include endometriosis, adenomyosis,
chronic pelvic infection or adhesions. A normal laparoscopy does not completely rule
out endometriosis, as the changes can be subtle and occasionally missed. Providers can
consider an empiric trial of oral contraceptives or GnRH agonists after non-gynecologic
causes have been ruled out. Some providers recommend a trial of antibiotics or nonsteroidal anti-inflammatories for potential infectious causes. In the case of depression,
whether overt or covert, antidepressants should be initiated.
Even when the etiology is determined, chronic pelvic pain can be difficult to treat. The
patient may need to be seen regularly and provided with much support. Comanagement with a psychologist, social worker or therapist may be helpful.

UNIT THREE:
SECTION B

GYNECOLOGY - Breasts

39. Disorders of the Breasts

TEACHING CASE 39
Disorders of the Breast
Student Handout
Clinical Case:
A 56-year-old woman G0P0 made an appointment to see her gynecologist because she
was concerned about a small lump in her right breast that she has been able to feel for 2
months. She has not had breast problems in the past and does not have a family history
of breast cancer. There are no apparent skin changes, asymmetry or skin dimpling.
Axillary or subclavian lymph nodes are not palpable. Breasts are symmetric, diffusely
cystic and non-tender. There is an area of firmness approximately 1 cm in diameter
with indiscreet boarders at 9:00 on her right breast. The area is slightly different in
consistency than the rest of the surrounding tissue. The patient was sent for a
mammogram that revealed dense breast tissue, but no discrete mammographic
abnormalities.

Discussion Questions:
1. What is the proper technique to perform a breast exam?
2. What are common risk factors for breast cancer?
3. What is your next step in this patients management?
4. What are some common benign breast disorders?

References:
Obstetrics and Gynecology by Beckmann 5th Edition, 2006; Chapter 32 Disorders of the Breast. Pages
316-327.
Essentials of Obstetrics and Gynecology by Hacker and Moore 4th Edition, 2004; Chapter 30 Breast
Disease: A Gynecologic Perspective. Pages 364-371.

Disorders of the Breast


Preceptor Handout

Every physician should understand the basic approach to evaluating the common
symptoms associated with the breast.
The APGO Educational Objectives related to this topic are the following:
A. Demonstrate the performance of a comprehensive breast exam*
B. Discuss the diagnostic approach to a woman with the chief complaint of breast
mass, nipple discharge or breast pain*
C. List history and physical findings that may suggest:*
1. Mastitis
2. Carcinoma
3. Fibrocystic changes
4. Intraductal papilloma
5. Fibroma

*Designated as Priority One in the APGO Medical Student Educational Objectives, 8th
Edition

Disorders of the Breast


Preceptor Handout
Clinical Case:
A 56-year-old woman G0P0 made an appointment to see her gynecologist because she
was concerned about a small lump in her right breast that she has been able to feel for 2
months. She has not had breast problems in the past and does not have a family history
of breast cancer. There are no apparent skin changes, asymmetry or skin dimpling.
Axillary or subclavian lymph nodes are not palpable. Breasts are symmetric, diffusely
cystic and non-tender. There is an area of firmness approximately 1 cm in diameter
with indiscreet boarders at 9:00 on her right breast. The area is slightly different in
consistency than the rest of the surrounding tissue. The patient was sent for a
mammogram that revealed dense breast tissue, but no discrete mammographic
abnormalities.
Discussion Questions:
1. What is the proper technique to perform a breast exam?

Annual exam
Inspection (with patient sitting) - check for contour, symmetry, skin
changes or nipple retraction
Palpation (in upright and supine positions) of breast, areola and nipples;
examine axillae; examine supraclavicular fossae

2. What are common risk factors for breast cancer?

The commonly quoted projection of the risk of breast cancer (1 in 8


women) represents a cumulative lifetime risk. For a woman aged 50-59
years, the lifetime risk of having a breast cancer diagnosis is 1in 36, while
for a woman-aged 70-79 years, the risk increases to 1 in 24.
Risk factors include patient age, prior personal history, family history,
genetics (BRCA mutations), nulliparity, late childbearing, early menarche,
late menopause, fibrocystic changes with atypia, history of breast
radiation, hormone exposure, obesity, alcohol, etc.

3. What is your next step in this patients management?

A persistent palpable breast mass requires evaluation. Mammography


alone is not sufficient to rule out malignant pathology in a patient with a
palpable breast mass. Ultrasonography or magnified mammographic
imaging of the breast containing the mass may provide additional
information and may identify cystic structures or variations in normal
breast architecture that account for the palpable abnormality. When cyst
aspiration is performed, the fluid may be discarded if it is clear

(transparent and not bloody) and the mass disappears. Otherwise, the
patient should be considered a candidate for a breast biopsy.

Solid masses require histologic diagnosis in most cases. Fine-needle


aspiration or stereotactic needle biopsy may be an alternative to open
breast biopsy in certain cases. If breast cancer or specific benign
condition is not detected by fine-needle aspiration or needle core biopsy,
open biopsy is necessary.

4. What are some common benign breast disorders?

Fibrocystic changes most common of all benign breast conditions;


commonly present as cyclic, bilateral, pain and engorgement
1. Nonproliferative lesion
2. Proliferative lesion (hyperplasia) atypia most common (50%)
3. Atypical hyperplasia increased risk for subsequent malignancy

Fibroadenoma second most common form of benign breast disease


1. Consists of fibrous and glandular tissue
2. Occurs in young women
3. Firm, painless, mobile mass

Intraductal Papilloma
1. Commonly found in peri and menopausal patients
2. Bloody, serous or turbid nipple discharge
3. Excisional biopsy is often needed

Galactocele
1. Cystic dilatation of a duct filled with thick, milky fluid
2. Secondary infection causes mastitis
3. Needle aspiration, often curative

References:
Obstetrics and Gynecology by Beckmann 5th Edition, 2006; Chapter 32 Disorders of the Breast. Pages
316-327.
Essentials of Obstetrics and Gynecology by Hacker and Moore 4th Edition, 2004; Chapter 30 Breast
Disease: A Gynecologic Perspective. Pages 364-371.

UNIT THREE:
SECTION C

GYNECOLOGY - Procedures

40. Gynecological Procedures

TEACHING CASE 40
Gynecologic Procedures
Student Handout
Clinical Case:
The patient is a 40 year old G0 who has menorrhagia due to a fibroid uterus. She has
anemia with a hematocrit of 27% despite oral iron therapy. She has periods lasting 1012 days each month. She also suffers from lupus and anti-phosphlipid antibody
syndrome, diagnosed when she was 25. Her manifestations mostly are arthritis, but she
has a history of a deep venous thrombosis (DVT) 6 years ago. Although her lupus
currently is not flaring, she takes prednisone 5 mg per day as well as coumadin 2.5 mg
per day. She does not have other medical problems and her only other surgery was a
tonsillectomy at age 16, during which she was told she had more than usual bleeding
but did not require transfusion. She desires definitive surgical management with
hysterectomy. She is married, works as an office manager, and never had children
because of her lupus. Her physical exam shows BP 120/70, weight of 160, height of
56. She has a number of small bruises on her extremities. Her uterus is palpable just
under her umbilicus, but is non-tender. Pelvic exam is only significant for the enlarged
uterus. Pelvic ultrasound confirms a large fibroid uterus, normal ovaries. Labs show
INR of 2.5, normal chem. panel.
Discussion Questions:
1. Describe the consent process for surgery for this patient.
2. What are the main surgical risks facing this patient?
3. What steps can you take to try to avoid these risks?
4. Which other health professionals would you consult both pre- and post-operatively?
5. What measures can you take post-operatively to assess for and diagnose
complications?
6. On post-operative day 2, the patient complains of feeling short of breath and her
pulse oximeter shows an oxygen saturation of 89% on room air. How would you
proceed?
References:
Obstetrics and Gynecology by Beckmann 5th Edition, 2006; Chapter 33 Gynecologic Procedures. Pages
328-337.
Essentials of Obstetrics and Gynecology by Hacker and Moore 4th Edition, 2004; Chapter 31
Gynecologic Procedures. Pages 372-384.

Gynecologic Procedures
Preceptor Handout
Evaluation and management of gynecologic problems frequently requires performing
diagnostic and therapeutic surgical procedures. Understanding the risks and benefits of
such procedures is important in counseling patients about their options for treatment and
reasons for having the procedures performed.
The APGO Educational Objectives related to this topic are the following:
A. Describe the components of pre-operative evaluation and planning, including the
complete medical history, the informed consent process and working with
consultants*
B. Describe the common peri-operative prophylactic measures, including steps taken to
reduce infections and deep vein thrombosis*
C. Describe the components of routine post-operative care*
D. List common post-operative complications*
E. Descsribe each procedure and list the indications and complications of each of the
following:
1. Colposcopy and cervical biopsy
2. Cone biopsy
3. Cryotherapy
4. Dilation and curettage
5. Electrosurgical excision of cervix
6. Endometrial biopsy
7. Hysterectomy
8. Hysterosalpingography
9. Hysteroscopy
10. Laparoscopy
11. Laser vaporization
12. Mammography
13. Needle aspiration of breast mass
14. Pelvic ultrasonography
15. Pregnancy termination
16. Vulvar biopsy
*Designated as Priority One in the APGO Medical Student Educational Objectives, 8th
Edition

Gynecologic Procedures
Preceptor Handout
Clinical Case:
The patient is a 40 year old G0 who has menorrhagia due to a fibroid uterus. She has
anemia with a hematocrit of 27% despite oral iron therapy. She has periods lasting 1012 days each month. She also suffers from lupus and anti-phosphlipid antibody
syndrome, diagnosed when she was 25. Her manifestations mostly are arthritis, but she
has a history of a deep venous thrombosis (DVT) 6 years ago. Although her lupus
currently is not flaring, she takes prednisone 5 mg per day as well as coumadin 2.5 mg
per day. She does not have other medical problems and her only other surgery was a
tonsillectomy at age 16, during which she was told she had more than usual bleeding
but did not require transfusion. She desires definitive surgical management with
hysterectomy. She is married, works as an office manager, and never had children
because of her lupus. Her physical exam shows BP 120/70, weight of 160, height of
56. She has a number of small bruises on her extremities. Her uterus is palpable just
under her umbilicus, but is non-tender. Pelvic exam is only significant for the enlarged
uterus. Pelvic ultrasound confirms a large fibroid uterus, normal ovaries. Labs show
INR of 2.5, normal chem. panel.
Discussion Questions:
1. Describe the consent process for surgery for this patient.
Pneumonic PREPARED
o Procedure
o Reasons for the procedure
o Expectations of the patient for the outcome, risks and recovery
o Preference of the patient for type of management
o Alternatives, including other procedures and medical management
o Risks, both general and those specific to the patient
o Expense, both medical and loss of work, worst case scenario
o Decision
2. What are the main surgical risks facing this patient?
Hemorrhage and transfusion
Thrombosis very difficult to balance these 2 risks
Lupus flare post-operatively
Infection even though she is on low dose prednisone she is likely
immunosuppressed
Damage to other pelvic organs

3. What steps can you take to try to avoid these risks?


Consider pre-operative transfusion, intravenous iron, etc to raise her hematocrit
Judicious discontinuation of her anti-coagulation
Pneumatic compression stockings, low-dose heparin immediately post-op
Peri-operative antibiotics
4. Which other health professionals would you consult both pre- and post-operatively?
Hematology for management of anti-coagulation
Rheumatology for management of lupus and possible steroid boost
Anesthesiology to alert them to her high risk status
5. What measures can you take post-operatively to assess for and diagnose
complications?
Serial hematocrits
Judicious re-start of her anti-coagulation after the immediate peri-operative
period
Urine output assessment
Continuous pulse oximetry
Physical exam
6. On post-operative day 2, the patient complains of feeling short of breath and her pulse
oximeter shows an oxygen saturation of 89% on room air. How would you proceed?
Spiral CT
Oxygen

References:
Obstetrics and Gynecology by Beckmann 5th Edition, 2006; Chapter 33 Gynecologic Procedures. Pages
328-337.
Essentials of Obstetrics and Gynecology by Hacker and Moore 4th Edition, 2004; Chapter 31
Gynecologic Procedures. Pages 372-384.

UNIT FOUR:

REPRODUCTIVE ENDOCRINOLOGY,
INFERTILITY AND RELATED TOPICS

41.
42.
43.
44.
45.
46.
47.
48.

Puberty
Amenorrhea
Hirsutism and Virilization
Normal and Abnormal Uterine Bleeding
Dysmenorrhea
Climacteric / Menopause
Infertility
Premenstrual Syndrome & Premenstrual Dysphoric Disorder

CLINICAL CASE 41
Puberty
At the end of this exercise the student will:
A. Be able to describe the physiological events that take place
B. The sequence of and expected ages of these changes.
A 15-year-old female comes in for exam because she has not had her period. She
seemed to be developing normally and had normal breast development that started
about 2 years ago; she has pubic hair. She met her developmental milestones and is of
normal height and weight. She has not had any significant medical illnesses. Her ROS is
negative and her family history is negative.
She is active in school and is a cheerleader. She works out with the team and runs. She
does well in school. She lives at home with her mom, dad and sister. She does not
currently have a boyfriend and has not been sexually active.
Physical exam:
Normal appearing adolescent, 100/60 with 130 lbs. and 57
HEENT-WNL:
Lungs-clear; heart-RRR no murmur or gallops; breast exam-Tanner 4 no abnormalities
noted; ABD- Normal; EXT Genitalia- appears normal with pubic hair Tanner 3; single
digital exam done of the vagina and palpates a normal vagina; rectal exam was negative;
uterus is palpated and feels to be normal size.
At this point, because her physical finding are considered to be WNL and it does not
seem as if there is an anatomic reason for her amenorrhea and slight delay in pubic hair
development that this likely just delayed puberty and reassurance can be given.
However patient is sent home with prescription for Provera to determine if her
endometrium is estrogen primed and therefore with intact pituitary and ovarian
function.
Discussion/teaching points:
The differential diagnosis for delayed puberty is constitutional delay, gonadal
dysgenesis, premature ovarian failure, androgen insensitivity syndrome, CNS tumor,
pan-hypopituitarism, and hypothyroidism.

The normal sequence of maturational events starts at age 11, with breast development
(thelarche), then pubic hair and axillary hair (adrenarche) and then menarche which
the mean age is 12.5. This patient did not start thelarche until age 13; therefore the
onset of menses will likely be delayed.

Tanner staging evaluates the stages of breast and pubic hear development
Tanner stage 1- pre-pubertal breast; Tanner stage 2-breast bud; Tanner stage 3-further
breast and areolar enlargement; Tanner stage 4-aerola and papilla form secondary
mound; Tanner stage 5-mature stage pubic hair development; Tanner stage 1-no pubic
hair; Tanner stage 2-sparse hair on labia majora; Tanner stage 3, -pubic hair to mons
pubis; Tanner stage 4 adult hair with no spread to thighs; Tanner stage 5- spread to
thighs.
The patient has normal anatomical findings, so this rules out androgen insensitivity as
well as mullerian agenesis. Additionally, one can treat with Provera to check for an
estrogen-primed endometrium. This will help determine if the pituitary and ovarian
hormones are adequate to produce estrogen and the hypothalamic/pituitary/ovarian
axis has not matured to the point of producing an ovulatory cycle and withdrawal bleed.
If the patient has a withdrawal bleed, reassurances can be given to her and her mother
that these are normal findings and she is just a little slower to mature. One could also
consider treating intermittently with Provera to ensure a regular withdrawal bleed and
to not interfere with the maturation process of the hypothalamic/pituitary/ovarian
axis.

TEACHING CASE 42

Amenorrhea

Student Handout
Clinical Case:
A 26-year-old G2P2 woman presents because of no periods for 9 months. She delivered
two full term healthy children vaginally and their ages are 5 and 3. She breastfed her
youngest for 1 year, menses returned right after she stopped, and were monthly and
normal until 9 months ago. She is not using any contraception, although intercourse is
infrequent. She feels very fatigued, has frequent headaches and has had trouble losing
weight. She has no history of abnormal Paps or STIs. She takes no medications. She is
married and works from home as a computer consultant. On exam, BP= 120/80, P= 64,
Ht= 58, Wt= 160 pounds. She appears tired but in no distress. Breasts show scant
bilateral milky white discharge with manual stimulation. Breast exam reveals no
masses, dimpling or retraction. Exam is otherwise normal, including pelvic exam. HCG
is negative.
Discussion Questions:
1. Does this patient have primary amenorrhea, secondary amenorrhea or
oligomenorrhea?
2. What is the differential diagnosis for this disorder?
3. What additional studies are needed?
4. Consider that this patient has a prolactin level of 130. The test, when repeated with
the patient fasting is 100. What is your next step?
5. How would your next step differ if the patient had normal labs with an estradiol level
of 30pcg/ml and an FSH of 2mIU. What treatment would you offer her? What is she
at risk for?

References:
Obstetrics and Gynecology by Beckmann 5th Edition, 2006; Chapter 36 Amenorrhea and Dysfunctional
Uterine Bleeding. Pages 359-364.
Essentials of Obstetrics and Gynecology by Hacker and Moore 4th Edition, 2004; Chapter 33
Amenorrhea, Oligomenorrhea, and Hyperandrogenic Disorders. Pages 398-408.

Amenorrhea
Preceptor Handout

The absence of normal menstrual bleeding may represent an anatomic or endocrine


problem. A systematic approach to the evaluation of amenorrhea will aid in the diagnosis
and treatment of its cause.
The APGO Educational Objectives related to this topic are the following:
A. Define amenorrhea and oligomenorrhea*
B. Describe the etiologies of amenorrhea and oligomenorrhea*
C. Describe the evaluation methods for amenorrhea and oligomenorrhea
D. Describe the treatment options for amenorrhea and oligomenorrhea
E. Counsel patients who decline therapy

*Designated as Priority One in the APGO Medical Student Educational Objectives, 8 th


Edition

Amenorrhea
Preceptor Handout
Clinical Case:
A 26-year-old G2P2 woman presents because of no periods for 9 months. She delivered
two full term healthy children vaginally and their ages are 5 and 3. She breastfed her
youngest for 1 yr, menses returned right after she stopped, and were monthly and
normal until 9 months ago. She is not using any contraception, although intercourse is
infrequent. She feels very fatigued, has frequent headaches and has had trouble losing
weight. She has no history of abnormal Paps or STIs. She takes no medications. She is
married and works from home as a computer consultant. On exam, BP= 120/80, P= 64,
Ht= 58, Wt= 160 pounds. She appears tired but in no distress. Breasts show scant
bilateral milky white discharge with manual stimulation. Breast exam reveals no
masses, dimpling or retraction. Exam is otherwise normal, including pelvic exam. HCG
is negative.
Discussion Questions:
1. Does this patient have primary amenorrhea, secondary amenorrhea or
oligomenorrhea?
Primary amenorrhea definition: age 14 without secondary sex characteristics,
age 16 with secondary sex characteristics
Secondary amenorrhea definition: 6 months of amenorrhea after a history of
normal menses
Oligomenorrhea: menses lasting >40 days, <6 months
2. What is the differential diagnosis for this disorder?
Pregnancy
Prolactin excess
About 1/3 have no symptoms
Pituitary adenoma
Thyroid dysfunction
Extremely elevated TSH may cause prolactin to rise
Medications
phenothiaziness, antidepressants, hypertension meds,
metoclopromide, all can cause elevated prolactin, narcotics,
estrogens can result in anovulation
Hypothalamic amenorrhea
Weight loss, excessive exercise, psychologic stress, chronic illness
related stress, brain or head injury, brain tumor
Outflow tract obstruction Ashermans syndrome
Usually associated with uterine procedures like D&C
Premature ovarian failure
age < 40, karyotype, associated with elevated FSH
Ovarian dysfunction
radiation, relative insensitivity to LH

3. What additional studies are needed?


Prolactin
TSH
FSH
Estradiol may be helpful
4. Consider that this patient has a prolactin level of 130. The test, when repeated with the
patient fasting is 100. What is your next step?
Pituitary MRI
Treat with dopamine agonist like bromocriptine
Normal menses and fertility very likely to return, needs contraception
5. How would your next step differ if the patient had normal labs with an estradiol level of
30pcg/ml and an FSH of 2mIU/ml. What treatment would you offer her? What is she at
risk for?
Hypothalamic amenorrhea, most likely due to stress
Replace estrogen in order to protect against osteoporosis

References:
Obstetrics and Gynecology by Beckmann 5th Edition, 2006; Chapter 36 Amenorrhea and Dysfunctional
Uterine Bleeding. Pages 359-364.
Essentials of Obstetrics and Gynecology by Hacker and Moore 4th Edition, 2004; Chapter 33
Amenorrhea, Oligomenorrhea, and Hyperandrogenic Disorders. Pages 398-408.

CLINICAL CASE 43

Hirsutism and Virilization


A 25-year-old G0 woman presents with complaints of increased hair growth on the face,
particularly the upper lip, chin and neck. Her menses started about age 13 and she has
never had regular cycles. Her periods come every 2-4 months. She saw a physician
when she was 14 who stated that it was not uncommon to have irregular cycles initially
and she has not seen a physician since. She noted the onset of facial hair around the
onset of menses and it has gradually worsened. She has not noticed any recent increase
in severity; however, she is seriously dating someone and is interested in getting rid of
this hair growth. Currently, she has been using a combination of shaving twice a week
and electrolysis when she can afford it. Her electrologist recommended this evaluation.
She would like to be able to conceive in the near future and wonders about her fertility.
Allergy and medications:
None
Ob-Gyn history:
G0. LMP: 2 months ago. Menarche: 13 years old. Cycle length: every 2-4 months.
Duration: 7-10 days with heavy flow for the first 3 days. No STD history. She has been
sexually active and has used condoms for contraception. No prior pap smear.
Past medical and surgical history:
None
Social history:
She smokes pack of cigarettes per day for 8 years. Occasional alcohol. No street
drugs. She works as a bus driver.
Family history:
A number of family members are obese. Mother has irregular cycles, obesity, diabetes,
hypertension and similar facial hair growth. Her father had some balding, obesity,
hypertension and died of a heart attack at age 50. Her sister is obese, has irregular
cycles, similar facial hair growth and she has 2 children. Her 2 brothers are healthy,
except for being overweight.
VS: Ht: 53 Wt: 204 pounds, BP=120/80, Pulse=88, Respirations=18
Obese African-American woman in no apparent distress

Physical examination:
HEENT: NC/AT
Terminal hair is noted on the upper outer 1/3 of lip, few sparse hairs in the sideburn
area and chin, neck with hair recently shaved.
Neck: No thyromegaly palpable. Supple and no lymphadenopathy
Lungs: Clear to auscultation and percussion
CV: Nl S1 and S2. No murmurs
Chest: Few midline terminal hairs
Breasts: Few periareolar hairs. No palpable masses, discharge or nipple inversions.
Abdomen: NT, ND, obese, No hepatosplenomegaly and No striae. Terminal hair noted in
a vertical band below the umbilicus.
Extremities: Non-tender. No edema. DTRs: 2+/= bilaterally. Upper inner thighs with
sparse terminal hair.
Pelvic exam:
-Normal external genitalia
-Vagina: moist, pink with rugae
-Cervix: Nullipara, non-tender, no lesions
-Pap smear was done
-Uterus: anteflexed, anteverted, midline and normal size
-Adnexae: no palpable masses and non-tender
-Rectal: peri-anal tuft of hair noted
-Ferriman-Gallwey Score: 10 (Normal <8)
Laboratory or studies:
-Urine hCG: negative
-Hgb: 11.0 Hct: 33.6
-Total Testosterone: 84 ng/dl (Normal range: 20-80)
-DHEA-S: 247 uG/dl (Normal range: 62-615 uG/dl)
-Prolactin: 15 ng/dL (Normal range <20)
-TSH: 2.6 uIU/mL (Normal range: 0.4-4.0)
-17-hydroxyprogesterone: 120 ng/dl (Normal range < 200)
-Fasting glucose: 86 (Normal: 60-110)
-Pap smear: Normal
-Endometrial biopsy with Pipelle: Hyperplasia without atypia

Diagnoses:
Hirsutism
Chronic anovulation
Obesity
Hyperplasia without atypia
Management:
This patient was treated with oral contraceptive pills. She will return at the end of 6
months for re-evaluation and possible treatment with an anti-androgen such as
spironolactone to further control her hirsutism. She was also counseled regarding
weight loss and exercise. She was referred to a nutritionist.
Regarding fertility, she was reassured that with appropriate weight loss and after
stopping the birth control pill, she may begin ovulating on her own. However, if she
does not start to cycle, she would be an excellent candidate for a fertility medication,
such as clomiphene citrate. She would have about a 60% chance of ovulating on that
medication without the need for stronger medication. Metformin or other insulinsensitizing agent may also enhance her response to clomiphene citrate without
increasing her chance for multiple gestations.
Teaching points:
1.

This patient has the diagnosis of polycystic ovary syndrome (PCOS). This
diagnosis is made on the clinical triad of hirsutism and chronic anovulation
after ruling out other abnormalities. Other diagnoses that may present
similarly to PCOS include: hypothyroidism, late-onset congenital adrenal
hyperplasia and Cushings syndrome. The normal 17-hydroxyprogesterone
level ruled out congenital adrenal hyperplasia and the physical examination
ruled out Cushings syndrome. If there were any concern about the
possibility of Cushings syndrome, a dexamethasone suppression test or a 24hour urine free cortisol test would be appropriate. It is not necessary to
measure FSH and LH to look for a 1:3 ratio. With the newer immunocyte
assay systems, this ratio is more likely to be 1:1 or slightly increased in the
LH direction; it is much less likely to be present as a 1:3 ratio.

2.

Patients with PCOS who have been left untreated for prolonged periods of
time are at increased risk for endometrial cancer due to unopposed estrogen.
Case reports exist where PCOS patients are diagnosed with endometrial
cancer in the mid-twenties. It is important to evaluate these patients prior to
initiating hormonal therapy. An endometrial biopsy is warranted in these
patients despite being under 35-years-old.

3.

PCOS patients are also at increased risk for diabetes and deserve screening
with a fasting glucose level.

4.

When this patient desires to conceive, she should discontinue the oral
contraceptive and anti-androgen (if she is taking one). She may continue to
have regular menstrual cycles, particularly if she has lost weight and is now
within 10% of her ideal body weight. After a few months, if she is again
anovulatory, she will likely need a progestin to start her cycle and then she
should be treated with a fertility drug, such as clomiphene citrate, with or
without an insulin-sensitizing agent. Approximately 60% of anovulatory
women respond to clomiphene alone and some respond to combination
therapy. If there is no response, she may need a referral to a reproductive
endocrinologist for gonadotropin therapy and/or IVF.

5.

If this patient had rapidly progressing hirsutism accompanied by virilization,


one would need to rule out a tumor. Total testosterone levels greater than
200 ng/dl and/or DHEA-S greater than 700-uG/dl would be suggestive of a
tumor and would require further evaluation.

6.

Increasing the sex hormone binding globulin (so there is less free
testosterone), reducing androgen production, blocking androgen receptors,
or interfering with the 5-alpha reductase enzyme will improve hirsutism.
Medical treatment with birth control pills or antiandrogens are effective at
slowing down hair growth. However, these treatments will not remove
terminal hair that is already present. Physical measures are recommended
after the endocrine abnormality is corrected. These treatments include
depilatories, bleaching agents, shaving/plucking, waxing, electrolysis and
laser. The only two permanent procedures are electrolysis and laser therapy,
where the hair follicle is actually treated.

7.

Weight loss counseling is particularly warranted in patients with obesity.


There are a number of metabolic abnormalities associated with obesity, in
addition to increased morbidity and mortality.

TEACHING CASE 44
Normal and Abnormal Uterine Bleeding
Student Handout
A 45-year-old G2P0020, LMP 21 days ago, presents with heavy menstrual bleeding.
Prior to 6 months ago her cycles came every 28-30 days, lasted for 6 days, and were
associated with cramps that were relieved by ibuprofen. In the last 6 months there has
been a change with menses coming every 25-32 days, lasting 7-10 days and associated
with cramps not relieved by ibuprofen, passing clots and using 2 boxes of maxi pads
each cycle. She is worried about losing her job if the bleeding is not better controlled.
She denies dizziness, but complains of feeling weak and fatigued. Her weight has not
changed in the last year. She denies any bleeding disorders or reproductive cancers in
the family. She uses condoms for contraception. She takes no daily medications and has
no other medical problems. She is married and works in a factory. On exam,
BP=130/88; P= 100; Ht=56; Wt=150 pounds. She appears pale. Pelvic exam shows
normal vulva, vagina and cervix: normal size, mildly tender, mobile uterus; non-tender
adnexae without palpable masses. Labs show Hgb: 9.0, HCT: 27%, HCG: negative,
Endometrial biopsy: normal secretory endometrium, Pelvic ultrasound: heterogeneous
myometrium, endometrial lining 1.4cm and irregular consistent with endometrial
polyp, normal ovaries.
Discussion Questions:
1. What possible etiologies could cause this patients bleeding?
2. Which of these etiologies is associated with anovulation?
3. Discuss the mechanism for anovulatory bleeding.
4. How can you tell if this patient is having ovulatory cycles?
5. What further tests would indicate if there was an anatomic problem?
6. Describe 3 possible medical and 2 possible surgical therapies.

References:
Obstetrics and Gynecology by Beckmann 5th Edition, 2006; Chapter 36 Amenorrhea and Dysfunctional
Uterine Bleeding. Pages 359-364.
Essentials of Obstetrics and Gynecology by Hacker and Moore 4th Edition, 2004; Chapter 34
Dysfunctional Uterine Bleeding. Pages 409-412.

Normal and Abnormal Uterine Bleeding


Preceptor Handout

The occurrence of bleeding at times other than expected menses is a common event.
Accurate diagnosis of abnormal uterine bleeding is necessary for appropriate
management.
The APGO Educational Objectives related to this topic are the following:
A. Describe the normal menstrual cycle *
B. Define abnormal uterine bleeding *
C. Describe the etiologies of abnormal uterine bleeding*
D. Describe the evaluation methods of abnormal uterine bleeding*
E. Describe the therapeutic options for abnormal uterine bleeding
F. Counsel patients about each therapeutic option and sequelae

*Designated as Priority One in the APGO Medical Student Educational Objectives, 8th
Edition

Normal and Abnormal Uterine Bleeding


Preceptor Handout

A 45-year-old G2P0020, LMP 21 days ago, presents with heavy menstrual bleeding.
Prior to 6 months ago her cycles came every 28-30 days, lasted for 6 days, and were
associated with cramps that were relieved by ibuprofen. In the last 6 months there has
been a change with menses coming every 25-32 days, lasting 7-10 days and associated
with cramps not relieved by ibuprofen, passing clots and using 2 boxes of maxi pads
each cycle. She is worried about losing her job if the bleeding is not better controlled.
She denies dizziness, but complains of feeling weak and fatigued. Her weight has not
changed in the last year. She denies any bleeding disorders or reproductive cancers in
the family. She uses condoms for contraception. She takes no daily medications and has
no other medical problems. She is married and works in a factory. On exam,
BP=130/88; P= 100; Ht=56; Wt=150 pounds. She appears pale. Pelvic exam shows
normal vulva, vagina and cervix: normal size, mildly tender, mobile uterus; non-tender
adnexae without palpable masses. Labs show Hgb: 9.0, HCT: 27%, HCG: negative,
Endometrial biopsy: normal secretory endometrium, Pelvic Ultrasound: heterogeneous
myometrium, endometrial lining 1.4cm and irregular consistent with endometrial
polyp, normal ovaries.
Discussion Questions:
1. What possible etiologies could cause this patients bleeding?
Dysfunctional uterine bleeding
Thyroid disorders
Endometrial polyp
Submucous leiomyoma
Adenomyosis
Endometrial hyperplasia/ carcinoma
Bleeding dyscrasia
Endometritis
Discuss how these etiologies can be divided into ovulatory (anatomic) disorders and
anovulatory disorders
2. Which of these etiologies is associated with anovulation?
Endometrial hyperplasia
Endometrial carcinoma
Dysfunctional uterine bleeding
Thyroid disorder
Note that hyperplasia and cancer, although anatomic, are primarily associated with
anovulation, or unopposed estrogen.

3. Discuss the mechanism for anovulatory bleeding


Progesterone withdrawal signals the endometrium to shed in a uniform way by
causing spiral artery spasm. Women who dont ovulate dont have progesterone
withdrawal, and usually have bleeding due to unopposed estrogen with either
estrogen withdrawal or estrogen excess. Neither of these mechanisms causes
spiral artery spasm, and therefore can result in non-uniform shedding of the
lining at irregular intervals.
Discuss endometrial phases (proliferative and secretory), normal endocrinologic
events of the menstrual cycle, and the role of laboratory tests for progesterone in
diagnosing ovulation.
4. How can you tell if this patient is having ovulatory cycles?
History consistent with ovulatory cycles (regular, presence of molimina)
Timed (luteal phase) endometrial biopsy- current one is secretory, rules out
anovulation in this cycle
LH surge kits (ovulation prediction kits), basal body temperatures, day 21 serum
progesterone level
5. What further tests would indicate if there was an anatomic problem?
Sonohysterogram
Hysterosalpingogram
Pelvic MRI
Diagnostic hysteroscopy
6. Describe 3 possible medical and 2 possible surgical therapies.
Medical:
o Oral contraceptive pills
o GnRH agonist
o High dose NSAIDs
o Tranexamic acid
o Levonorgestrol IUD (Mirena)
Surgical:
o Endometerial ablation
o Hysteroscopic myomectomy or polypectomy
o Hysterectomy
Other discussion points:
o Estrogen/ progestin therapy is less effective in controlling bleeding
from an anatomic source and more effective for dysfunctional
bleeding
o Discuss how the desire for fertility would affect options
References:
Obstetrics and Gynecology by Beckmann 5th Edition, 2006; Chapter 36 Amenorrhea and Dysfunctional
Uterine Bleeding. Pages 359-364.
Essentials of Obstetrics and Gynecology by Hacker and Moore 4th Edition, 2004; Chapter 34
Dysfunctional Uterine Bleeding. Pages 409-412.

CLINICAL CASE 45

Dysmenorrhea
A 14-year-old female comes to your office complaining of severe dysmenorrhea (painful
periods) for the past six months. She began menstruating 10 months ago with her first
two periods occurring about 2 months apart without pain or any other symptoms. Since
then, she menstruates every 28 days and also notices nausea, diarrhea and headaches
during her periods. The pain has gotten so bad for 3 days each month that she often
misses school.
You speak to the patient without her mother and ask if she has ever been sexually active
in any way. The patient denies this activity and you believe her. She is a good student, is
involved in sports and after school programs, and you think it is unlikely that she is
pretending to have dysmenorrhea to get out of school. She denies use of drugs or
alcohol, and you believe it is unlikely she is drug seeking. She says that she gets partial
relief by using 3-4 Advil, two or three times a day during her period.
The review of systems, past medical history and social history are noncontributory. The
patients mother, but no other relatives, has endometriosis.
Physical exam:
The patients vital signs are normal and she is afebrile. Abdominal exam reveals no
masses or organomegaly, and no tenderness or rebound. Because the patient is virginal
and you do not want to induce undue pain, you defer the pelvic exam and do a rectal
exam showing a normal size non-tender uterus, which is mobile and anteflexed. There
are no nodules on the back of the uterus, and there are no adnexal masses or
tenderness.
Laboratory:
Urinalysis is negative for blood, nitrites and leukocytes.
Diagnosis:
High likelihood of primary dysmenorrhea based on the onset of pain and associated
systemic symptoms with regular cycles, and partial response to Advil.
Secondary dysmenorrhea due to endometriosis is possible, but less likely based on the
normal uterus and ovaries at the time of physical exam, and the short time this patient
has been menstruating. However, this is a familial condition and her mother has
endometriosis.

Management:
Regular use of non-steroidal anti-inflammatory agents starting just before or at the
beginning of menstruation are likely to give this patient relief.
If this is not sufficient or if she develops gastric upset with this treatment, combined
oral contraception are very likely to help this condition (whether it is primary
dysmenorrhea or secondary dysmenorrhea due to endometriosis. However, her mother
may be concerned about the use of oral contraceptives, worrying that it might increase
the likelihood of sexual activity. You can tell her there is data showing that medical use
of oral contraceptives is not associated with increased sexual activity.
If these two treatments are not successful, you plan on referring her to a gynecologist
for further workup.
Teaching points:
1. Dysmenorrhea is painful menstruation. This condition is quite common and is
estimated to contribute to recurrent disability in 10-15% of women in their
early reproductive years.
2. Dysmenorrhea can be divided into two types:
Primary dysmenorrhea: Onset is usually in the teens to twenties with the onset of
ovulation. Some studies indicate it is present in up to 90% of teenagers. Because it is
believed to be due to an excess of prostaglandin F2AlPHA (PGF2a)production in the
endometrium. This potent smooth-muscle stimulant causes intense uterine
contractions and resulting pain. It also gets into the systemic circulation resulting in (by
decreasing frequency) nausea, tiredness, nervousness, dizziness, diarrhea and
headache in up to 45% of patients. There are no abnormal physical findings in the
gynecological exam for primary dysmenorrhea. The diagnosis is one of exclusion.
Treatment of primary dysmenorrhea: Non-steroidal anti-inflammatory agents (NSAIDs)
and, if needed, combination oral contraceptives are extremely effective. NSAIDs are
prostaglandin-synthetase inhibitors, while oral contraceptives inhibit ovulation and
progesterone stimulation of prostaglandin production. Within three months of starting
combined oral contraceptives, 90% of women have a marked decrease in pain. Long
acting progesterone (depo-Provera) is a birth control method that is also effective in
dysmenorrhea. Failure of these regimens should lead to a reevaluation for a possible
secondary cause.
Secondary dysmenorrhea: are due to causes other than excess production of
prostaglandins. Causes can be divided into the following categories:

Extrauterine causes
Endometriosis (endometrial glands outside the uterus)

Tumors (benign or malignant) or cysts


Inflammation
Adhesions
Psychogenic (rare)
Non-gynecologic causes
Intramural causes
Adenomyosis (endometrial glands in the wall of the uterus)
Leiomyomata (fibroids/benign tumors in the wall of the uterus)
Intrauterine causes
Leiomyomata
Polyps
Intrauterine contraceptive devices (IUDs)
Infection
Cervical stenosis and cervical lesions

The diagnosis of secondary dysmenorrhea is more difficult than primary dysmenorrhea


because the symptoms and physical findings are variable. Symptoms often include
menorrhagia (heavy periods) and/or pain throughout the menstrual cycle. Abnormal
findings on abdominal or pelvic exam should raise suspicion of a secondary etiology or
pathology.
Treatment of secondary dysmenorrhea: depends on the etiology and often requires
imaging studies, office procedures, or surgery for diagnosis and treatment. A referral to
a gynecologist is often useful in this situation. One of the most common causes of
secondary dysmenorrhea is endometriosis, which has been found in up to 50% of
premenopausal women and 71-87% of women with chronic pelvic pain including
dysmenorrhea. This topic is discussed elsewhere in this series. Treatment for secondary
dysmenorrhea includes medical induction of menopause, laparoscopic surgery for
removal of endometriosis or adhesions, laparoscopic nerve ablation supplying to the
uterus, or hysterectomy.

TEACHING CASE 46

Menopause

Student Handout
Clinical Case:
A 53-year-old, G3P3, whose last menstrual period was 4 months ago presents to the
office with hot flashes, emotional lability, and insomnia. She experiences the hot flashes
2-3 times per day and occasionally at night. She has been having trouble sleeping and is
extremely fatigued. Since age 14, her periods have been regular until 2 years ago, when
they began to space out to every 2-3 months. She is sexually active and recently has
noted some dyspareunia. The patient works as a receptionist and rarely exercises. She
smokes 2 packs of cigarettes a day and drinks alcohol socially. She recently started
taking a soy supplement. She does not have any pertinent gynecological, medical or
surgical history. Her family history is significant for her mother sustaining a hip
fracture at age 60 and a sister with breast cancer and high cholesterol. On examination,
she has normal vital signs and she is 123 lbs and 54 tall. On pelvic examination, she
has mildly decreased vaginal rugae and a pale, small cervix. No masses or tenderness
are palpated on bimanual exam.
Discussion Questions:
1. What are the symptoms of menopause?
2. How do you make the diagnosis of menopause?
3. What are the patients risk factors for osteoporosis?
4. How do you diagnose and treat atrophic vaginitis?
5. How do you counsel a patient regarding estrogen and alternative therapies?
6. What laboratory and diagnostic tests would you order for this patient?

References:
Essentials of Obstetrics and Gynecology by Hacker and Moore, 4th Edition, Chapter 34.
Obstetrics and Gynecology by Beckmann, et al, 5th Edition, Chapter 38.

Menopause
Preceptor Handout

Women spend as much as one-third of their lives in the postmenopausal years. It is


important for physicians to understand the physical and emotional changes caused by
estrogen depletion.
The APGO Educational Objectives related to this topic are the following:
A. Describe physiologic changes in the hypothalamic-pituitary-ovarian axis associated
with perimenopause/menopause
B. Perform an assessment of the symptoms and physical findings associated with
hypoestrogenism *
C. Describe appropriate management of menopausal/ perimenopausal symptoms
D. Counsel patients regarding menopausal issues *
E. List long term changes associated with menopause, including osteoporosis

*Designated as Priority One in the APGO Medical Student Educational Objectives, 8 th


Edition

Menopause
Preceptor Handout
Clinical Case:
A 53-year-old, G3P3, whose last menstrual period was 4 months ago presents to the
office with hot flashes, emotional lability, and insomnia. She experiences the hot flashes
2-3 times per day and occasionally at night. She has been having trouble sleeping and is
extremely fatigued. Since age 14, her periods have been regular until 2 years ago, when
they began to space out to every 2-3 months. She is sexually active and recently has
noted some dyspareunia. The patient works as a receptionist and rarely exercises. She
smokes 2 packs of cigarettes a day and drinks alcohol socially. She recently started
taking a soy supplement. She does not have any pertinent gynecological, medical or
surgical history. Her family history is significant for her mother sustaining a hip
fracture at age 60 and a sister with breast cancer and high cholesterol. On examination,
she has normal vital signs and she is 113 lbs and 54 tall. On pelvic examination, she
has mildly decreased vaginal rugae and a pale, small cervix. No masses or tenderness
are palpated on bimanual exam.
Discussion Questions:
1. What are the symptoms of menopause?

Hypoestrogenism is the basis for the common changes of menopause.


Discuss the common signs and symptoms of menopause, such as amenorrhea,
hot flashes, loss of memory, fatigue, altered libido, dyspareunia, urinary
symptoms, breast changes, etc.

2. How do you make the diagnosis of menopause?

Menopause is the permanent cessation of menses and usually occurs between


the ages of 50 and 55, with an average of 50-52 years.
Perimenopausal symptoms usually begin 3 to 5 years before amenorrhea or
postmenopausal levels of hormones.

3. What are the patients risk factors for osteoporosis?

This patients risk factors include menopause, reduced weight for height, family
history of osteoporosis, cigarette smoking, and sedentary lifestyle.
Additional risk factors for discussion include age at menopause or
oophorectomy, white or Asian origin, calcium intake, parity, alcohol and caffeine
intake, and corticosteroid use.

4. How do you diagnose and treat atrophic vaginitis?

Patient usually has vulvar irritation and a moderate discharge (clear or yellow,
can be blood-tinged). Associated urinary symptoms may be present.
Examination shows a clear, watery discharge, with vulvar erythema. Excoriation
may be present. A pale vaginal mucosa, with patches of erythema and even
superficial blood vessels are consistent with atrophy.
The pale or yellow discharge has a pH of 5.5 or higher.
Basal cells replace superficial vaginal epithelial cells and can be seen on a saline
wet mount or Pap test.
Treatment is estrogen and takes 4 to 6 weeks for symptomatic relief.
Associated infections should be treated-consider a sulfa cream.
Discuss importance of evaluating any postmenopausal bleeding.

5. How do you counsel a patient regarding estrogen and alternative therapies?

Risks and benefits of therapy should be reviewed (WHI and other studies).
Contraindications should be discussed.
Treatment options for menopausal symptoms and osteoporosis should be
outlined.
Acknowledge frequent use of complementary and alternative treatments.
Lifestyle modifications should be stressed.

6. What laboratory and diagnostic tests would you order for this patient?

Laboratory and diagnostic tests should focus on the patients history and
symptoms, as well as preventive screening. For example, a TSH should be sent
due to her fatigue and a lipid profile due to the family history. A bone density is
indicated as well.
General health maintenance/screening tests should be ordered. These include a
mammogram, bone density, colonoscopy, etc.
Discuss the guidelines for ordering the above tests (i.e. colonoscopy at age 50,
bone density at age 65, etc.)
Consider discussing new Pap test recommendations.

References:
Obstetrics and Gynecology by Beckmann 5th Edition, 2006; Chapter 38 Menopause. Pages 374-383.
Essentials of Obstetrics and Gynecology by Hacker and Moore 4th Edition, 2004; Chapter 36
Climacteric. Pages 422-428.

TEACHING CASE 47

Infertility
Student Handout
Clinical Case:
A 37-year-old female and her 37-year-old husband present with the complaint of a
possible fertility problem. The couple has been married for 2 years. The patient has a 4year-old daughter from a previous relationship. The patient used birth control pills
until one-and-a- half-years-ago. The couple has been trying to conceive since then and
report a high degree of stress related to their lack of success. The patient reports good
health and no problems in conceiving her previous pregnancy or in the vaginal delivery
of her daughter. She reports that her periods were regular on the birth control pill, but
have been irregular since she discontinued taking them. She reports having periods
every 5-7 weeks. Past history is remarkable only for mild depression. Imipramine 150
mg qhs for the last 8 months is her only medication. She works as a cashier, runs 12-24
miles each week for the last 2 years, and has no history of STDs, abnormal Paps,
smoking, alcohol or other drugs. She has had no surgery.
The patients partner also reports good health and reports no problems with erection,
ejaculation or pain with intercourse. He has had no prior urogenital infections or
exposure to STDs. He has had unprotected sex prior to his current relationship, but has
not knowingly conceived. He has no medical problems or past surgery. He works as a
long-distance truck driver and is on the road 2-3 weeks each month. He smokes a pack
of cigarettes a day since age 18 and drinks 2-3 cans of beer 3-4 times a week when hes
not driving. He occasionally uses amphetamines to stay awake while driving at night.
The couple has vaginal intercourse 3-5 times per week when he is at home.
The patient is 59 and weighs 130 pounds. Breast exam reveals no tenderness or
masses, but bilateral galactorrhea on compression of the areola. Pelvic exam reveals
normal genitalia, a well-estrogenized vaginal vault mucosa and cervical mucus
consistent with the proliferative phase. The uterus is anteflexed and normal in size
without masses or tenderness. Several tests were ordered.
Discussion Questions:
1. What is the definition of infertility?
2. What are the etiologies of infertility?
3. What is the initial work-up for infertile couples and what tests would you add for
this particular couple?

4. Studies were reviewed and showed a normal TSH, a prolactin of 60 ng/ml (normal
range < 20 ng/ml) and a semen analysis with 2cc of semen, 4 million sperm per ml,
30% normal forms and 20% motility. Basal body temperature chart shows a
monophasic temperature graph. What is the differential diagnosis at this point?
5. What is the appropriate management for both these diagnoses?

References:
Obstetrics and Gynecology by Beckmann 5th Edition, 2006; Chapter 39 Infertility. Pages 384-395.
Essentials of Obstetrics and Gynecology by Hacker and Moore 4th Edition, 2004; Chapter 35 Infertility
and Assisted Reproductive Technologies. Pages 413-421.

Infertility
Preceptor Handout

The evaluation and management of an infertile couple requires an understanding of the


processes of conception and embryogenesis, as well as sensitivity to the emotional stress
that can result from the inability to conceive.
The APGO Educational Objectives related to this topic are the following:
A. Define infertility *
B. Describe the causes of male and female infertility *
C. Describe the evaluation and management of infertility
D. List the psychosocial issues associated with infertility

*Designated as Priority One in the APGO Medical Student Educational Objectives, 8 th


Edition

Infertility
Preceptor Handout
Clinical Case:
A 37-year-old female and her 37-year-old husband present with the complaint of a
possible fertility problem. The couple has been married for 2 years. The patient has a 4year-old daughter from a previous relationship. The patient used birth control pills
until one-and-a- half-years-ago. The couple has been trying to conceive since then and
report a high degree of stress related to their lack of success. The patient reports good
health and no problems in conceiving her previous pregnancy or in the vaginal delivery
of her daughter. She reports that her periods were regular on the birth control pill, but
have been irregular since she discontinued taking them. She reports having periods
every 5-7 weeks. Past history is remarkable only for mild depression. Imipramine 150
mg qhs for the last 8 months is her only medication. She works as a cashier, runs 12-24
miles each week for the last 2 years, and has no history of STDs, abnormal Paps,
smoking, alcohol or other drugs. She has had no surgery.
The patients partner also reports good health and reports no problems with erection,
ejaculation or pain with intercourse. He has had no prior urogenital infections or
exposure to STDs. He has had unprotected sex prior to his current relationship, but has
not knowingly conceived. He has no medical problems or past surgery. He works as a
long-distance truck driver and is on the road 2-3 weeks each month. He smokes a pack
of cigarettes a day since age 18 and drinks 2-3 cans of beer 3-4 times a week when hes
not driving. He occasionally uses amphetamines to stay awake while driving at night.
The couple has vaginal intercourse 3-5 times per week when he is at home.
The patient is 59 and weighs 130 pounds. Breast exam reveals no tenderness or
masses, but bilateral galactorrhea on compression of the areola. Pelvic exam reveals
normal genitalia, a well-estrogenized vaginal vault mucosa and cervical mucus
consistent with the proliferative phase. The uterus is anteflexed and normal in size
without masses or tenderness. Several tests were orderd.
Discussion Questions:
1. What is the definition of infertility?

Inability to become pregnant despite 12 months of trying to conceive


without using contraception. About 15% of couples experience this
problem.

2. What are the etiologies of infertility?

Ovulatory dysfunction (around 20%) - anovulation


Male factor (around 30%) decreased sperm count, decreased motility or
normal forms

Tubal and pelvic (30%) tubal damage due to pelvic infection, pelvic
factor - endometriosis, pelvic adhesions
Unexplained (10%)
Unusual problems (10%)

3. What is the initial work-up for infertile couples and what tests would you add for
this particular couple?
Physical examination
Medical history
Semen analysis
Check for ovulation basal body temperature, ovulation prediction kits,
luteal phase serum progesterone
Hysterosalpingogram: check for normal uterus and open fallopian tubes
Discussion about frequency and timing of intercourse
For this couple specifically, TSH and prolactin
4. Studies were reviewed and showed a normal TSH, a prolactin of 60 ng/ml
(normal range < 20 ng/ml) and a semen analysis with 2cc of semen, 4 million
sperm per ml, 30% normal forms and 20% motility. Basal body temperature
chart shows a monophasic temperature graph. What is the differential diagnosis
at this point?

Anovulation secondary to hyperprolactinemia from imipramine


Oligospermia repeat semen analysis once or twice to confirm the
diagnosis

5. What is the appropriate management for both these diagnoses?

For anovulation secondary to imipramine, the patient should taper her


imipramine. If she was still anovulatory, the patient could empirically use
clomiphene citrate for ovulation induction
For oligospermia, options include in vitro fertilization, artificial
insemination with partners sperm, artificial insemination with donor
sperm, adoption.

References:
Obstetrics and Gynecology by Beckmann 5th Edition, 2006; Chapter 39 Infertility. Pages 384-395.
Essentials of Obstetrics and Gynecology by Hacker and Moore 4th Edition, 2004; Chapter 35 Infertility
and Assisted Reproductive Technologies. Pages 413-421.

CLINICAL CASE 48
Premenstrual Syndrome (Premenstrual Dysphoria Disorder)
GS, a 37-year-old married woman, comes to your office for an annual checkup. She has
recently moved to town, and all her previous medical care was in a different city. She
has not seen a gynecologist for 2 years and states that she wants to establish a
relationship with a physician in her new surroundings.
The patient is a gravida 3, para 3. She has regular periods, although they have gotten
somewhat longer in the past year or so. She is currently not sexually active and is taking
no medications or supplements.
Past history reveals that she underwent an appendectomy as a child and has had two
diagnostic laparoscopies for pelvic pain, with the most recent done 3 years ago. She
has no pain at the present time, has no medical conditions and is not allergic to any
medications.
Her family history reveals that her mother suffered from depression. Her 40-year-old
sister was recently diagnosed with breast cancer. Upon review of systems, she
describes occasional constipation and diarrhea. She has recently had difficulty sleeping
and feels that she gets tired more easily than she should. Upon further questioning, she
reveals that she has difficulty falling asleep, often because she is thinking about what
has happened during the day and/or what may be coming up the next day. The patient
and her three children have recently moved to town, while her husband has remained
in their previous city to fulfill his job obligation. This domestic separation has been
going on for approximately 6 months.
On physical examination, all findings are normal. The patient did appear to be a bit
nervous and startled easily as you entered the room.
On further questioning, the patient thinks that her jitteriness and sleeplessness have led
to increased irritability with the children. She worries a great deal, particularly about
her domestic situation and being separated from her husband. She has difficulty
concentrating at her job (she works as a bank teller) and also feels that her memory is
failing her, as she loses her keys or misplaces items at home from time to time. Further
questioning also reveals that the patient has observed no pattern indicating that the
symptoms occur only during the luteal phase. You also note that at the time of the
examination, when she presents with nervousness, GS is in the follicular phase of her
cycle.
She saw a physician assistant in a primary care practice regarding these symptoms. He
told her that he believes she has PMS. The patient does believe that her symptoms may
get worse at different times of the month, but she has never been able to keep track of
them long enough to know whether there is a specific cyclic pattern to these problems.
General lab tests were performed and were normal. Under the assumption that it is
PMS, he recommended a series of treatments, all of which have been unsuccessful, i.e.
birth control pills, progesterone suppositories, vitamin B6 supplementation, diuretics
and nonsteroidal anti-inflammatory drugs, specifically Ibuprofen and Naproxen
Sodium. She has taken all of these medications and has also tried to get more exercise

and eat right. She believes that the combination of being separated from her husband,
moving to a new town, and the stress of doing her job accurately has overwhelmed her.
She does not understand why the PMS has not improved and asks whether a
hysterectomy might be the solution.
Treatment:
Because the physical examination, thyroid function tests, electrolytes, liver function test
and a complete blood count are normal, you are confident that the patient does not
have any underlying medical conditions. You suggest to the patient that she may have
an anxiety disorder, perhaps generalized anxiety disorder. You initially start her on
Alprazolam, 0.25 mg, three times a day and suggest that she monitor her symptoms and
return in one week.
The patient returns in one week and reports significant improvement in her sleep
patterns, as well as her mental functioning. She feels much calmer. You reassure the
patient that there is no underlying medical problem and that she is not going crazy,
but appears only to have an anxiety disorder that can be treated successfully. You
explain to her that life stressors can exacerbate her underlying anxiety disorder.
You also recommend that she avoid caffeine and alcohol. Although she feels better, the
patient wishes to discontinue the medication to see if her lifestyle changes might make
a difference.
She returns 1 month later, and her symptoms have returned. You then initiate therapy
with Buspirone, 10 mg, three times a day, and explain to her that it will take 2 to 3
weeks for this medication to take effect. You also explain that it does not have any
sedating qualities and will not be habit forming. The patient returns 3 months later, at
which time she is functioning well and is quite comfortable with the current dosage of
Buspirone.
Teaching points:
1.

Differentiating PMS from anxiety may depend on prospective


documentation of symptoms. Without the documentation or with a
history that is unclear, making a firm diagnosis of PMS/PMDD may be
difficult. Alternatively, if symptoms are compatible with anxiety, this
should be a primary consideration.

2.

An empathetic, sensitive approach to the patients concerns is needed.


Understanding the environment in which these patients find themselves
is often helpful in making the diagnosis.

3.

May women who believe they have PMS actually have a different
condition. To some patients, PMS is a more acceptable diagnosis. This is
certainly more commonly seen in an ob-gyn office than in the office of a
mental health care professional.

4.

Initial management with a benzodiazepine will provide an earlier


response potential. The use of buspirone, with its benefits of not being
sedating and not being habit forming, might be useful for long-term
management.

UNIT FIVE:

NEOPLASIA

49.
50.
51.
52.
53.
54.

Gestational Trophoblastic Neoplasia


Vulvar Neoplasms
Cervical Disease and Neoplasia
Uterine Leiomyomas
Endometrial Carcinoma
Ovarian Neoplasms

TEACHING CASE 49
Gestational Trophoblastic Neoplasia (GTD)
Student Handout
Clinical Case:
A 15-year-old primigravida presents for routine prenatal care. She is 14 weeks
pregnant by last menstrual period. She has some nausea but otherwise feels well. The
pregnancy to date has been unremarkable. She has support from her parents and the
father of the baby.
The uterus is enlarged, measuring 27 cm from the pubic symphysis. Fetal heart tones
are not auscultated by Doppler. She denies vaginal bleeding or passage of tissue from
the vagina. Vaginal exam is unremarkable.
Routine prenatal labs were unremarkable. She is Rh-positive. Quantitative beta HCG
levels were markedly elevated at 112,320 MICU/ml. TSH was low and further thyroid
testing revealed the patient to be mildly hyperthyroid.
Ultrasound showed the uterus to be enlarged, with multiple internal echoes and a
snow storm appearance. No fetus is noted. Ultrasound also showed enlarged multiloculated ovarian cysts bilaterally. Chest x-ray was normal.
Discussion Questions:
1. What is the differential diagnosis?
2. What additional evaluation is needed to confirm the diagnosis?
3. What is the final diagnosis?
4. What is the epidemiology and clinical course of this condition?
5. What is your management plan?

References:
Obstetrics and Gynecology by Beckmann 5th Edition, 2006; Chapter 42 Gestational Trophoblastic
Neoplasia. Pages 411-417.
Essentials of Obstetrics and Gynecology by Hacker and Moore 4th Edition, 2004; Chapter 43
Gestational Trophoblastic Neoplasia. Pages 486-494.

Gestational Trophoblastic Neoplasia (GTD)


Preceptor Handout

Gestational trophoblastic neoplasia is important because of its malignant potential and


the associated risks of morbidity and mortality.

The APGO Educational Objectives related to this topic are the following:
A. List the symptoms and physical findings of a patient with GTD
B. Describe the diagnostic methods utilized for a patient with GTD

*Designated as Priority One in the APGO Medical Student Educational Objectives, 8th
Edition

Gestational Trophoblastic Neoplasia (GTD)


Preceptor Handout

Clinical Case:
A 15-year-old primigravida presents for routine prenatal care. She is 14 weeks
pregnant by last menstrual period. She has some nausea but otherwise feels well. The
pregnancy to date has been unremarkable. She does have support from her parents and
the father of the baby.
The uterus is enlarged, measuring 27 cm from the pubic symphysis. Fetal heart tones
are not auscultated by Doppler. She denies vaginal bleeding or passage of tissue from
the vagina. Vaginal exam is unremarkable.
Routine prenatal labs were unremarkable. She is Rh-positive. Quantitative beta HCG
levels were markedly elevated at 112,320 MICU/ml. TSH was low and further thyroid
testing revealed the patient to be mildly hyperthyroid.
Ultrasound showed the uterus to be enlarged, with multiple internal echoes and a
snow storm appearance. No fetus is noted. Ultrasound also showed enlarged multiloculated ovarian cysts bilaterally. Chest x-ray was normal.
Discussion Questions:
1. What is the differential diagnosis?
Poor dates, most likely if irregular menses
Multiple gestation
Molar pregnancy (complete or incomplete)
2. What additional evaluation is needed to confirm the diagnosis?
Ultrasound is very helpful to assess for presence of one or more fetuses, and
to confirm gestational age.
Ultrasound can also reveal any associated ovarian enlargement.
3. What is the final diagnosis?
Hydatidiform mole with bilateral theca lutein cysts
4. What is the epidemiology and clinical course of this condition?
Gestational trophoblastic neoplasia is the most curable gynecological
malignancy.

Although patients with hydatidiform mole are classically described as having


uteruses that are large for dates, this only occurs in approximately half of the
patients.
Molar pregnancies are more likely to occur in women 15-years-old or less, or
40-years-old and greater.
Gestational trophoblastic neoplasia is frequently associated with
hyperthyroidism due to the release of a thyrotropin-like compound by the
molar tissue.
Patients with molar pregnancy have increased risk of trophoblastic disease
in later pregnancies (recurrence rate is 1%) and should have early
ultrasound in every subsequent pregnancy.

5. What is your management plan?


Primary treatment is suction evacuation of the uterus.
Patients with gestational trophoblastic neoplasia should not attempt
subsequent pregnancy until after six months of negative beta HCG levels,
thus reliable contraception use needs to be discussed and implemented.
If beta HCG does not rapidly decrease, methotrexate can be used as
secondary treatment.
Thyroid function should also be followed until normalized.
Chest x-ray, pelvic examination and ultrasound should be followed to rule
out choriocarcinoma and to document the resolution of the ovarian cysts.

References:
Obstetrics and Gynecology by Beckmann 5th Edition, 2006; Chapter 42 Gestational Trophoblastic
Neoplasia. Pages 411-417.
Essentials of Obstetrics and Gynecology by Hacker and Moore 4th Edition, 2004; Chapter 43
Gestational Trophoblastic Neoplasia. Pages 486-494.

CLINICAL CASE 50
Vulvar Neoplasms
A 67-year-old woman presents with the complaint of a pruritic area on the right side of
her vulva. She has noticed this for about three months, and has used a variety of overthe-counter creams, including imidazole and corticosteroid preparations, without
success.
Gynecological history:
She underwent an uneventful menopause at age 52. She tried hormone replacement
therapy for three years, which she finally discontinued because of irregular bleeding.
The bleeding stopped when she discontinued therapy. She has no other gynecological
complaints. She does have a history of abnormal Paps, including a conization at age 35.
Her last Pap was approximately 7 years ago.
The patient has a long-standing history of hypertension. She is presently taking a betablocker and states that her hypertension is well controlled. She has no other medical
complaints.
Physical exam:
Physical examination is unremarkable. Pressure is 134/82. Examination of the vulva
reveals a 7 mm raised, firm, irregular, lesion on the right labia majora. Excoriations are
also noted. The rest of the vulva is normal. The vagina is atrophic. The cervix is grossly
normal. The uterus is small and the ovaries are nonpalpable.
Laboratory:
Vulvar biopsy was performed, with removal of the entire lesion. Results were squamous
cell carcinoma of the vulva. Stromal invasion was less than 1 mm. Margins were clear of
tumor.
Management:
This is a Stage I lesion with a central location. Wide local excision with close
observation is appropriate management. For more advanced lesions, radical excision
with inguinal node dissection is utilized. This surgery has significant morbidity. The
surgical lymphadenectomy is bilateral with central lesions and ipsilateral for lateral
lesions. Adjuvant radiation therapy is frequently utilized.

Teaching points:
All vulvar lesions without a clear diagnosis must be biopsied.
Squamous cell carcinoma typically occurs in menopausal women, pruritus is the most
common chief complaint. The most common type (86%) is squamous cell cancer as
with most cancers of the lower genital tract, is likely associated with oncogenic strains
of Human Papilloma Virus (HPV).
The malignancy behaves differently from the typical squamous cell carcinomas, which
occur on the face and torso and are associated with overexposure to sunlight. Inguinal
node metastases is common with more advanced lesions. Radical vulvar surgery and
inguinal lymphadenopathy carries high morbidity.

TEACHING CASE 51
Cervical Disease and Neoplasia
Student Handout
Clinical Case:
Ms Stevenson is a generally healthy 26 year-old female, G1P0010 with a last menstrual
period approximately 16 weeks ago referred for the management of an abnormal Pap
smear which showed High Grade Squamous Intraepithelial Lesion (HGSIL). This Pap
smear was obtained 10 weeks ago when she underwent an elective termination of an
unplanned pregnancy at approximately six weeks of gestation. She has not had any
prior Pap smears. She has never been tested for sexually transmitted infections. The
combination of the undesired pregnancy and the abnormal Pap smear, however, have
been a wake-up call and today she requests testing for everything. She received
Depo-provera at the time of the termination, and has not had a period yet. She smokes
pack of cigarettes, does clerical work for a moving company, and is engaged to be
married in 6 months.
Discussion Questions:
1. According to recent guidelines published by the American College of Obstetricians
and Gynecologists, how many Pap smears should this patient have had given her age
and clinical history?
2. Which historical risk factors does this patient have for having cervical dysplasia or
for having cervical dysplasia progress to cervical cancer?
3. What is meant by the term "high-grade squamous intraepithelial lesion"?
4. What would you recommend as the next step in the evaluation of this patient's
abnormal pap smear?
5. Would typing for the human papilloma virus aid in the management of this patient?
6. From a reproductive health perspective, how would you counsel this particular
patient about smoking cessation?

References:
Obstetrics and Gynecology by Beckmann 5th Edition, 2006; Chapter 44 Cervical Neoplasia and
Carcinoma. Pages 430-447.
Essentials of Obstetrics and Gynecology by Hacker and Moore 4th Edition, 2004; Chapter 39 Cervical
Dysplasia and Cancer. Pages 447-458

Cervical Disease and Neoplasia


Preceptor Handout

Detection and treatment of pre-invasive lesions reduces the medical and social costs of, as
well as the mortality associated with, carcinoma of the cervix.
The APGO Educational Objectives related to this topic are the following:
A. Describe the pathogenesis of cervical cancer
B. Identify the risk factors for cervical neoplasia and cancer *
C. Perform an adequate Pap smear *
D. Describe the appropriate utilization of new technologies for evaluating cervical
neoplasia
E. Describe the initial management of a patient with an abnormal Pap smear *

*Designated as Priority One in the APGO Medical Student Educational Objectives, 8th
Edition

Cervical Disease and Neoplasia


Preceptor Handout
Clinical Case:
Ms Stevenson is a generally healthy 26 year-old female, G1P0010 with a last menstrual
period approximately 16 weeks ago referred for the management of an abnormal Pap
smear which showed High Grade Squamous Intraepithelial Lesion (HGSIL). This Pap
smear was obtained 10 weeks ago when she underwent an elective termination of an
unplanned pregnancy at approximately six weeks of gestation. She has not had any
prior Pap smears. She has never been tested for sexually transmitted infections. The
combination of the undesired pregnancy and the abnormal Pap smear, however, have
been a wake-up call and today she requests testing for everything. She received
Depo-provera at the time of the termination, and has not had a period yet. She smokes
pack of cigarettes, does clerical work for a moving company, and is engaged to be
married in 6 months.
Discussion Questions:
1. According to recent guidelines published by the American College of Obstetricians and
Gynecologists, how many Pap smears should this patient have had given her age and
clinical history?

Answer requires knowing the guidelines


First cytology 3 years after coitarche (age 19 for this pt) or at age 21, whichever
occurs first, then annually through age 30.
So, she should have had 7 or 8 evaluations by now.

2. Which historical risk factors does this patient have for having cervical dysplasia or for
having cervical dysplasia progress to cervical cancer?

Answer requires knowledge of risk factors


She has poor compliance with screening, early age of coitarche (< 19), cigarette
smoking
Abnormal Pap is presumptive evidence of HPV infection
She may have HIV
She probably does not have an autoimmune disease
She does not meet research definitions for "multiple lifetime sexual partners", >
5. Yet.

3. What is meant by the term "high-grade squamous intraepithelial lesion"?

Answer requires knowledge of possible Pap smear readings: normal, LGSIL,


HGSIL, suspicious for invasive cancer. Atypical squamous cell readings can also
be discussed as can the universe of abnormal glandular cell readings.

Answer also requires students to understand the hypothetical correlation


between cytology and histology (biopsy).
Cells were identified suggesting abnormal cellular maturation between 1/3 and
full thickness of the squamous epithelial layer of the cervix.
Consider constructing the cartoon of the 8 12 cell layer thick squamous
epithelial layer.

4. What would you recommend as the next step in the evaluation of this patient's
abnormal pap test?

Answer requires knowledge of abnormal Pap smear management.


Colposcopy with directed biopsy and ECC.
Students could also discuss HIV testing and smoking cessation

5. Would typing for the human papilloma virus aid in the management of this patient?

Answer requires knowledge of the role of HPV typing


In this case, HSIL, the answer is no as most patients are positive, so it is not
useful as a triage tool.
Low risk HPV types include 6, 8, and 11, correlate with cervical warts
High risk HPV types include 16 and 18, correlate with cervical dysplasia
HPV typing plays a role in the management of Atypical Squamous Cell (ASC)
readings as only about 50% of these patients are actually infectyed with a high
risk viral type

6. From a reproductive health perspective, how would you counsel this particular patient
about smoking cessation?

Answer requires specific knowledge about cigarettes and reproductive health


Obviously, quit, quit, quit
Discussion topics include the abnormal pap test, her future contraception
options
Also irregular periods, earlier menopause, infertility, miscarriage, IUGR, PROM,
PTL, SIDS

References:
Obstetrics and Gynecology by Beckmann 5th Edition, 2006; Chapter 44 Cervical Neoplasia and
Carcinoma. Pages 430-447.
Essentials of Obstetrics and Gynecology by Hacker and Moore 4th Edition, 2004; Chapter 39 Cervical
Dysplasia and Cancer. Pages 447-458

TEACHING CASE 52

Uterine Leiomyoma
Student Handout
Clinical Case:
A 42-year-old G3 P3 female presents with a history of abnormal bleeding and pelvic
pain. She was well until approximately age 35, when she began developing
dysmenorrhea and progressive menorrhagia. The dysmenorrhea was not fully relieved
by NSAIDS. Over the next several years, the dysmenorrhea and menorrhagia became
more severe. She then developed intermenstrual bleeding and spotting, as well as pelvic
pain, which she describes as a constant feeling of pressure. She also complains of
urinary frequency. Past gynecological history is otherwise non-contributory. She
delivered three children by caesarean section, the last with a tubal ligation at age 30.
Her past medical history is unremarkable.
Physical Examination reveals a well-developed, well-nourished woman in no distress.
Vital signs and general physical exam are unremarkable. Abdominal examination
reveals an irregular-sized mass into extending halfway between the pubic symphysis
and umbilicus and to the right of the midline. Pelvic exam reveals a normal appearing
vagina and cervix. The uterus is markedly enlarged and irregular, especially on the right
side where it appears to reach the lateral pelvic sidewalls. The examiner is unable to
palpate normal ovaries due to the mass.
Beta HCG is negative. CBC reveals hemoglobin of 10.3 and hematocrit of 31.2. Indices
are hypochromic, microcytic. Serum ferritin confirms mild iron deficiency anemia. Pap
smear is reported negative for malignancy, adequate for evaluation. Ultrasound shows a
large irregular mass, filling the pelvis and extending into the lower abdomen. The mass
does extend into the right side of the pelvis. There is mild hydronephrosis on that side.
The ovaries are not visualized. Endometrial biopsy reveals proliferative endometrium.
Discussion questions:
1.
2.
3.
4.

What are the likely causes of the mass?


Describe the pathological changes?
Discuss the appropriate management of women with fibroids?
What are the indications for hysterectomy in women with fibroids?

References:
Obstetrics and Gynecology by Beckmann 5th Edition, 2006; Chapter 45 Uterine Leiomyoma and
Neoplasia. Pages 448-454.
ACOG Practice Bulletin 16. Surgical Alternatives to Hysterectomy in the Management of Leiomyomas May
2000.

Uterine Leiomyoma
Preceptor Handout

Uterine leiomyomas represent the most common gynecologic neoplasm and are often
asymptomatic.
The APGO Educational Objectives related to this topic are the following:
A. Discuss the prevalence of uterine leiomyomas
B. Describe the symptoms and physical findings in patients with uterine
leiomyomas*
C. Apply diagnostic methods to confirm uterine leiomyomas
D. List the indications for medical and surgical treatment of uterine leiomyomas

*Designated as Priority One in the APGO Medical Student Educational Objectives, 8 th


Edition

Uterine Leiomyoma
Preceptor Handout
Clinical Case:
A 42-year-old G3 P3 female presents with a history of abnormal bleeding and pelvic
pain. She was well until approximately age 35, when she began developing
dysmenorrhea and progressive menorrhagia. The dysmenorrhea was not fully relieved
by NSAIDS. Over the next several years, the dysmenorrhea and menorrhagia became
more severe. She then developed intermenstrual bleeding and spotting, as well as pelvic
pain, which she describes as a constant feeling of pressure. She also complains of
urinary frequency. Past gynecological history is otherwise non-contributory. She
delivered three children by caesarean section, the last with a tubal ligation at age 30.
Her past medical history is unremarkable.
Physical Examination reveals a well-developed, well-nourished woman in no distress.
Vital signs and general physical exam are unremarkable. Abdominal examination
reveals an irregular-sized mass into extending halfway between the pubic symphysis
and umbilicus and to the right of the midline. Pelvic exam reveals a normal appearing
vagina and cervix. The uterus is markedly enlarged and irregular, especially on the right
side where it appears to reach the lateral pelvic sidewalls. The examiner is unable to
palpate normal ovaries due to the mass.
Beta HCG is negative. CBC reveals hemoglobin of 10.3 and hematocrit of 31.2. Indices
are hypochromic, microcytic. Serum ferritin confirms mild iron deficiency anemia. Pap
smear is reported negative for malignancy, adequate for evaluation. Ultrasound shows a
large irregular mass, filling the pelvis and extending into the lower abdomen. The mass
does extend into the right side of the pelvis. There is mild hydronephrosis on that side.
The ovaries are not visualized. Endometrial biopsy reveals proliferative endometrium.
Discussion questions:
1. What are the likely causes of the mass?
The most common cause of a large irregular uterine mass is uterine
leiomata. The clinical picture is typical of such a patient. The physician
must be sure that the patient does not have ovarian neoplasia. Usually
this is accomplished when the ultrasound confirms the diagnosis of
fibroids.
2. Describe the pathological changes?
Bundles of unstriated muscle. A leiomyosarcoma will have at least 10
abnormal mitoses per high power field.
Fibroids are common, leiomyosarcoma is very rare

3. Discuss the appropriate management of women with fibroids?


Expectant therapy is acceptable if intervention not warranted by
symptoms. No intervention is needed for women with asymptomatic
fibroids. Many women with fibroids are asymptomatic.
o Fibroids can be subserosal, intramural or submucosal. Submucosal
fibroids are frequently associated with bleeding.
o The most frequent symptoms of uterine fibroids are pain, bleeding
and pressure symptoms.
Pregnancies in women with fibroid are usually uneventful
Fibroids are rarely a cause of infertility. There are specific criteria for the
use of myomectomy in infertile patients.
Progestin therapy for bleeding. OCs may be utilized.
GNRH agonists are only utilized preoperatively
LNG-IUS can be effective for pain and bleeding in uteri up to 10 weeks
size
Hysteroscopic resection
Uterine ablation
Uterine artery embolization
Myomectomy
Hysterectomy
4. What are the indications for hysterectomy in women with fibroids?
Palpable abdominally and a concern to the patient
Excessive uterine bleeding
o Profuse bleeding, flooding, clots or periods > 8 days
o Anemia due to the blood loss
Pelvic pain caused by the fibroids
o Acute and severe
o Chronic lower abdominal or back pressure
o Bladder pressure causing urinary frequency

References:
Obstetrics and Gynecology by Beckmann 5th Edition, 2006; Chapter 45 Uterine Leiomyoma and
Neoplasia. Pages 448-454.
ACOG Practice Bulletin 16. Surgical Alternatives to Hysterectomy in the Management of Leiomyomas May
2000.

TEACHING CASE 53
Endometrial carcinoma
Student Handout
Clinical Case:
A 56 year-old G0 presents to the clinic with complaints of intermittent vaginal bleeding.
She went through the menopause 2 years ago and had no vaginal bleeding until 6
months ago when she had a three day episode of light bleeding. Since that time, she has
had another 3 such episodes. Past medical history is remarkable for well-controlled
hypertension, depression and borderline diabetes for which the patient is on a diet to
which she is minimally adherent. She never used contraceptives but was unable to
become pregnant. She has had a laparoscopic choleycystectomy. She takes Lisinopril
and Zoloft. Her family history is non-contributory. On examination, she has normal
vital signs, she weighs 247 pounds. Her heart, lungs and abdominal exams are normal.
On pelvic examination, she has normal external genitalia, normal vagina and cervix.
The bimanual exam is difficult secondary to the patients habitus, but the uterus feels
slightly enlarged and no adnexal masses are palpable.
Discussion Questions:
1. What is your differential diagnosis for this patient?
2. What is the etiology of endometrial cancer?
3. What risk factors does this patient have for endometrial cancer?
4. What factors are protective against endometrial cancer?
5. What are the next steps in the diagnostic work-up of this patient?
6. You obtain an ultrasound which shows a 10 cm uterus and no adnexal masses.
Endometrial biopsy reveals complex hyperplasia with atypia. Are further
diagnostic tests indicated?
7. How would you treat the patient?

References:
Obstetrics and Gynecology by Beckmann 5th Edition, 2006; Chapter 46 Endometrial Hyperplasia and
Cancer. Pages 455-463.
Essentials of Obstetrics and Gynecology by Hacker and Moore 4th Edition, 2004; Chapter 42 Uterine
Corpus Cancer. Pages 478-485.

Endometrial carcinoma
Preceptor Handout

Endometrial cancer is the most common malignancy.


The APGO Educational Objectives related to this topic are the following:
A. List the risk factors for endometrial carcinoma *
B. Describe the symptoms and physical findings of a patient with endometrial
cancer*
C. Outline the appropriate management of the patient with postmenopausal
bleeding *
D. Discuss the use of the diagnostic methods for a patient with endometrial
carcinoma

*Designated as Priority One in the APGO Medical Student Educational Objectives, 8 th


Edition

Ovarian Neoplasms
Preceptor Handout
Clinical Case:
A 56 year-old G0 presents to the clinic with complaints of intermittent vaginal bleeding.
She went through the menopause 2 years ago and had no vaginal bleeding until 6
months ago when she had a three day episode of light bleeding. Since that time, she has
had another 3 such episodes. Past medical history is remarkable for well-controlled
hypertension, depression and borderline diabetes for which the patient is on a diet to
which she is minimally adherent. She never used contraceptives but was unable to
become pregnant. She has had a laparoscopic choleycystectomy. She takes Lisinopril
and Zoloft. Her family history is non-contributory. On examination, she has normal
vital signs, she weighs 247 pounds. Her heart, lungs and abdominal exams are normal.
On pelvic examination, she has normal external genitalia, normal vagina and cervix.
The bimanual exam is difficult secondary to the patients habitus, but the uterus feels
slightly enlarged and no adnexal masses are palpable.
Discussion Questions:
1. What is your differential diagnosis for this patient?

Important to consider diagnoses that are common and those that are serious
Fibroids, polyps
Hormonally related bleeding
Endometrial hyperplasia or endometrial carcinoma

2. What is the etiology of endometrial cancer?


Excess of endogenous or exogenous estrogen unopposed by progestin
leading initially to endometrial hyperplasia
Continued unopposed estrogen causes complex hyperplasia with atypia and
ultimately endometrial carcinoma
3. What risk factors does this patient have for endometrial carcinoma?
Obesity
Nulliparity
Late age at menopause
Infertility
Diabetes and history of gallbladder disease
Other risk factors this patient does not have include use of menopausal
estrogens and tamoxifen and a family history of genetically linked cancers
such as hereditary colon cancer and ovarian cancer

4. What factors are protective against endometrial cancer?

The major protective factor is use of combined oral contraceptives. A benefit


is seen after 3 months but is most prominent after long-term use of
combination oral contraceptives. This is an under-recognized benefit of
combined oral contraceptives.

5. What are the next steps in the diagnostic work-up of this patient?

Ultrasound
Endometrial biopsy

6. You obtain an ultrasound which shows a 10 cm uterus and no adnexal masses.


Endometrial biopsy reveals complex hyperplasia with atypia. Are further diagnostic
tests indicated?
Controversy re: need for formal D&C in the OR
15-40% of patients with diagnosis of endometrial hyperplasia have coexisting early stage endometrial cancer
Diagnosis/treatment is primarily surgical although medical management is
used in exceptional cases
7. How would you manage this patient?

Pre-operative work-up: Physical exam and CXR as staging is primarily


surgical
Intra-operative: Staging procedures include pelvic washings, TAH/BSO,
lymph node dissection, except in women with early stage disease (like this
patient)
In early stage disease (Stage1, Grade1) hysterectomy with BSO is sufficient
Stage 1 Limited to uterus
Stage 2 Tumor invades cervix but not beyond
Stage 3 Local or regional spread
Stage 4 Distant metastases
Post-operative: Depends on stage of endometrial cancer, most common
adjuvant therapy is radiation: vaginal brachytherapy
Non-surgical management is sometimes chosen by nulliparous women and
by women with such major co-morbidities that surgery is not an option
Treatment with high dose progestin (provera, megace)
Most respond but diligent follow-up essential given high recurrence

References:
Obstetrics and Gynecology by Beckmann 5th Edition, 2006; Chapter 46 Endometrial Hyperplasia and
Cancer. Pages 455-463.
Essentials of Obstetrics and Gynecology by Hacker and Moore 4th Edition, 2004; Chapter 42 Uterine
Corpus Cancer. Pages 478-485.

TEACHING CASE 54
Ovarian Neoplasms
Student Handout
Clinical Case:
A 48 year old G3P3 woman comes to the office for a health maintenance exam. She has
no concerns. She is in good health. She had three normal vaginal deliveries and
underwent a tubal ligation after the birth of her third child 15 years ago. She has no
history of abnormal Pap smears or sexually transmitted diseases. Her cycles are
regular and her last menstrual period was 18 days ago. She is not taking any
medications. Her family history is significant for a maternal aunt who was diagnosed
with ovarian cancer at age 60. On examination, she has normal vital signs. Her heart,
lungs and abdominal exams are normal. On pelvic examination, she has normal
external genitalia, normal vagina and cervix. On bimanual exam, she has a slightly
enlarged uterus and a palpable right adnexal mass which is confirmed on the vaginorectal exam.
Discussion Questions:
1. What is the next step in the management of this patient?
2. How would your approach be different if the patient was postmenopausal at 62
years of age?
3. You obtain an ultrasound which shows a 6 cm right complex ovarian cyst. What is
your differential diagnosis?
4. What risk factors does this patient have for ovarian cancer?
5. List physical exam elements which help support or rule out the diagnosis of ovarian
cancer?
6. Assuming the mass is persistent and you need to surgically explore the patient,
describe the pre-operative, intra-operative and post-operative management of this
patient.

References:
Obstetrics and Gynecology by Beckmann 5th Edition, 2006; Chapter 47 Ovarian and Adnexal Masses.
Pages 464-476.
Essentials of Obstetrics and Gynecology by Hacker and Moore 4th Edition, 2004; Chapter 40 Ovarian
Cancer. Pages 459-468.

Ovarian Neoplasms
Preceptor Handout

Adnexal masses are a common finding in both symptomatic and asymptomatic patients.
Management is based on determining the origin and character of these masses.

The APGO Educational Objectives related to this topic are the following:
A. Outline the approach to a patient with an adnexal mass *
B. Compare the characteristics of functional cysts, benign ovarian neoplasm
and ovarian malignancies *
C. Describe the symptoms and physical findings of a patient with ovarian
malignancy *
D. List the risk factors for ovarian cancer *
E. Describe the histological classification of ovarian neoplasm
F. Counsel a woman with risk for ovarian cancer *

*Designated as Priority One in the APGO Medical Student Educational Objectives, 8th
Edition

Ovarian Neoplasms
Preceptor Handout
Clinical Case:
A 48 year old G3P3 woman comes to the office for a health maintenance exam. She has
no concerns. She is in good health. She had three normal vaginal deliveries and
underwent a tubal ligation after the birth of her third child 15 years ago. She has no
history of abnormal Pap smears or sexually transmitted diseases. Her cycles are
regular and her last menstrual period was 18 days ago. She is not taking any
medications. Her family history is significant for a maternal aunt who was diagnosed
with ovarian cancer at age 60. On examination, she has normal vital signs. Her heart,
lungs and abdominal exams are normal. On pelvic examination, she has normal
external genitalia, normal vagina and cervix. On bimanual exam, she has a slightly
enlarged uterus and a palpable right adnexal mass which is confirmed on the vaginorectal exam.
Discussion Questions:
1. What is the next step in the management of this patient?

Need to discuss the work up of an adnexal mass in a pre-menopausal patient.


If cystic, mobile, and less than 10 cm, observe (+/- oral contraceptives) and
repeat USN in 6-8 weeks, if persists than surgical exploration, if gone or
smaller than functional cyst.
If solid, fixed, size >10 cm, or bilateral, then surgical exploration.

2. How would your approach be different if the patient was postmenopausal at 62 years
of age?

Needs surgical exploration

3. You obtain an ultrasound which shows a 6 cm right complex ovarian cyst. What is
your differential diagnosis?

Need to discuss the differential diagnosis for an ovarian mass, benign and
malignant.
Benign:
o Functional cyst
o Endometrioma
o Serous/mucinous cystadenoma
o Gonadal stromal tumors
o Germ cell tumors (teratomas)

Malignant:
o Epithelial tumors (serous, mucinous, clear cell, endometrioid,
Brenner)
o Germ cell tumors (dysgerminoma, endodermal sinus tumor, immature
teratoma)
o Sex cord stromal tumors (Sertoli-Leydig, Granulosa)

4. What risk factors does this patient have for ovarian cancer?

Need to discuss the risk factors for ovarian cancer.


Risk Factors:
o Positive Family History
o History breast cancer
o Nulliparity
o Age at 1st pregnancy
o Talc
o Obesity
o OCP use decreases risk

5. List history and physical exam elements which help support or rule out the diagnosis of
ovarian cancer.

Need to discuss presenting symptoms of ovarian cancer.


Presenting symptoms for epithelial ovarian cancer:
o Abdominal discomfort (50%)
o GI (20%)
o urinary (15%)
o vaginal bleeding (15%)
o weight loss (15%)
o Symptoms may mimic pregnancy
o Germ cell tumors may present with acute pain
o Precocious pseudopuberty and virilization seen with some germ cell
and sex cord/stromal tumors

6. Assuming the mass is persistent and you need to surgically explore the patient,
describe the pre-operative, intra-operative and post-operative management of this
patient.

Need to discuss pre-operative work-up, staging of ovarian cancer, and


treatment.
Pre-operative work-up: CXR, LFTs, tumor markers, BE, IVP
Intra-operative: Staging procedure includes TAH/BSO, nodes, omentectomy,
cytoreduction
Post-operative: chemotherapy

References:
Obstetrics and Gynecology by Beckmann 5th Edition, 2006; Chapter 47 Ovarian and Adnexal Masses.
Pages 464-476.
Essentials of Obstetrics and Gynecology by Hacker and Moore 4th Edition, 2004; Chapter 40 Ovarian
Cancer. Pages 459-468.

UNIT SIX:

HUMAN SEXUALITY

55. Sexuality and Modes of Sexual Expression

CLINICAL CASE 55
Sexuality & Modes of Sexual Expression
Case 1
At the conclusion of this exercise, the student will be able to provide a preliminary
assessment of patients with sexual concerns and make referrals when appropriate.
D.S.D. is a 29-year-old G2P1 who comes to see you because of decreased sex drive and
pain with intercourse. She reports that since the birth of her child about one year ago,
her sexual relationship with her husband never returned to normal. She does admit to
being very stressed out lately because she started a new job six months ago and she is
trying to balance it out with being a mother. She also reports being very tired most of
the time.
Physical exam:
Young woman in no distress who starts crying when asked about her relationship with
her husband. The rest of her physical exam is all normal.
Laboratory:
Hematocrit and TSH normal.
Assessment:
Decreased sex drive, possible depression
Plan:
Careful history and physical exam.
Rule out any possible underlying medical problems, e.g. hypothyroidism.
Counseling:
Possible referral to a therapist.

Discussion:
Careful history and physical exam need to be done to rule out any possible underlying
medical problem such as anemia, hypothyroidism or diabetes. Also need to rule out
substance abuse, such as smoking or alcohol, or any other medications that can affect
sexual function.

This patient most likely has underlying emotional problems and possible depression.
She needs to be counseled on marriage expectations in light of her exhaustion and
ability to cope with new and stressful responsibilities. Couple counseling might also be
helpful and consider referral to a sex therapist.
Teaching points:
1. Need to rule out any underlying medical conditions which can affect libido, such
as diabetes, hypertension, depression, or the presence of smoking or substance
abuse. Need to also check medications list for ones that can affect sexual
function.
2. Need to do a thorough physical exam, especially gynecologic, to rule out any
pathology, such as PID or endometriosis.
3. Decide the extent of the problem to determine if it can be managed by PCP or
need referral to sex therapist. Single disorders of less than 1 year duration in the
context of a stable relationship are more likely to be amenable to simple
interventions. Patients with one or more problems in the context of unstable
relationship are best referred to a sex therapist.

Case 2
At the conclusion of this exercise the student will be able to discuss how a womans means
of sexual expression affects the detection and management of her health care problems.
L.W. is a 33-year-old G0 who comes to the office for health maintenance exam. Her
previous medical and surgical history is negative. She has regular menses, has never
had a history of abnormal Pap smears. Her last one was 2 years ago. She lives with Lisa,
her current partner of 6 months. She smokes 1 pack per day and has for the last 10
years. She does not use alcohol or any other drugs. She is on no medications. She has
no complaints and wants to know what screening measures she needs.
Physical exam:
Young healthy woman in no distress.
Exam all normal.
Laboratory:
Pap smear normal

Assessment:
Healthy woman, smoker
Plan:
-History and physical exam
-Pap smear
-Counseling regarding smoking cessation, discuss exercise and calcium intake
-Discuss desires for fertility

Discussion:
This is a young healthy woman whose only issue is smoking. Her sexual orientation
does not put her at any additional risk for STDs, but does not exclude her from
contracting a sexually transmitted disease. She should be counseled, like any other
patient, that she needs a yearly Pap smear. You also need to be sympathetic to lesbian
issues, so she is comfortable discussing any relationship problems.
Teaching points:
1. There are no definitive psychological or dynamic traits that differentiate lesbian
women from heterosexual women except the wish for psychological and
physical intimacy with another woman.
2. Lesbian women are less likely than are gay men to engage in promiscuous sexual
encounters and run less risk of HIV transmission. While contraception is not an
issue, protection from sexually transmitted disease is a concern and should be
discussed where relevant.
3. Be prepared to discuss desire to have/raise children.

UNIT SEVEN:

VIOLENCE AGAINST WOMEN

56. Sexual Assault


57. Domestic Violence

CLINICAL CASE 56
Sexual Assault
At the conclusion of this exercise, the student will be able to provide a preliminary
assessment and discuss management of patients subjected to sexual assault.
A 24-year-old woman was seen in the emergency department at 2:00 am for alleged
sexual assault. She reported that she was on a blind date and began heavy petting at
about midnight. This continued for quite some time, and the date would not stop his
advances. She stated that she had been held down while her clothes were removed and
then was forced to have sex. She was very tearful, distraught, agitated and admitted
that she had been drinking alcohol. The emergency department staff was very busy with
a number of other patients. The gynecologist finally arrived at 4:00 am, although the
patient had arrived at 2:00 am. No female assistant or stand-by could be found for
another hour, so the examination began around 5:30 am.
Physical exam:
Minor abrasions on the patients back and several fresh areas of ecchymoses were
noted on her upper arms and breasts. The sexual assault kit materials were used, and
specimens were obtained from the vagina, urethra, rectum and oral cavity for analysis
and culture. Fingernail clippings, hair brushings and acid phosphatase swabs from the
vaginal discharge were obtained. The vaginal discharge was examined and motile
sperm were noted on a wet preparation.
Laboratory: Cultures, swabs and blood work.
Assessment:
Sexual assault
Plan:
Careful history and physical exam
Specimen collection/sexual assault kit
Take photographs of any injuries
Fill out and sign sexual assault chain-of-custody form
Notify social services, the assault crisis center and the police
Arrange for proper follow-up
Discussion:
Rape has traditionally been defined by law as forcible vaginal penetration without
consent. However, this is changing in many states. Many states have redefined rape so
that both men and women can be either victim or perpetrator. Some states have

degrees of rape, others limit admissibility of a victims previous sexual conduct, and
others have limited the requirement that the victims testimony be corroborated by
other evidence. Reforms continue. This case exemplifies date rape case in which
consent for sexual intercourse cannot be presumed without a clear expression of
consent, and because of mere acquiescence to sexual intercourse.
Rape is a crime that is seldom witnessed. Therefore, it is very important that the
gathering of corroborating evidence during the medical examination be done in such a
way that prosecution can be undertaken. The history and physical examination should
be conducted within a standardized legal format. Most hospitals have sexual assault kits
for this purpose. The physician should use these instruments precisely and should
express no opinions, conclusions or diagnosis in the record. The record should describe
the physicians findings and examination methods in detail.
Informed consent should be obtained before the examination that will allow
photographs and passage of information to authorities, as well as a routine
examination. There may be reporting requirements in the jurisdiction of your practice,
especially if the patient is a minor. The examination should involve general inspection
for signs of trauma, evaluation of external genitalia and a vaginal speculum
examination. Certain features are critical. Any external lesions should be photographed.
The external genitalia should be carefully inspected. Sexual dysfunction is common
among rapists, and failure to ejaculate or erectile failure may make internal vaginal
fluid specimens unhelpful. In such instances, signs of soft tissue trauma to the genitalia
may be the only corroborating evidence. Likewise, acid phosphatase assays from
vaginal secretions may be very helpful when the rapist has had a vasectomy and the
semen contains no sperm. Fifty percent of tests run 12 hours after intercourse show
acid phosphatase concentrations in the normal range, making the timing of the
examination important, as well. Blood serotyping and genetic screenings are becoming
important aspects of sexual assault evaluations.
All specimens should be collected in the presence of a witness and taken directly to the
pathologist. Anyone participating in this system of collection and transport must sign a
chain-of-custody statement to avoid mistakes or exchanges of specimens. Any break in
the chain of custody makes it impossible to prosecute a case. It is important for
physicians to be ready to testify in court if prosecution is attempted. Hearsay rules may
prevent evidence alone from the examination to be admitted, although this is less
common when evidence is properly labeled. This can be the first contact with the legal
system for the obstetrician-gynecologist. In this setting, the physician serves as a
patient advocate and an expert witness within the system, making it an ideal
introduction to law. Over time, proper legal management of a sexual assault evaluation
may be as therapeutic as appropriate medical management, which could include longterm psychotherapy.

Teaching points:
1.

In sexual assault cases, the physician has two clear duties: 1) medical
treatment of the patient; and 2) collection and preservation of evidence.

2.

The evidence must be collected with care and completeness, and chain-ofcustody requirements must be maintained or such evidence may be
inadmissible in court.

3.

Having available persons from social services who can provide immediate
counseling and emotional support in an emergency department setting is
often helpful.

CLINICAL CASE 57
Domestic Violence
At the conclusion of this exercise, the student will be able to provide a preliminary
assessment and discuss management options of patients subjected to domestic violence.
A.W. is a 25-year-old G4P3 woman who makes an appointment to consult you about her
PMS. She complains that she is not herself for several days before her period and
that she cant stop crying. She snaps irritably at her husband, who is a good provider
for [her] and for the children. She doesnt have her usual patience with the children,
aged 3 years, 2 years, and 8 months. She startles easily and is clumsy. Just last month,
she accidentally broke a favorite figurine her mother had given her. She thinks she
might have felt better when she took birth control pills, but her husband doesnt think
she should take drugs that interfere with natural functions, especially with this PMS
problem. He doesnt like IUDs, diaphragms or condoms, either. When you screen the
patient for depressive symptoms, she denies appetite disturbance. Her sleep is
somewhat fitful, but she has to keep an ear cocked to hear the children so that they
wont disturb her husband, as he has to get to work in the morning. Her energy and
sex drive are not great, but she believes thats pretty natural with three children. She
manages to keep up with them and the housekeeping. She enjoys seeing other people,
but doesnt have much time for socializing.
Her husband, who has accompanied her to the appointment, confirms her account. He
says she would be a great little wife and mother if it werent for this darned PMS. He
remains in the examining room throughout the interview and general physical and
pelvic exams, holding his wifes hand and patting her on the back from time to time. He
tells you that you shouldnt worry about the cost of treatment because he wants his wife
to have whatever she needs.
Physical exam:
All normal, except a small bruise on A.W.s right arm.
Laboratory:
Hematocrit and TSH normal.
Assessment:
Possible depression, protective husband, bruise on arm, possible domestic violence
Plan:
Careful history and physical exam
Rule out any possible underlying medical problems, e.g. hypothyroidism
Review previous medical records
Counseling
Possible referral to therapist or social worker

Discussion:
A.W.s symptoms do seem to occur in the premenstrual phase. It is not clear whether
she meets criteria for major depression or dysthymic disorder as well, but since the
treatment for PMS is selective serotonin reuptake inhibitors (SSRIs), which are
antidepressants, you reason that you will treat the depression, if it is present.
After ordering a laboratory workup, you prescribe sertraline 50 mg/day. You also
arrange to have her previous medical records sent to your office. Prior to A.W.s return
visit 2 weeks later, you review her records and notice that she has made numerous
visits to physicians with vague complaints of headaches and abdominal pains over the
years. She has also been seen in the emergency department for a succession of
lacerations and broken bones. Bruises were noted on these visits, but always explained
by the patient.
When A.W. appears for her visit, again accompanied by her husband, you ask the office
nurse and clerk to engage him in a lengthy discussion of insurance benefits. When he is
not present, you tell A.W. that you are glad to have the opportunity to speak with her
alone. She indicates that the medication has not made much of a difference in her
symptoms. You tell her that people sometimes have symptoms like hers when others in
their home are hurting them, and that you have noticed many injuries in her past
medical history. A.W. looks very frightened. You assure her that you are there to help
and that you will keep her statements strictly confidential. A.W. breaks down in tears
and tells you that her husbands temper sometimes gets the best of him, and she says,
He would kill me if he knew I had told anyone. You assure A.W. that no one has the
right to hurt anyone and discreetly provide her with information about domestic
violence. After several visits in the company of her husband, A.W. comes alone one day
and tells you, It took me a while to face the fact that I was being abused and to get up
the nerve to leave, but one day my husband hit our oldest daughter, and I realized I had
to get out. The kids and I are living with my mother now, and I am going to school so
that I can take care of us and make us a new life.

Teaching points:
1.

An overly involved husband is often a sign of domestic violence.

2.

A history of domestic violence is seldom volunteered, especially on the


first visit, and it will be necessary to interview the patient alone in order
to obtain the history.

3.

Victims of domestic violence frequently present with a succession of


rather vague physical complaints.

S-ar putea să vă placă și